NBME 30

Pataasin ang iyong marka sa homework at exams ngayon gamit ang Quizwiz!

184 Exam Section 4: Item 34 of 50 National Board of Medical Examiners' Comprehensive Basic Science Self-Assessment 34. A 70-year-old man comes to the emergency department 2 hours after the sudden onset of severe left-sided low back pain that radiates to his abdomen. He has a 25-year history of hypertension and type 2 diabetes mellitus. He is in severe distress. His pulse is 110/min, and blood pressure is 80/60 mm Hg. Physical examination shows diaphoresis and a soft, nontender abdomen; there is no tenderness of the back or spine. Neurologic examination shows no abnormalities. Urinalysis shows no blood. Which of the following is the most likely diagnosis? A) Aortic aneurysm rupture B) Diverticulitis C) Lumbar disc herniation D) Pyelonephritis E) Renal colic

A. A ruptured abdominal aortic aneurysm classically presents as sudden onset abdominal or back pain, with vital signs reflecting hemorrhagic shock (tachycardia, hypotension). Abdominal aortic aneurysms are more commonly seen in male smokers with a positive family history and a history of atherosclerotic disease. Other risk factors include hypercholesterolemia, hypertension, and trauma. They commonly occur in the infrarenal abdominal aorta and may present as a palpable, pulsatile abdominal mass if large and in a thin patient. Ruptured aneurysms require emergent surgical intervention and repair. Incorrect Answers: B, C, D, and E. Diverticulitis (Choice B) presents with left lower quadrant abdominal pain, sometimes accompanied by diarrhea, fever, and tenderness to palpation in the left lower quadrant. Treatment is generally antibiotic therapy and bowel rest. Lumbar disc herniation (Choice C) often occurs following chronic disc degeneration combined with a minor traumatic event such as bending or lifting a heavy object causing an acute increase in the intradiscal pressure. Acute disc herniation presents as sudden back and extremity pain, with potential neurologic symptoms (eg, paresthesias, weakness) in a dermatomal and myotomal distribution related to the impinged spinal nerve. Pyelonephritis (Choice D) refers to an infection of the kidney that commonly results from retrograde spread of a urinary tract infection. Patients present with fever, chills, and flank pain. Urinalysis may indicate bacteriuria, pyuria, and hematuria. Renal colic (Choice E) describes the sudden onset of unilateral flank pain that radiates to the lateral abdomen or groin with intermittent, severe spikes in intensity. It is associated with nephrolithiasis and obstruction of the ureter. Patients typically present with hematuria on urinalysis. Education

1 Exam Section 1: Item 1 of 50 National Board of Medical Examiners® Comprehensive Basic Science Self-Assessment 1. Shortly after delivery, a full-term male newborn is found to have black hair with a white forelock. His mother, a brunette, also has a white forelock and wears hearing aids. Physical examination shows heterochromia of irides. Otoacoustic emissions testing and brain stem auditory evoked responses show bilateral sensorineural hearing loss. Which of the following is the most likely cause of the findings in this patient? A) Abnormal neural crest development B) Abnormality of connexins C) Deficiency of homogentisic acid oxidase activity D) Deficiency of tyrosinase activity E) Failure of internalization of melanin granules by keratinocytes F) Failure of melanosome transportation along dendrites

A. Abnormal neural crest development leads to Waardenburg syndrome. Waardenburg syndrome is a syndrome of patchy depigmentation of the skin, hair, irises, and cochlear dysfunction that primarily illustrates an autosomal dominant inheritance pattern. Because of genetic mutations of genes encoding transcription factors, neural crest cells do not properly differentiate into melanoblasts (melanocyte precursors), or melanoblasts do not migrate to their appropriate location. Patients typically have a white forelock and eyelashes, depigmented skin patches, iridic heterochromia, and sensorineural deafness. The eyes may also be laterally displaced. The clinical diagnosis may be confirmed with genetic testing. Treatment includes audiologic evaluation and genetic consultation. Incorrect Answers: B, C, D, E, and F. An abnormality of connexins (Choice B) would lead to abnormal formation of the plasma membrane channels of diverse cell types. Different combinations of sensorineural hearing loss, ichthyosis, alopecia, and peripheral neuropathy may occur. Depigmentation would be atypical. Deficiency of homogentisic acid oxidase activity (Choice C) would lead to decreased metabolism of the amino acids phenylalanine and tyrosine, which instead degrade into homogentisic acid. Homogentisic acid accumulates in the skin and joints, causing increased pigmentation and arthritis, respectively. Depigmentation would be atypical. Deficiency of tyrosinase activity (Choice D) occurs in oculocutaneous albinism, which presents with uniformly hypopigmented hair and skin (versus the patchy depigmentation of Waardenburg syndrome) and eye abnormalities (eg, iris hypopigmentation, refractive errors, nystagmus). In tyrosinase deficiency, melanocytes are unable to synthesize melanin from the amino acid tyrosine. Iridic heterochromia and sensorineural deafnes

188 Exam Section 4: Item 38 of 50 National Board of Medical Examiners' Comprehensive Basic Science Self-Assessment 38. A previously healthy 55-year-old man comes to the physician because of a 2-day history of pain with urination, urinary frequency, low back pain, and nausea. His temperature is 38.5°C (101.3°F), pulse is 88/min, respirations are 20/min, and blood pressure is 144/88 mm Hg. Physical examination shows acute low back tenderness. Urinalysis shows 3+ protein; numerous WBCS, epithelial cells, and bacteria; and a few RBCS. Treatment with a quinolone antibiotic is initiated, but his symptoms do not resolve. Urine culture grows Escherichia coli resistant to ciprofloxacin. The most likely cause of this resistance is a mutation in which of the following elements? A) DNA gyrase B) DNA polymerase C) RNA polymerase D) 30S ribosomal subunit E) 50S ribosomal subunit

A. Ciprofloxacin is a fluoroquinolone antibiotic. Its mechanism of action is to inhibit prokaryotic DNA gyrase (also known as topoisomerase), an enzyme necessary for DNA replication. If a bacterium develops a mutation within DNA gyrase which prevents the fluoroquinolone from binding to it, then it will become resistant to the fluoroquinolone class of antibiotics. Such a mutation could be transferred between bacteria via transposable genetic element. A transposable genetic element, also known as a transposon, describes a segment of DNÃ that can move position within or across chromosomes or between a chromosome and plasmid; in this way, genetic material can be transferred. Plasmids are extrachromosomal segments of double-stranded DNA that are commonly seen in bacteria; they may contain genes that confer survival advantages to the host and are usually classified by their function. One type of plasmid is the resistance plasmid, which contains an antibiotic resistance gene. This is the genetic material which confers antibiotic resistance to the host bacterium and is one potential mechanism of developing fluoroquinolone resistance in Escherichia coli. Incorrect Answers: B, C, D, and E. DNA polymerase (Choice B) is an enzyme targeted by acyclovir, famciclovir, ganciclovir, foscarnet, and cidofovir which are all antiviral medications. This enzyme does not play a role in fluoroquinolone function nor in resistance to fluoroquinolones. RNA polymerase (Choice C) is inhibited by the antibiotic rifampin. Mutation in this protein would cause antibiotic resistance to rifampin, not ciprofloxacin. Antibiotics which inhibit the 30S ribosomal subunit (Choice D) include aminoglycosides (eg gentamicin, neomycin, amikacin) and tetracyclines (eg, tetracycline, doxycycline, minocycline). A mutation in this protein complex would not affect the

98 Exam Section 2: Item 48 of 50 National Board of Medical Examiners® Comprehensive Basic Science Self-Assessment 48. A 25-year-old man who immigrated to the USA from Pakistan 2 months ago comes to the physician for a follow-up examination. He was diagnosed with hepatitis C at the age of 5 years but has had no symptoms or clinical signs of this condition. Physical examination shows no abnormalities. Serum studies done today show an AST activity of 68 U/L, ALT activity of 100 U/L, and hepatitis C virus RNA of 1.5 million IU/mL. A photomicrograph of the liver biopsy specimen is shown. Which of the following characteristic features of virus-induced histopathology is seen in this patient? A) Apoptosis B) Coagulation necrosis C) Endoplasmic reticulum proliferation D) Immune-mediated cell lysis E) Mallory hyaline inclusion

A. Apoptosis is demonstrated in the photomicrograph provided, which shows a councilman body (eosinophilic cytoplasm) in the lower left quadrant of the slide. Councilman bodies represent hepatocytes in a state of apoptosis or necrosis, and their distinct appearance histologically contrasts them against normal surrounding cells and architecture. They possess a small nuclear fragment but no distinct nucleus and an intensely eosinophilic cytoplasm. They can be identified both in acute and chronic viral hepatitis, although in chronic hepatitis they may be associated with features of chronic injury, including spotty necrosis, multinucleated hepatocytes, and Kupffer cell hyperplasia. More severe forms of hepatic injury, including bridging and confluent necrosis, are less common in chronic hepatitis C. Incorrect Answers: B, C, D, and E. Coagulation necrosis (Choice B), in contrast to apoptosis, is cell death that occurs as a result of ischemic injury and involves distinctly different cellular pathways. It is histologically characterized by absent nuclei but preserved cell architecture that may persist for several days, followed by cellular destruction via heterolysis. Endoplasmic reticulum (ER) proliferation (Choice C) may occur in cellular ischemia because of cessation of the ATP dependent sodium-potassium transporter in the plasma membrane; increased quantity of ER may also occur in states of high protein synthesis (eg, in plasma cells), and in cases of considerable or repeated exposure to toxic substances degraded by the smooth ER (eg, in hepatocytes). This is not seen on this photomicrograph. Immune-mediated cell lysis (Choice D) describes several distinct processes, but commonly involves the binding of an antibody to the surface of cells, leading either to complement fixation with activation of the classical complement ca

113 Exam Section 3: Item 13 of 50 National Board of Medical Examiners' Comprehensive Basic Science Self-Assessment 13. A 5-year-old girl is brought to the physician because of a 4-day history of stumbling gait, muscle rigidity, slurred speech, and decreased cognitive function. Ophthalmologic examination shows decreased visual acuity and inability to discern colors. Assays of isolated muscle mitochondria show decreased ability to oxidize NADH but normal FADH, oxidation. Which of the following electron transport chain complexes is most likely impaired in this patient? A) I B)II C) II D) IV E) V

A. Complex I, NADH oxireductase, transfers electrons from NADH to ubiquinone while generating a proton gradient. For every two electrons transferred from NADH, four protons are pumped across the membrane. Complex I is a common site of electron leakage and free radical generation. Mutations in genes encoding complex I are commonly implicated in the pathogenesis of Leber Hereditary Optic Neuropathy (LHON), a mitochondrial-inherited condition that results in degeneration of the optic nerve and retina. LHON presents at variable age, typically with acute central field visual loss, often in one eye, followed by loss in the contralateral eye. Optic nerve atrophy and permanent loss of visual acuity result. An afferent pupillary defect may be present prior to complete visual loss, and color vision loss plus the development of a central scotoma is common. LHON has also been associated with a neurodegenerative variant in which patients also present with spasticity, ataxia, tremors, and cardiac arrhythmias. On the basis of mitochondrial inheritance, children of affected mothers will be affected regardless of gender, whereas those of affected fathers and normal mothers will be phenotypically normal. Incorrect Answers: B, C, D, and E. II (Choice B) accepts electrons and protons from FADH2, forming FAD. In this example, the analysis showed normal FADH2 oxidation. III, IV, and V (Choices C, D, and E) do not play a role in accepting electrons from NADH or FADH2. Educational Objective: Complex I, NADH oxireductase, transfers electrons from NADH to ubiquinone while also generating a proton gradient. For every two electrons transferred from NADH, four protons are pumped across the membrane. Mutations in genes encoding complex I are commonly implicated in the pathogenesis of Leber Hereditary Optic Neuropathy, a mitochondrial-inherited cond

140 Exam Section 3: Item 40 of 50 National Board of Medical Examiners' Comprehensive Basic Science Self-Assessment 40. A2-year-old girl is brought to the emergency department by her parents because of a 1-day history of respiratory difficulty. She visited her grandparents 2 days ago; they live on a farm with well water and a septic system. Physical examination shows cyanosis, tachypnea, tachycardia, and subcostal and intercostal retractions. A drop of blood from a fingerstick is brown and remains the same color when placed on a piece of filter paper. Arterial blood gas analysis on room air shows a Po, and pH within the reference ranges. Her oxygen saturation is decreased. Which of the following is the most likely cause of this patient's condition? A) Impaired binding of O, to hemoglobin B) Increased NADH production C) Polymerization of mutant hemoglobin D) Stimulation of the hexose-monophosphate shunt pathway E) Unba

A. Methemoglobinemia occurs when an abnormal fraction of hemoglobin in the blood exists in the ferric (Fe3-) form (methemoglobin). Methemoglobin cannot effectively bind oxygen, which impairs tissue oxygen delivery. Most cases of methemoglobinemia are acquired, typically caused by exposure to an oxidant stressor such as an anesthetic (eg, benzocaine), nitrate, or dapsone. The patient in the question most likely had exposure to nitrates through contaminated agricultural groundwater from nitrogen-based fertilizer. Symptoms include shortness of breath, fatigue, lethargy, lightheadedness, and in severe cases, arrhythmia, seizure, and multi-system organ failure. Vital signs typically show reduced oxygen saturation, and examination may show cyanosis. Characteristically, blood appears darkened on gross visualization. Treatment includes supplemental oxygen plus intravenous administration of methylene blue, a reducing agent that donates electrons to heme. Reduction to the ferrous (Fe2+) form restores the ability of hemoglobin to bind, carry, and deliver oxygen to tissues. Incorrect Answers: B, C, D, and E. NADH is produced by glycolysis and the tricarboxylic acid cycle. NADH delivers protons and electrons to the electron transport chain, which uses them in oxidative phosphorylation to produce ATP. Increased NADH production (Choice B) would not result in hypoxia or methemoglobinemia. Polymerization of mutant hemoglobin (Choice C) is characteristic of the pathogenesis of sickle cell disease. Sickle cell disease is caused by mutations in globin genes that result in an abnormal hemoglobin S (HbS), which polymerizes in hypoxia and causes sickling of erythrocytes. This can result in sickle cell pain crisis, splenomegaly, and acute chest syndrome. The hexose-monophosphate shunt pathway (Choice D), also known as the pentose phosphate pa

158 Exam Section 4: Item 8 of 50 National Board of Medical Examiners' Comprehensive Basic Science Self-Assessment 8. During an experimental study, a glutamate antagonist is administered to 100 patients with a history of acute cerebral infarction to target the N-methyl-D-aspartate receptor. Which of the following is the intended result of this therapy? A) Decreased calcium entry into neurons O B) Enhanced synaptic release of glutamate C) Increased removal of cellular debris by glial cells D) Promotion of vasodilation E) Stimulation of glutamate transporter mechanisms

A. Decreasing calcium entry into neurons leads to decreased excitotoxicity, which has been investigated as a pharmacologic target to promote recovery from acute cerebral infarction. Excitotoxicity describes the phenomenon in which neurons endure additional damage or cell death beyond an initial inciting insult such as a traumatic brain injury, stroke, spinal cord injury, or neurodegenerative disease. In response to damage to the central nervous system (CNS), glutamate may be released rapidly into the synaptic clefts. Glutamate binds N-methyl-d-aspartic acid (NMDA) receptors, which allows the influx of sodium and calcium cations and depolarizes the neuron, leading to the frequent firing of action potentials. Unchecked excitatory stimuli can result in very high intracellular calcium concentrations with resultant activation of apoptotic phospholipases and proteases such as calpain. Calcium also accumulates in the mitochondria, leading to mitochondrial dysfunction and the release of apoptotic signals. Consequently, antagonism of the NMDA receptor could protect neurons against excitotoxicity. NMDA antagonists such as memantine are utilized in dementia because of their neuroprotective role; they have not yet been translated into stroke treatments. Incorrect Answers: B, C, D, and E. Enhanced synaptic release of glutamate (Choice B), which occurs during ischemic stroke, leads to excitotoxicity and consequent neuronal apoptosis. A substance that increases synaptic glutamate concentration would further increase cell death and would therefore not be beneficial. Increased removal of cellular debris by glial cells (Choice C) occurs days after acute cerebral infarction and ultimately results in a glial scar rather than neuronal recovery. NMDA receptors do not directly control the phagocytosis of neuronal debris by glial cells. The p

111 Exam Section 3: Item 11 of 50 National Board of Medical Examiners' Comprehensive Basic Science Self-Assessment 11. A 23-year-old woman comes to the physician because of a 3-week history of frequent thirst and urination; she also has had a 3-kg (6.6-lb) weight loss during this period. Physical examination shows dehydration and tachypnea. Serum studies show a glucose concentration of 330 mg/dL, 2+ ketones, and pH of 7.2. Following the administration of intravenous fluids and insulin, there is marked improvement. The activity of which of the following enzymes has most likely increased in this patient's hepatocytes because of this treatment? A) Glucokinase B) Glucose 6-phosphatase C) Glycogen phosphorylase D) Phosphoenolpyruvate carboxykinase E) Phosphorylase kinase

A. Diabetic ketoacidosis (DKA) occurs because of accumulation of serum ketones such as beta-hydroxybutyric acid, acetoacetic acid, and acetone in the setting of insulin deficiency. Dysregulated fatty acid degradation and ketone synthesis occurs because of the relative state of starvation created through absent insulin signaling. Patients present with complaints of fatigue, polyuria, shortness of breath, and diffuse abdominal discomfort. Vital signs may show tachycardia and tachypnea. Laboratory studies typically show increased serum glucose and ketones, metabolic acidosis, and a high anion-gap because of the accumulation of unmeasured anions in serum. Treatment is through replacement of insulin, volume repletion, correction of electrolyte abnormalities, and treatment of any precipitating conditions. Insulin promotes the peripheral tissue uptake of glucose, glycolysis, and hepatic glucose storage as glycogen, resulting in decreased glucose concentration in the serum. Glucokinase is an enzyme that phosphorylates glucose to yield glucose-6-phosphate as the first step of hepatic glycolysis and glycogen synthesis when serum glucose concentration is increased. It is upregulated by insulin. Incorrect Answers: B, C, D, and E. Glucose 6-phosphatase (Choice B) is an enzyme located in the liver that functions in gluconeogenesis and glycogen degradation and converts glucose-6-phosphate to glucose for its release into the blood. It is downregulated by insulin signaling. Glycogen phosphorylase (Choice C) is stimulated by glucagon and functions in glycogenolysis. During periods of fasting or low serum glucose, glycogenolysis serves to maintain serum glucose concentration and prevent hypoglycemia. Glycogenolysis is downregulated by insulin. Phosphoenolpyruvate carboxykinase (Choice D) is an enzyme that catalyzes the irreversible conve

180 Exam Section 4: Item 30 of 50 National Board of Medical Examiners' Comprehensive Basic Science Self-Assessment 30. A screening program was instituted for detection of depressive symptoms among college students. At the first examination, evidence of depression was found in 250 of 1250 subjects. During the following year, an additional 100 students developed depressive symptoms. Which of the following is the annual incidence of depression in this population of students? A) 10% B) 16% C) 20% D) 28%

A. Incidence is an important epidemiological measure that assesses the rate of occurrence of new disease in an at-risk population. The incidence rate of disease in a population equals the number of new cases of the disease divided by the number of persons at risk for contracting the disease. Incidence is a measure of risk and is the probability of a given condition occurring within a specified period of time. It is often expressed as a single number of new cases but is more appropriately expressed as a fraction of a population. Importantly, when individuals are added to or are removed from a population, the denominator must also change to reflect the new number of persons at risk for contracting the condition. The incidence rate is mathematicaly expressed as (the number of new cases)/(the number of persons at risk). In this case, an initial 250 cases of depression are identified in a population of 1250 subjects. In the following year, only 1000 subjects are at risk for developing depression, because the other 250 subjects have already developed it. This becomes the denominator for the incidence. Over the year, 100 individuals develop depression, which supplies the numerator. The incidence of depression over the year is therefore 10% (100/1000). Incorrect Answers: B, C, and D. 16% (Choice B) would be correct if the number of students developing depression over the year was 160. However, the number of students is 100 and thus the incidence is calculated as 100/1000 = 0.1 (10%). 20% (Choice C) is the prevalence of depression in the subject population at the initiation of the study. Prevalence is defined as the number of cases of a condition as a fraction of a population at a point-in-time. Mathematically, prevalence is calculated by dividing the number of existing cases by the number of people in the population under stud

58 Exam Section 2: Item 8 of 50 National Board of Medical Examiners® Comprehensive Basic Science Self-Assessment 8. One day after being admitted to the hospital for treatment of peritonitis, a 42-year-old man has progressive shortness of breath and moderate distress. Pulse oximetry on room air shows an oxygen saturation of 53%. Bilateral basilar crackles are heard on auscultation. Acute lung injury is suspected. The patient is intubated and mechanically ventilated with 90% oxygen at a lung-protective tidal volume of 550 mL and respirations of 16/min. Positive end-expiratory pressure also should be implemented to prevent which of the following complications? A) Absorption atelectasis B) Alveolar overdistention C) Oxygen toxicity D) Pleural exudate formation E) Pulmonary hypertension F) Tension pneumothorax

A. Mechanical ventilation is indicated for respiratory failure. Positive end-expiratory pressure (PEEP) may be utilized to prevent the pressure in the alveoli from dropping to zero or becoming negative during the respiratory cycle. This helps keep alveoli stented open and able to participate in gas exchange. The patient is suffering acute lung injury and has impaired gas exchange caused by fluid filling the alveoli, resulting in absorption atelectasis. Oxygenation can be improved by implementing positive end-expiratory pressure to resist alveolar collapse and atelectasis. Incorrect Answers: B, C, D, E, and F. Alveolar overdistention (Choice B) is a potential complication of mechanical ventilation from excessive alveolar pressure. Large tidal volumes and shear stress from alveoli collapsing and opening can result in inflammatory cytokine release and secondary ventilator- induced lung injury. Lung-protective tidal volume strategies reduce the risk for alveolar overdistension. Oxygen toxicity (Choice C) is a potential complication of mechanical ventilation with a high inspired fraction of oxygen. Cell damage occurs secondary to reactive oxygen species, which can worsen lung injury. Pleural exudate formation (Choice D) is not prevented by positive end-expiratory pressure. Underlying causes for exudative pleural effusion include infection, malignancy, and autoimmune disorders. Pulmonary hypertension (Choice E) has numerous potential causes. It may be secondary to conditions such as left heart disease, chronic hypoxemic lung disease, and connective tissue disorders. Positive end-expiratory pressure is not used to prevent pulmonary hypertension apart from reducing transient hypoxic vasoconstriction. Tension pneumothorax (Choice F) is a potential complication of mechanical ventilation, especially with large tidal volumes or hi

t disease) B) Osteosarcoma C) Osteomalacia D) Osteomyelitis O E) Osteoporosis

A. Osteitis deformans (

137 Exam Section 3: Item 37 of 50 National Board of Medical Examiners' Comprehensive Basic Science Self-Assessment 37. A 55-year-old woman is brought to the physician because of a 6-month-history of progressive difficulty walking. Neurologic examination shows spasticity, hyperreflexia, and clonus in the lower extremities. Babinski sign is present bilaterally. A drug with which of the following mechanisms of action will most likely decrease the spasticity in this patient? A) Activation of y-aminobutyric acid receptors in muscle spindle afferents B) Activation of glutamate receptors in gamma motoneurons C) Activation of serotonin receptors in alpha motoneurons D) Inhibition of glycine receptors in Golgi tendon organ afferents E) Inhibition of nicotinic receptors in Renshaw cells F) Inhibition of a, receptors in excitatory interneurons

A. Muscle spindles are stretch receptors located within the muscle which sense changes in the length of a muscle fiber. When rapidly stretched, a reflex arc that consists of an afferent signal transmitted to the spinal cord with direct synapse on efferent neurons results. This arc triggers rapid contraction of the stretched muscle. Arcs are normally modulated by upper motor neuron inhibition, but when this is lost in cases of upper motor neuron dysfunction (such as following a stroke or in the setting of multiple sclerosis) uncontrolled hyperreflexia, spasticity, and clonus results. Muscle contraction is controlled by signal transduction from upper and lower motor neurons, by the motor end plate, and by myocyte calcium release and excitation-contraction coupling. In this patient, progressive difficulty ambulating along with spasticity, hyperreflexia, and clonus suggests a loss of upper motor neuron inhibitory control over spinal reflex arcs, which results in an exaggerated response to muscle stretch. The transmission of the stretch reflex is inhibited by y- aminobutyric acid (GABA) signaling in the afferent neuron, which is the pathway acted upon by muscle relaxants such as baclofen. Incorrect Answers: B, C, D, E, and F. Activation of glutamate receptors in gamma motoneurons (Choice B) would result in the maintenance of a taut muscle spindle by activating the gamma motoneuron. The gamma motoneuron principally controls the spindle apparatus; activation of the neuron by glutamate (an excitatory neurotransmitter) would not decrease spasticity of the muscle itself as it would not diminish the sensitivity of the reflex arc. Activation of serotonin receptors in alpha motoneurons (Choice C) may inhibit the stretch reflex as it has been shown to inhibit glutamate release onto motor neurons. However serotonergic signaling is in

109 Exam Section 3: Item 9 of 50 National Board of Medical Examiners® Comprehensive Basic Science Self-Assessment 9. A 36-year-old man is brought to the emergency department 30 minutes after he sustained burns in a house fire. He is 185 cm (6 ft 1 in) tall and weighs 86 kg (190 lb); BMI is 25 kg/m2. Physical examination shows second-degree burns over approximately 40% of the body surface area. Intravenous fluids are administered, and he is admitted to the intensive care unit for supportive care. Compared with a healthy individual, which of the following best represents this patient's nutritional and fluid requirements? Nutrition Fluid A) Increased increased B) Increased unchanged O C) Increased decreased D) Unchanged increased OE) Unchanged unchanged OF) Unchanged decreased G) Decreased increased H) Decreased unchanged 1) Decreased decreased

A. Severe partial- or full-thickness, also called second- or third-degree, burns compromise the ability of the skin to maintain fluid balance. Exposure to a hot environment such as a fire can result in large insensible evaporative fluid losses to the external environment because of the compromise of the epidermal barrier, while the cutaneous burns themselves result in third-spacing and intravascular losses to dermal edema. These losses are further exacerbated by tachypnea or airway burns resulting in additional insensible losses and pulmonary edema, respectively. Fluid resuscitation in severe burns is achieved through administration of isotonic crystalloid solution, which is the standard of care. The amount is guided by the Parkland formula, which calculates the amount of fluid to administer over the first day based on the percentage of body surface area burned (partial or full thickness only) and the weight of the patient. Resuscitation should be additionally guided based on clinical parameters including hemodynamic stability, glomerular filtration rate, electrolyte disturbances, and urine output. Nutritional requirements are also increased in burn patients because the burn injury causes a hypermetabolic state and increased stress response. For patients with severe or extensive burns, parenteral nutrition may be required because of inability to consume sufficient calories to maintain metabolic balance. Failure to meet nutritional needs can lead to impaired wound healing, increased risk for infection, and organ dysfunction. Incorrect Answers: B, C, D, E, F, G, H, and I. This patient with extensive burns will require increased nutrition, not unchanged (Choices D, E, and F) or decreased (Choices G, H, and I) nutrition. Increased caloric intake and availability is needed to meet the needs of a burn patient's hypermetaboli

55 Exam Section 2: Item 5 of 50 National Board of Medical Examiners® Comprehensive Basic Science Self-Assessment 5. A 66-year-old man is brought to the emergency department 30 minutes after the sudden onset of substernal chest pain that radiates to the neck and left arm; the pain is associated with weakness, nausea, and profuse sweating. He was jogging during the onset of pain. He has a 5-year history of hypertension treated with indapamide. His pulse is 90/min, and blood pressure is 150/90 mm Hg. Cardiac examination shows an S, Treatment with sublingual nitroglycerin resolves his pain within 2 minutes. An ECG shows a 2-mm ST-segment elevation in the anterior leads. Treatment with a fibrinolytic drug is beneficial to this patient because of which of the following mechanisms of action? A) Catalyzing the formation of plasmin B) Catalyzing the platelet Ilb/llla receptor complex C) Inhibition of ADP-induced platelet-fib

A. Myocardial infarction classically presents with acute-onset chest pain, often radiating to the neck, jaw, or arm, along with shortness of breath, nausea, and/or lightheadedness. Risk factors include a history of smoking, diabetes mellitus, hypertension, hyperlipidemia, and/or obesity. If the infarction results in left ventricular dysfunction, patients may experience cardiogenic pulmonary edema resulting in shortness of breath, tachypnea, hypoxia, audible rales, and S3 or S4gallops. Diagnosis is made using clinical history, analysis of cardiac biomarkers, and an ECG, which may show ST-segment elevation or depression, inverted T-waves, or Q-waves. Treatment requires the use of antiplatelet agents (eg, aspirin, clopidogrel) plus anticoagulants (eg, heparin), pain control, and revascularization through angioplasty, thrombolysis, or coronary artery bypass grafting. For patients who cannot receive percutaneous coronary intervention in a timely manner, thrombolytics are indicated. Thrombolytics such as tissue plasminogen activator function by catalyzing the formation of plasmin from plasminogen. Plasmin is a serine protease that cleaves fibrin clots. Incorrect Answers: B, C, D, E, and F. Catalyzing the platelet Ilb/Illa receptor complex (Choice B) is not the mechanism of action of thrombolytics. The platelet Ilb/Illa receptor binds to fibrinogen during the process of platelet aggregation. Inhibition of ADP-induced platelet-fibrinogen binding (Choice C) occurs with ADP receptor blockers (eg, clopidogrel) that inhibit expression of the Ilb/llla receptor and with monoclonal antibodies (eg, abciximab) that directly bind the receptor. Inhibition of conversion of fibrinogen to fibrin (Choice D) is the mechanism of heparin anticoagulants and direct thrombin inhibitors such as lepirudin and bivalirudin. Thrombin catalyzes the conv

173 Exam Section 4: Item 23 of 50 National Board of Medical Examiners® Comprehensive Basic Science Self-Assessment 23. A74-year-old woman is admitted to the hospital to undergo a cholecystectomy. During the operation, she becomes acutely hypotensive. The hypotension persists for 5 minutes despite administration of appropriate pressor agents. Which of the following morphologic lesions is most likely to occur in this patient as a result of this incident? A) Enlarged neurons in the hippocampus B) Infarction of the pineal gland C) Lacunar infarcts of the basal ganglia D) Posterior pituitary necrosis E) Rupture of a berry aneurysm

A. The hippocampus, located in the mesial temporal lobe, is a highly susceptible region of the brain to hypoxic injury, which may occur during acute hypotensive episodes because of decreased cerebral perfusion. The CA1 region of the hippocampal cortex is particularly vulnerable to ischemia because of its high metabolic demands. During ischemia, the blood supply to the brain is interrupted, resulting in decreased adenosine triphosphate (ATP) production and consequent dysfunction of sodium transporters. Sodium accumulates within the cell, which drives the influx of extracellular fluid, defined as cytotoxic edema. Cytotoxic edema may occur within minutes of hypoxia and results in the enlargement of neurons within the affected region. Incorrect Answers: B, C, D, and E. Infarction of the pineal gland (Choice B) typically occurs because of a cardiac or aortic thromboembolism. The hippocampus would be more likely injured by hypoxia than the pineal gland. Lacunar infarcts of the basal ganglia (Choice C) result from chronic hypertension and diabetes mellitus, which lead to endothelial damage with subsequent hyaline thickening and sclerosis of the arterial wall. These risk factors ultimately result in blockage of perforating arteries and consequent ischemic damage. Acute hypotension is not a risk factor for lacunar infarcts. Posterior pituitary necrosis (Choice D) may occur in the setting of postpartum hemorrhage as part of a condition of pituitary hypofunction called Sheehan syndrome. In pregnancy, the pituitary gland undergoes hyperplasia and demonstrates increased metabolic demands, making the pituitary gland (particularly the anterior pituitary gland) vulnerable to ischemic damage. In non-pregnant patients, the pituitary gland is not especially vulnerable to ischemic damage. Rupture of a berry aneurysm (Choice E) typically r

72 Exam Section 2: Item 22 of 50 National Board of Medical Examiners® Comprehensive Basic Science Self-Assessment 22. An investigator is conducting a study of the effect of salt intake on renal function. The diet of an experimental animal is modified to contain only 25% of normal salt content. Which of the following is most likely to be increased in this experimental animal? A) Epithelial sodium channel transcription B) Extracellular fluid C) Plasma atrial natriuretic peptide concentration D) Serum potassium concentration E) Urine pH

A. The nephron is characterized by an apical epithelial membrane that faces the tubular lumen and a basal layer that faces the interstitium. This epithelial membrane, whether in the convoluted tubules, loops, or collecting duct is the site of reabsorption of solute or water into the blood. The collecting duct is the terminal segment of the nephron and reabsorbs sodium and water while secreting potassium and protons, primarily through epithelial sodium channels and aquaporins. It is the site of action of aldosterone. Increased dietary salt intake suppresses aldosterone, while decreased dietary salt intake increases aldosterone activity. Aldosterone increases epithelial sodium channel transcription and activity, resulting in increased sodium reabsorption. Incorrect Answers: B, C, D, and E. Increased dietary sodium can lead to fluid accumulation in extracellular tissue. Decreased dietary salt would be unlikely to increase extracellular fluid (Choice B). Plasma atrial natriuretic peptide (ANP) concentration (Choice C) would likely be decreased with low salt intake. ANP is a hormone secreted by atrial cardiac myocytes in response to increased hydrostatic pressure. It results in diuresis, natriuresis, and decreased blood pressure. Decreased dietary salt intake generally decreases systolic blood pressure in hypertensive individuals, whereas increased dietary salt intake would be more likely to increase blood pressure and therefore plasma ANP concentration. Decreased dietary salt would increase aldosterone activity, which increases epithelial sodium channel activity and apical potassium secretion, leading to decreased, not increased, serum potassium concentration (Choice D). Aldosterone causes increased H+-ATPase activity, resulting in H* secretion into the urine. This would decrease, not increase, urine pH (Choice E). Educati

161 Exam Section 4: Item 11 of 50 National Board of Medical Examiners® Comprehensive Basic Science Self-Assessment 11. A 10-year-old boy who has had type 1 diabetes mellitus for 1 year is receiving insulin. One hour after his morning dose of insulin, he becomes tremulous and diaphoretic and has tachycardia. Several hours later, his symptoms resolve. His blood glucose concentration is now increased. Which of the following is the most likely cause of this patient's hyperglycemia? A) Activation of hepatic adenylyl cyclase B) Activation of muscle glycogen synthase C) Activation of muscle phosphorylase D) Activation of muscle protein phosphatase E) Inhibition of hepatic protein kinase A

A. This patient presents with symptoms of hypoglycemia which are generally neuroglycopenic (eg, agitation, headache, blurry vision, diplopia, tremor, loss of consciousness), autonomic (eg, diaphoresis, tremoring, nausea, vomiting, cramping, tachycardia, anxiety), and nonspecific (eg, fatigue, weakness, lethargy); symptoms are often exacerbated by periods of fasting or exercise. The patient's tremulousness, diaphoresis, and tachycardia are likely caused by hypoglycemia, a common adverse effect of insulin administration in patients with type 1 diabetes mellitus. Resolution of hypoglycemic symptoms is likely because of glucagon release from the pancreas and cortisol release from the adrenal gland, with their resultant effects on gluconeogenesis and glycogen degradation. The pancreas releases glucagon during episodes of hypoglycemia. Glucagon acts on hepatocytes via a cyclic adenosine monophosphate pathway, stimulating adenylyl cyclase and activating protein kinase A, which activates glycogenolysis and gluconeogenesis. The liver possesses the ability to release free glucose into the serum during the process of glycogenolysis as it possesses the enzyme glucose-6-phosphatase, which hydrolyzes glucose-6-phosphate to create free glucose molecules. Incorrect Answers: B, C, D, and E. Activation of muscle glycogen synthase (Choice B) would occur in glycogenesis, not glycogenolysis. Glycogenesis would not be stimulated in a hypoglycemic patient, and if it were occurring, it would contribute to further hypoglycemia. Activation of muscle phosphorylase (Choice C) and muscle protein phosphatase (Choice D) occurs during glycogenolysis in the muscle to release glucose monomers to be used in skeletal muscle glycolysis. Muscle does not release free glucose into the serum as occurs during hepatic glycogenolysis. Inhibition of hepatic prote

50 Exam Section 1: Item 50 of 50 National Board of Medical Examiners® Comprehensive Basic Science Self-Assessment 50. The graph shows the mean steady state plasma concentration of Drug X as a function of time in a subject who has normal weight (solid curve) and a subject who is obese (dashed curve). Both subjects take 10 mg of Drug X daily. Which of the following best describes the disposition of Drug X in the obese patient compared with the patient who has normal weight? A) Greater volume of distribution B) Higher clearance C) Lower bioavailability D) Slower absorption rate E) Shorter half-life 1 3 4 6 7 Time (days)

A. Volume of distribution is the theoretical volume in which a drug exists given its plasma concentration following a dose of medication. There are three main compartments into which a medication can distribute: the intravascular compartment, the interstitial compartment, and the intracellular compartment (including fat). With an increased volume of distribution, the same dose of medication will lead to a smaller plasma concentration (ie, it will distribute more widely into tissue), and it will take longer for the medication to reach a steady state within the plasma. Large molecules or protein-bound molecules generally remain within the vasculature and quickly reach a steady state concentration given the relatively low volume of the intravascular compartment. Smaller hydrophilic molecules tend to move into the interstitial space, which increases their volume of distribution and increases the amount of time it takes for the medication to reach a steady state concentration. Small lipophilic molecules, however, exhibit the largest volume of distribution as they are commonly taken up by adipocytes, which greatly increases the volume in which they are distributed. Therefore, it takes these molecules longer to reach a steady state concentration, especially in obese patients in which the volume of distribution is even greater given the presence of increased fat stores, as seen in this graph. Incorrect Answers: B, C, D, and E. Higher clearance (Choice B) of a medication indicates that more medication is removed from the body in a given period of time. Differences in clearance are typically because of changes in hepatic or renal function and are commonly displayed in a graph that shows the decline in drug concentration. An increase in clearance with the same volume of distribution would require a higher maintenance dose to reac

77 Exam Section 2: Item 27 of 50 National Board of Medical Examiners® Comprehensive Basic Science Self-Assessment 27. A61-year-old man has had three episodes of painless gross hematuria during the past week. The left flank is mildly tender to palpation. Urinalysis shows blood but no WBCS, bacteria, or cellular casts. He says to the physician, "Tell me like it is. Do I have cancer?" Which of the following is the most appropriate response? A) "Cancer is one of the possible causes of blood in your urine. We will need to perform additional tests to determine the cause." B) "Hematuria can be caused by many things. Trust me. I will let you know if we find anything worrisome." C) "It is too soon to tell. Don't worry about that for now." D) "Yes, cancer looks likely, but many effective treatments are now available." E) "Yes, I'm afraid this usually is due to cancer. Would you like to have someone with you while we discuss i

A. When patients express the desire to be informed about serious diagnoses, physicians should inform patients directly and compassionately. This physician should inform the patient that his hematuria may reflect an underlying cancer but that the diagnosis is not yet confirmed. Once the patient has been informed that this diagnosis is a possibility, the physician should validate the patient's emotions by listening receptively, using reflective statements, inferring emotions from the expressed content, and/or normalizing the patient's experience. The physician should also deliver education and explain the next steps using small pieces of information. Incorrect Answers: B, C, D, and E. Informing the patient that his diagnosis is unclear (Choices B and C) would avoid the patient's question about whether he has cancer. The physician should respond that cancer is a possibility. Answering the patient's question about whether he has cancer with a definitive "yes" (Choices D and E) would be premature. Further discussion and treatment planning should occur only after the cancer diagnosis is definitively made. Educational Objective: When patients express the desire to be informed about serious diagnoses, physicians should inform patients directly and compassionately about possible or confirmed diagnoses. The physician should also deliver education and explain the next steps using small pieces of information. %3D Previous Next Score Report Lab Values Calculator Help Pause

52 Exam Section 2: Item 2 of 50 National Board of Medical Examiners® Comprehensive Basic Science Self-Assessment 2. A 22-year-old man is brought to the physician because of a 1-week history of bleeding gums and bruising of his legs and a 3-month history of weakness and fatigue. His temperature is 37°C (98.6°F). Physical examination shows pallor, boggy gingival hypertrophy, and several petechial lesions and ecchymoses scattered over the lower extremities. The spleen tip is palpated 3 cm below the left costal margin. Laboratory studies show: Hemoglobin Hematocrit 8.2 g/dL 24% Leukocyte count Platelet count 32,000/mm3 23,000/mm3 Microscopic examination of a peripheral blood smear shows 42% blast forms. Which of the following is the most likely diagnosis? A) Acute lymphocytic leukemia B) Acute myelogenous leukemia C) Chronic lymphocytic leukemia D) Chronic myelogenous leukemia E) Hairy cell leukemia

B. Acute myelogenous leukemia (AML) is the most likely diagnosis in this patient with pancytopenia, gingival hypertrophy, and 42% blasts on peripheral smear. AML is a neoplasm arising from cells within the myeloid lineage, which is distinct from the lymphoid lineage. There are myriad mutations that can be present in malignant myeloid cells and the types of mutations confer different risk profiles. Patients with poor risk mutations undergo stem cell transplant if they are able to achieve remission after induction and consolidation chemotherapy with cytarabine and idarubicin. Common presenting symptoms include fatigue, malaise, fevers, and weight loss. Gingival hypertrophy is commonly seen in a particular subtype of AML known as acute promyelocytic leukemia and is a result of infiltration of the gums with malignant cells. Leukemia cutis, a rash that represents infiltration of the skin by malignant cells, can also occasionally be seen. Other findings include petechiae or purpura from thrombocytopenia and anemia from diffuse marrow infiltration by blasts. Induction therapy for AML is considered urgent and is usually done with cytarabine and idarubicin, which is followed by consolidation therapy. Patients with poor risk mutations invariably go on to stem cell transplant if they can achieve remission initially. Incorrect Answers: A, C, D, and E. Acute lymphocytic leukemia (ALL) (Choice A) is primarily a malignancy of childhood but is occasionally seen in adults. Cytopenias are common and peripheral smear also shows blasts as in this patient, although these blasts will be of the lymphoid lineage and appear different from myeloid cells. Gingival hypertrophy is not generally associated with ALL. Chronic lymphocytic leukemia (Choice C) is a malignancy of mature, differentiated B lymphocytes. Common findings include lymphocytosis

40 Exam Section 1: Item 40 of 50 National Board of Medical Examiners® Comprehensive Basic Science Self-Assessment 40. An 18-month-old boy has pinched, wrinkled facies and total loss of subcutaneous fat. He is alert. His height is at the 50th percentile and his weight is at the 2nd percentile for his age. The liver is not enlarged, and there is no edema. Which of the following is the most likely diagnosis? A) Kwashiorkor B) Marasmus C) Pellagra D) Rickets E) Scurvy

B. Acute protein-energy malnutrition is characterized clinically by a decrease in mid-upper arm circumference, weight for length, or body mass index. Unlike in chronic malnutrition, children maintain their linear growth because the weight is more quickly impacted than the height. Acute malnutrition can be divided into two groups based on the presence or absence of edema. Marasmus, or wasting syndrome, occurs when the overall caloric intake is deficient, but no specific vitamins or minerals are missing. Wasting of muscle mass and consumption of body fat stores is characteristic. It is not associated with edema. Affected children appear emaciated and weak and may be irritable. Autonomic functions may suffer, causing bradycardia, hypotension, and hypothermia. Redundant skin folds, as seen in this case, arise because of loss of subcutaneous fat. In contrast, kwashiorkor is also a form of protein-energy malnutrition but is accompanied by edema. The edema begins in dependent areas but, with continued malnutrition, progresses to involve the presacral, genitalia, and preorbital areas. Children with either marasmus or kwashiorkor malnutrition have complications in nearly every organ system of the body including immune dysfunction, increased cortisol concentrations, decreased intestinal motility and absorptive capacity, and a tenuous fluid balance predisposing to acute heart failure. Incorrect Answers: A, C, D, and E. Kwashiorkor (Choice A), also known as edematous malnutrition, is differentiated from marasmus by the presence of edema. Skin lesions and fatty liver, both absent in this case, are typically present in kwashiorkor. Pellagra (Choice C) is caused by an acquired deficiency of niacin (nicotinic acid, vitamin B3). It is characterized by diarrhea, dementia, and dermatitis. Signs of protein-energy malnutrition are not pres

152 Exam Section 4: Item 2 of 50 National Board of Medical Examiners' Comprehensive Basic Science Self-Assessment 2. A 42-year-old man develops supraventricular tachycardia 15 minutes after undergoing an abdominal operation. An intravenous infusion with an ultrashort-acting B-adrenoreceptor blocking agent is started. This drug is most likely which of the following? A) Atenolol B) Esmolol C) Metoprolol D) Propranolol E) Sotalol

B. B-adrenoreceptor antagonists are used for the treatment of supraventricular tachycardia because of their ability to slow atrioventricular node conduction. Multiple agents are available, with varying activity at B, and B2 receptors, pharmacokinetics, and pharmacodynamics. Esmolol is the most rapid-acting B-adrenoreceptor blocking agent, with a rapid onset and short half-life. These properties make it useful for the immediate and titratable treatment of various conditions, including supraventricular tachycardia, aortic dissection, hypertensive emergency, and thyrotoxicosis. It is available in an intravenous form only and must be administered in a closely monitored setting. Incorrect Answers: A, C, D, and E. Atenolol (Choice A) and metoprolol (Choice C) are Brselective antagonists with greater activity at B, receptors than B2 receptors. They are used in the treatment of hypertension, arrhythmias, and heart failure. Propranolol (Choice D) is a nonselective B antagonist with similar activity at B, and B2 receptors. It may be used in the treatment of hypertension, arrhythmias, migraines, essential tremor, and hyperthyroidism. Sotalol (Choice E) is a nonselective B antagonist that also displays type III antiarrhythmic properties with activity at cardiomyocyte potassium channels, which increases ventricular action potential duration, effective refractory period, and prolongs the QT interval. It is used in the treatment of ventricular arrhythmias and refractory atrial fibrillation. There is an increased risk for torsades de pointes. Educational Objective: B-Adrenoreceptor blocking agents are common medications used for the treatment of hypertension, congestive heart failure, and tachyarrhythmias. Esmolol is the most rapid-acting agent in the class and exhibits a short half-life. %3D Previous Next Score Report Lab Values Calc

19 Exam Section 1: Item 19 of 50 National Board of Medical Examiners® Comprehensive Basic Science Self-Assessment 19. A 13-year-old girl is brought to the physician by her mother because of a mass in her armpit for 3 days. She has a history of several similar masses since early childhood that have resolved with treatment. She also has had several episodes of pneumonia, rhinitis, and perianal abscesses since birth. Her temperature is 37°C (98.6°F). Physical examination shows a 2-cm, raised, red, tender, fluctuant mass in the right axilla. There is mild hepatosplenomegaly. A complete blood count shows a mildly increased leukocyte count. A chest x-ray shows bilateral interstitial infiltrates. A photomicrograph of lung tissue obtained on biopsy is shown. A nitroblue tetrazolium test shows deficient reduction by granulocytes. The axillary mass is incised and drained, and culture of the fluid grows Staphylococcus aureus

B. Chronic granulomatous disease (CGD) is the most likely diagnosis in this patient with a Staphylococcus aureus abscess, history of prior similar infections, and an abnormal nitroblue tetrazolium test. CGD is defined by deficiency of the NADPH oxidase complex, which is essential for normal neutrophil intracellular killing of pathogens. NADPH oxidase uses oxygen as a substrate for the generation of free radicals (superoxide anions). Free radical oxygen species are subsequently used for the creation of hydrogen peroxide and hypochlorous acid. Activation of this pathway leads to the respiratory burst which results in bacterial death. Deficiency of NADPH oxidase renders phagocytes incapable of neutralizing catalase-positive bacteria, which are capable of neutralizing their own hydrogen peroxide, thus leaving the host cells without the substrate necessary to complete the respiratory burst. Diagnosis is made by an abnormal dihydrorhodamine test or a nitroblue tetrazolium reduction test. In this latter test, normal phagocytes use the action of NADPH to reduce nitroblue, which leads to a color change from yellow to blue. Patients with CGD will not demonstrate color change. Recurrent pneumonia is the most common presenting infection in patients with CGD, and the most common infecting bacteria include Staphylococcus species, Aspergillus species, Burkholderia cepacia, and Nocardia species. Patients with CGD are also at risk for fungal infections, especially Aspergillus species. Incorrect Answers: A, C, D, and E. presentingChédiak-Higashi syndrome (Choice A) is a rare, autosomal recessive disorder of the immune system caused by mutations in the lysosomal trafficking regulator gene (LYST) that encodes a protein essential for normal formation and transportation of lysosomes within the cell. The clinical manifestations include freq

110 Exam Section 3: Item 10 of 50 National Board of Medical Examiners' Comprehensive Basic Science Self-Assessment 10. A 1-year-old boy receives a left cochlear implant that consists of an array of electrodes. Six months ago, he was diagnosed with severe bilateral hearing deficit. Which of the following elements of the auditory system must remain intact for this patient to perceive auditory stimulation via activation of the electrodes in the implant? A) Auditory hair cells B) Auditory nerve C) Ossicles D) Round window E) Tympanic membrane

B. Cochlear implants directly stimulate the auditory nerve endings in the cochlea and therefore require a functioning auditory nerve. Sensorineural hearing loss typically results from damage of the hair cells in the cochlea. Cochlear implants bypass the damaged cochlea. The external microphone of the implant converts mechanical sound waves to an electrical signal that is transmitted to an electrode within the cochlea located adjacent to the auditory nerve endings. Normally, sound transmission in the cochlea begins when the stapes footplate of the middle ear vibrates against the oval window of the cochlea (inner ear), which creates vibrations in the perilymph of the cochlea. These vibrations are ultimately transmitted to the Organ of Corti, which contains hair cells that generate electrical impulses and sends the impulses to the spiral ganglion of the cochlea (auditory nerve endings). Axons of the spiral ganglion unite to form the auditory nerve, and ultimately the signal is transmitted to the auditory cortex. In sensorineural hearing loss, the damaged hair cells can no longer convert mechanical vibrations to electrical impulses. Incorrect Answers: A, C, D, and E. The auditory hair cells (Choice A) and round window (Choice D) are components of the cochlea and therefore do not need to function for this patient to perceive auditory stimulation via a cochlear implant. Cochlear implants bypass the damaged cochlea by transmitting electrical signals directly to the auditory nerve endings. The round window connects the cochlea back to the middle ear and flexes backward when sound is transmitted, allowing the movement of fluid within the cochlea. The ossicles (Choice C) are the bones of the middle ear (ie, malleus, incus, stapes) that move in response to vibrations of the tympanic membrane (Choice E). The tympanic membrane (ie,

136 Exam Section 3: Item 36 of 50 National Board of Medical Examiners' Comprehensive Basic Science Self-Assessment 36. An otherwise healthy 25-year-old woman in the third trimester of an uncomplicated pregnancy has a hemoglobin concentration of 10.5 g/dL and a hematocrit of 30%. The mean corpuscular volume and mean corpuscular hemoglobin are within the reference range. Which of the following is the most likely cause of her anemia? A) Erythroblastosis fetalis B) Expansion of intravascular volume C) Folic acid deficiency D) Increased affinity of hemoglobin for O2 E) Intrauterine hemorrhage

B. Expansion of intravascular volume is a physiologic change seen beginning in the second trimester of pregnancy. The total blood volume increases by 50% with a concomitant rise in red blood cell production but to a lesser extent, leading to dilutional anemia which is typically asymptomatic. In pregnancy, the normal values for hemoglobin are adjusted accordingly. For example, a normal hemoglobin in a non-pregnant female is 12.0-16.0 g/dL. In contrast, hemoglobin >10.5 g/dL is considered normal in the second trimester. While this mild dilutional anemia of pregnancy is physiologic, there may be additional iron deficiency anemia which does require treatment. Physiologic changes occur during pregnancy in almost every organ system. Increased stroke volume, cardiac output, pulse, and blood pressure are all physiologic cardiovascular changes. Incorrect Answers: A, C, D, and E. Erythroblastosis fetalis (Choice A) occurs when a sensitized Rh-negative mother carries an Rh-positive fetus. Antibodies to Rh antigen may form during previous delivery or exposure of maternal antigen presenting cells to fetal red blood cells (eg, trauma, vaginal bleeding). When the mother becomes pregnant for a second time with an Rh-positive fetus, the antibodies to Rh antigen cross the placenta and bind Rh antigen on the fetal erythrocytes, leading to a potentially fatal hemolytic anemia in the fetus, not the mother. Pregnancy is a risk factor for folic acid deficiency (Choice C) because of the state of high cellular turnover. Folic acid deficiency leads to impaired DNA synthesis and is characterized by erythrocyte macrocytosis and hypersegmented neutrophils. Results of laboratory studies show anemia, increased mean corpuscular volume, and normal white cell and platelet indices. Increased affinity of hemoglobin for 02 (Choice D) is a characteristic o

150 Exam Section 3: Item 50 of 50 National Board of Medical Examiners' Comprehensive Basic Science Self-Assessment 50. A 21-year-old man is brought by ambulance to the emergency department 30 minutes after sustaining a gunshot wound to the leg. He is unconscious. His pulse is 150/min, respirations are 26/min and spontaneous, and blood pressure is 80/60 mm Hg. Physical examination shows pallor. Which of the following sets of changes is most likely in this patient compared with a normal baseline? Arteriovenous Plasma Lactic Acid Plasma ADH Systemic Capillary Fluid Transfer Po 2 Difference (Vasopressin) A) ↑ ↑ filtration B) ↑ ↑ reabsorption C) ↑ filtration D) ↑ ↑ filtration OE) ↑ reabsorption F) ↑ ↑ filtration G) ↑ ↑ reabsorption H) ↑ filtration |) ↑ reabsorption OJ) reabsorption

B. Hypovolemic shock occurs secondary to a significant reduction in intravascular volume, commonly related to hemorrhage, dehydration, or gastrointestinal volume losses. This patient has sustained a gunshot wound to the leg which has led to hemorrhagic hypovolemic shock. Decreased intravascular volume results in decreased cardiac preload and therefore stroke volume. Cardiac output is a product of stroke volume and pulse, so pulse increases to compensate for decreased preload and stroke volume. In addition, there is arteriolar vasoconstriction to increase vascular resistance in an attempt to maintain mean arterial pressure. If hemorrhagic shock is progressive and severe, compensation fails, which leads to the development of hypotension and inadequate organ perfusion, such as in this patient. The arteriovenous oxygen difference is an indication of how much oxygen is removed in the capillaries as blood circulates. In hemorrhagic shock, oxygen extraction in tissues is increased, leading to an increased arteriovenous PO2 difference. Furthermore, inadequate organ perfusion and oxygenation shifts metabolism to anaerobic metabolism and lactic acid production, which leads to an anion gap metabolic acidosis. Baroreceptors in the carotid sinus respond to hypotension by stimulating ADH release from the pituitary gland, increasing the expression of aquaporin channels in the collecting duct to increase reabsorption of free water to restore intravascular volume at the expense of osmolality. Systemic capillary fluid transfer describes the movement of fluid between the capillaries and surrounding tissues. In hypotension, capillary hydrostatic pressure is decreased, and fluid is reabsorbed from surrounding tissue into the intravascular circulation. Incorrect Answers: A, C, D, E, F, G, H, I, and J. Filtration in systemic capillary fluid

143 Exam Section 3: Item 43 of 50 National Board of Medical Examiners' Comprehensive Basic Science Self-Assessment 43. A 64-year-old man comes to the physician because of a 6-month history of progressive shortness of breath with exertion. He was diagnosed with a hematological malignancy 4 years ago. His pulse is 85/min and blood pressure is 115/60 mm Hg. Physical examination and chest x-ray show no abnormalities. Echocardiography shows a greater than normal echodense ventricular wall with normal thickness. Right-sided cardiac catheterization shows resistance to diastolic filling. A right ventricular endomyocardial biopsy specimen shows interstitial deposits of pink proteinaceous material and degeneration of myocytes. Which of the following is the most likely precursor of the proteinaceous material found in this patient's heart? A) Atrial natriuretic factor B) Lambda light chains C) BzMicroglobulin D) Serum amyloid A

B. Lambda light chain deposition from amyloidosis is the most likely cause of this patient's diastolic dysfunction and endomyocardial biopsy showing proteinaceous deposits and myocyte degeneration. Amyloidosis refers to the deposition of small molecular weight proteins in various tissues resulting in progressive organ dysfunction. These proteins, which are typically soluble in plasma, develop particular mutations that change their shape such that they assume a B-pleated sheet configuration, causing them to precipitate. They characteristically appear bright green on polarized light microscopy when stained with Congo red. Commonly, these protein fragments are derived from immunoglobulin light chains secreted by a monoclonal population of plasma cells; this syndrome is called AL amyloidosis. Diagnosis is supported by the presence of free light chains found in urine and/or plasma. End organ damage is largely dependent upon the location of amyloid protein deposition. In this patient, deposition is seen in the myocardium leading to diastolic dysfunction. This is a common finding in patients with AL amyloidosis and commonly contributes both to morbidity and mortality. Ôn echocardiography, the myocardium has a speckled appearance. Other findings may include proteinuria, hepatosplenomegaly, and carpal tunnel syndrome. Amyloid may also deposit in the tongue and cause macroglossia. Incorrect Answers: A, C, D, and E. Atrial natriuretic factor (Choice A) is a peptide secreted from the atria when the cardiac chambers are distended. It acts as a diuretic agent to excrete sodium via the kidneys in an attempt to restore normal circulating volume. While it may be increased in patients with heart failure, it does not cause heart failure. B2-Microglobulin (Choice C) can form insoluble fibrils in patients with end stage renal disease on h

128 Exam Section 3: Item 28 of 50 National Board of Medical Examiners® Comprehensive Basic Science Self-Assessment 28. In a patient with HIV-1 infection, the viral genome is most likely to be found in helper T lymphocytes and which of the following? A) Endothelial cells OB) Macrophages C) Natural killer cells O D) Neutrophils E) Suppressor T lymphocytes

B. Macrophages are the other cell type in addition to CD4+ T lymphocytes in which the HIV-1 viral genome is most likely to be found as they also possess CD4 and CCR5 co-receptor on their cell surface. To enter the cell, HIV must first bind to CD4 via the HIV gp120 and gp41 subunits. This binding induces a conformational change in the HIV envelope allowing for binding to a coreceptor such as CCR5 or CXCR4. Following this, the HIV envelope fuses with the cellular membrane and expels its contents into the host cell. The HIV RNA genome is subsequently reverse transcribed into DNA and inserted into the host cell genome via HIV integrase, followed by transcription of HIV genes and translation into HIV proteins. These proteins are packaged and cleaved to form the mature HIV virus. Following infection with HIV, macrophages may also may serve to perpetuate infection for several reasons: they are present in high numbers at sites of viral entry, are resistant to the cytotoxic effects of HIV infection allowing for HIV persistence and replication, and have a remarkably long life span with the ability to survive for months to years in peripheral tissues. They also distribute widely throughout the body, including the central nervous system. Incorrect Answers: A, C, D, and E. Endothelial cells (Choice A), natural killer cells (Choice C), and neutrophils (Choice D) do not express CD4 on their surface so HIV is unable to enter and infect these cell types. Suppressor T lymphocytes (Choice E), also known as regulatory T cells, are a different class of CD4+ T lymphocytes that can also acquire HIV infection. However, cell types beyond the T lymphocyte lineage that may be infected include macrophages more specifically. Educational Objective: HIV binds to CD4 to enter CD4+ T lymphocytes. Macrophages also possess CD4 and CCR5, which allows for

144 Exam Section 3: Item 44 of 50 National Board of Medical Examiners' Comprehensive Basic Science Self-Assessment 44. A 16-year-old girl comes to the clinic because of vaginal itching for the past 5 days. She requests that her parents not be informed about the appointment. During the examination, she says she occasionally has unprotected sexual intercourse and asks for oral contraceptives. Preliminary laboratory studies show gram-negative diplococci. Which of the following is the most appropriate next step in patient care? A) Consult a colleague for advice about treating a minor without parental consent B) Discuss with her the need for testing her for other sexually transmitted diseases C) Interview her sexual partners D) Notify her parents of the findings E) Treat her infection and refer her to social services

B. Minors (any person younger than 18 years) may be tested and treated for sexually transmitted diseases (STDS) without parental consent. Though parental consent is required for most medical care, exceptions include medical care related to sex (eg, STDS, contraception, pregnancy care), substance use, and life-threatening medical or psychiatric conditions. This physician should therefore honor the patient's wish to not inform her parents and instead encourage the patient to discuss these medical issues with her parents. Additionally, emancipated minors may make their own medical decisions. This patient likely has vaginitis caused by Neisseria gonorhoeae (a gram-negative diplococcus). The physician should discuss with her the need for testing for other STDS such as Chlamydia trachomatis and human immunodeficiency virus (HIV). Incorrect Answers: A, C, D, and E. Parental consent is not required for this patient's sex-related medical concerns (Choices A and D). The physician should honor the patient's desire not to inform her parents. Interviewing this patient's sexual partners (Choice C) is not necessary. This patient should be encouraged to share her STD diagnosis with her partner or partners. Referring this patient to social services (Choice E) would be inappropriate, as there is no evidence of child abuse or neglect. Many teenagers engage in sexual activity and should be counseled about the risks rather than referred to social services. Educational Objective: Parental consent is not required for medical care related to sex (eg, STDS, contraception, pregnancy care), substance use, or life-threatening medical or psychiatric conditions. %3D Previous Next Score Report Lab Values Calculator Help Pause

108 Exam Section 3: Item 8 of 50 National Board of Medical Examiners' Comprehensive Basic Science Self-Assessment 8. Alterations of the p53 tumor suppressor gene are most likely seen in patients who have which of the following neoplasms? A) Burkitt lymphoma B) Colonic adenocarcinoma C) Malignant melanoma D) Small cell undifferentiated carcinoma

B. Neoplasms of the colon (benign or malignant) have their molecular pathogenetic roots in the progressive accumulation of mutations that lead to disordered or dysregulated cell growth, loss of control of the cell cycle, evasion of immune detection or destruction, angiogenesis, and loss of DNA damage repair capability. Colonic neoplasia often begins with the formation of a polyp, a small growth that may be flat, sessile, or pedunculated. Histologically, polyps may be adenomatous (tubular, tubulovillous, or villous), serrated (premalignant), hamartomatous (disorganized growth of native tissue), or hyperplastic. Adenomatous polyps typically occur as a result of chromosomal instability; most precursor mutations involve the APC gene, a tumor suppressor. When both copies of APC are mutated, increased cellular proliferation may occur. Inherited mutations of APC are common in familial adenomatous polyposis syndrome. A second mutation that follows often involves the KRAS oncogene. KRAS codes for a signaling protein involved in MAP- kinase and PI3-K pathways that promote cell growth, cell differentiation, and cell survival. Only a single mutated allele in oncogenes (in contrast to the requirement for two mutated alleles in tumor suppressor genes) is needed to promote neoplasia. When constitutively active, uncontrolled growth and division can result, reflected grossly as an adenomatous polyp. Additional mutations in P53 (a tumor suppressor) that follow mutations in KRAS may permit sufficient loss of cell cycle control resulting in malignant transformation. This pathway (APČ, KRAS, and P53 mutations, generally in sequence) accounts for 80 to 85% of sporadic cases of colorectal carcinoma. The other 15% of cases are a result of microsatellite instability from DNA mismatch repair, including both sporadic mutations and inherited Lyn

105 Exam Section 3: Item 5 of 50 National Board of Medical Examiners' Comprehensive Basic Science Self-Assessment 5. The prevalence of a disease is three times greater in women than in men, but the incidence of the disease is no different in women than it is in men. Which of the following best explains these findings? A) The case fatality rate is higher among women B) The duration of the disease is shorter in men O C) The number of new cases in women is increasing D) The proportion of asymptomatic cases is higher in women E) Women use medical care facilities more often than men

B. Prevalence, as applied to biostatistics, defines the number of cases of a disease or condition as a fraction of a population in consideration at a point in time. Prevalence, therefore, indicates the percentage of current cases of a condition within the population under study at a given timepoint. Mathematically, prevalence is calculated by dividing the number of existing cases by the number of people in the population under study or at risk for the condition. In contrast, incidence is the number of new cases over the total population at risk over a specified period of time. Subjects who have already developed the condition at the initial time point are not at risk and thus not included in the denominator when calculating incidence. In the case of a chronic condition, prevalence will increase over a period of time and exceed incidence because subjects who develop the disease or condition will remain affected for a long period of time. In the case of an acute illness, cases will turn over and individuals will recover from the disease at a similar rate as new cases develop, so prevalence and incidence are similar. In this case, the prevalence of disease is much higher in women than men though the incidence rates are the same between the groups. This indicates that the condition has a longer duration in women than men. Incorrect Answers: A, C, D, and E. Case fatality is one variable which affects prevalence. If the case fatality rate were higher among women (Choice A), then prevalence would decrease because of cases leaving the total population as a result of death. It would be lower than that of men, not higher. An increasing number of new cases in women (Choice C) would be reflected in incidence, not prevalence. The incidence of the condition in men and women is the same, so the number of new cases in both groups is t

133 Exam Section 3: Item 33 of 50 National Board of Medical Examiners' Comprehensive Basic Science Self-Assessment 33. A 92-year-old woman who was recently admitted to a nursing home because of progressive dementia has large purpuric lesions over the dorsa of both forearms and hands. Her family states that she has had similar lesions on and off for years and that, to the best of their knowledge, these lesions occur in the absence of trauma. The woman appears well cared for and has no other indications of recent or remote injury. Her platelet count is normal. Which of the following is the most likely explanation for the purpura? A) Abuse by a family member B) Atrophy of dermal collagen C) Autoimmune disease D) Hereditary hemorrhagic telangiectasia E) Platelet dysfunction

B. Senile purpura, or actinic purpura, are benign patches of extravasated red blood cells which occur on sun-exposed areas in elderly patients. These occur because of atrophy of dermal collagen from age and actinic (sun-related) damage which causes the dermal vessels to become fragile and easily broken. As a result, even minor bumps or pressure on these areas causes dark purple bruising to develop. The most common locations are the dorsal hands and forearms. No treatment is necessary for the bruising as it will fade over time, but new bruises will also continue to form. Actinic purpura may be exacerbated by anticoagulant or anti-platelet agents, but these do not need to be present for the purpura to form. Incorrect Answers: A, C, D, and E. Bruising may be a sign of abuse by a family member (Choice A), but the distribution of purpura on the dorsal hands and arms and otherwise well-kempt appearance in this patient suggest actinic purpura as a more likely cause. Autoimmune disease (Choice C), including systemic lupus erythematosus and dermatomyositis, may cause erythematous patches to form in a sun-exposed distribution but these are not purpuric. The malar rash of systemic lupus erythematosus and facial, cervical, and thoracic erythema of dermatomyositis are two examples of this. Other autoantibody-mediated diseases, such as Henoch-Schönlein purpura, would not present with purpura only on the hands and forearms and would be unlikely to develop at an elderly age. Hereditary hemorrhagic telangiectasia (HHT) (Choice D), also known as Osler-Weber-Rendu syndrome, is an inherited blood vessel disorder. Cutaneous findings include mucocutaneous telangiectasias particularly on the lips, oral mucosa, and fingertips. Arteriovenous malformations form within the brain, lungs, liver, and intestines and can cause significant morbidity

127 Exam Section 3: Item 27 of 50 National Board of Medical Examiners' Comprehensive Basic Science Self-Assessment 27. A 65-year-old woman with stage I estrogen receptor-positive breast carcinoma undergoes a mastectomy and radiation therapy. After completion of the chemotherapy, she begins treatment with tamoxifen. The initial site of binding of this agent to its receptor most likely occurs in which of the following areas of the tumor cells? A) Cellular membrane B) Cytosol C) Endoplasmic reticulum D) Golgi apparatus E) Nucleus

B. Tamoxifen is a selective estrogen receptor modulator that is used in the treatment of breast cancer. It acts as an estrogen receptor antagonist in the breast but an estrogen receptor agonist in the endometrium and bone. Estrogen is a steroid hormone and is consequently lipophilic. Its receptor is located in the cytosol. Following binding to the receptor, the receptor-ligand complex translocates to the nucleus, where it binds to DNA to exert its effects. Because of tamoxifen's unopposed estrogen stimulation of the endometrium, it increases the risk for endometrial hyperplasia, with the potential for atypia and consequent endometrial cancer. Other risks of tamoxifen use include deep venous thrombosis and pulmonary embolism, because of the hypercoagulability caused by alterations in the coagulation system. It decreases the risk for recurrent invasive breast cancer in bilateral breasts by approximately 30 to 50%, although it results in little difference in overall mortality. Incorrect Answers: A, C, D, and E. Cellular membrane (Choice A) receptors typically bind to peptide hormones, among other ligands that are hydrophilic. These hormones include insulin and glucagon. Estrogen is a hydrophobic steroid hormone. Endoplasmic reticulum (Choice C) receptors include the ryanodine receptor and the inositol-1,4,5-triphosphate receptor, both of which stimulate the release of calcium into the cell. The estrogen receptor is not located in the endoplasmic reticulum. The golgi apparatus (Choice D) is responsible for carbohydrate synthesis, as well as for packaging and directing cellular proteins to various locations in the cell. It is not a common location of steroid hormone binding. Nucleus (Choice E) receptors are also bound by steroid hormones. In contrast to the receptor for estrogen, which remains in the cytosol until it binds

36 Exam Section 1: Item 36 of 50 National Board of Medical Examiners® Comprehensive Basic Science Self-Assessment 36. An investigator is studying Po, in an experimental animal. It is found that Po, in the renal vein is relatively high compared with venous Po, from most other organs. Which of the following best explains this finding? A) Bicarbonate reabsorption causes a Bohr shift of the oxyhemoglobin dissociation curve of the renal vein B) Blood flow per gram of tissue is greater in kidneys than in other organs OC) Erythropoietin synthesized in the kidneys increases local hematocrit and Po2 D) Fenestrations in glomerular capillaries promote the convective transport of O, from Bowman space to the efferent arteriole E) Most of the active transport to form urine in the kidney occurs in the renal podocytes

B. The kidneys receive approximately one-fourth to one-fifth of total cardiac output, and because of the small size of the kidneys, they receive among the largest blood flow per gram of tissue. Blood flow functions to provide oxygen and nutrients. In the kidneys, an additional function of blood flow beyond native supply of the organ parenchyma is to contribute to filtration of plasma to create ultrafiltrate and urine. Thus, the blood flow in the kidney is in excess of what is needed to meet its metabolic demands. This results in a small difference in arterial and venous Po, and Po, in the renal vein is relatively high compared to other organs that depend on blood flow to meet local metabolic demand. Incorrect Answers: A, C, D, and E. Shifting of the oxyhemoglobin dissociation curve (Choice A) describes hemoglobin's sigmoidal affinity for oxygen depending on the partial pressure of oxygen. The oxyhemoglobin dissociation curve can shift and is affected by H+ and the partial pressure of CO, not bicarbonate. Other factors that can affect the oxyhemoglobin dissociation curve include exercise, altitude, and temperature. Erythropoietin is a hormone produced by kidneys that stimulates proliferation of erythrocytic progenitors in the bone marrow. It does not increase local hematocrit and Po, (Choice C). Fenestrations in glomerular capillaries do not promote the convective transport of O, from Bowman space to the efferent arteriole (Choice D). Fenestrations in the glomerulus function for the filtration of plasma solutes and water to create ultrafiltrate in the Bowman space. Most of the active transport to form urine in the kidney does not occur in the renal podocytes (Choice E). The renal podocytes are cells in the Bowman space, which is the first site of filtration. Filtration between the glomerulus and Bowman space is secondar

156 Exam Section 4: Item 6 of 50 National Board of Medical Examiners' Comprehensive Basic Science Self-Assessment 6. A 47-year-old man comes to the physician because of a 2-week history of a persistent, nonproductive cough. He has a 3-year history of hypertension treated with a thiazide diuretic. A second drug was added to his medication regimen 2 weeks ago. Vital signs are within normal limits. Pulmonary examination shows no abnormalities. Which of the following drugs was most likely added to this patient's medication regimen? A) Clonidine B) Enalapril OC) Hydrochlorothiazide D) Losartan E) Metoprolol F) Nifedipine

B. Thiazide diuretics, such as hydrochlorothiazide (HCTZ), and ACE inhibitors, such as enalapril, are first line therapies for the treatment of hypertension. ACE inhibitors target the renin-angiotensin-aldosterone system, which is one of the primary regulators of blood pressure. ACE is also responsible for the degradation of bradykinin (a potent vasodilator), and ACE inhibitors lead to increased bradykinin concentrations. Bradykinin is a protussive mediator, and accumulation may result in a persistent, nonproductive cough. Persistent coughing is a major reason for the discontinuation of ACE inhibitors. Incorrect Answers: A, C, D, E, and F. Clonidine (Choice A) is an a2-adrenergic agonist that results in decreased sympathetic outflow with subsequent relaxation of vascular tone. Adverse effects include orthostatic hypotension, xerostomia, sedation, and bradycardia. There is also a risk for severe rebound hypertension when the medication is stopped. Hydrochlorothiazide (Choice C) is a thiazide diuretic that inhibits NaCI absorption in the distal convoluted tubule, resulting in natriuresis and diuresis. Adverse effects include hyponatremia, hypercalcemia, and hyperuricemia. Losartan (Choice D) is an angiotensin receptor blocker that is commonly prescribed for patients who cannot tolerate an ACE inhibitor. ACE activity is not affected, and bradykinin concentrations show no abnormalities. Metoprolol (Choice E) is a B1-adrenergic antagonist that primarily affects the heart. Adverse effects include bradycardia, heart block, and/or impaired cardiac output. Nifedipine (Choice F) is a dihydropyridine calcium-channel blocking (CCB) agent. CCB agents are potent arteriolar vasodilators. Adverse effects of CCBS include peripheral edema, headache, lightheadedness, and facial flushing. Educational Objective: ACE inhibitors are commonly

164 Exam Section 4: Item 14 of 50 National Board of Medical Examiners' Comprehensive Basic Science Self-Assessment 14. A 65-year-old woman with cirrhosis comes to the physician because of progressive shortness of breath during the past 3 months. Her pulse is 94/min, respirations are 20/min, and blood pressure is 100/60 mm Hg. Physical examination shows ascites and marked edema of the lower extremities up to the hips. A chest x-ray shows bilateral pleural effusions. Echocardiography shows a left ventricular ejection fraction of 55%. Which of the following is the most likely cause of the increased intra-alveolar fluid in this patient? A) Decreased cardiac systolic function B) Decreased plasma colloid osmotic pressure C) Increased capillary hydrostatic pressure D) Increased vascular permeability E) Lymphatic obstruction

B. Third-spacing of fluids from the intravascular space to the tissue interstitium and serosal spaces (pleural, peritoneal) is a common complication of cirrhosis. Patients commonly present with peripheral edema, pulmonary edema, pleural effusions, and ascites. Within the splanchnic circulation, portal hypertension leads to increased hydrostatic pressure and movement of fluid into the peritoneum. For the rest of the circulation, such as the pulmonary capillary bed and the alveoli, decreased plasma colloid osmotic pressure caused by hypoalbuminemia results in fluid movement out of the vasculature. The liver synthesizes albumin, which is the most prevalent serum protein. Reduced liver synthesis, decreased dietary intake, and water retention all lower the albumin concentration in the serum. Other manifestations of cirrhosis include increased bilirubin, jaundice, and signs of portal hypertension such as esophageal varices, splenomegaly, caput medusae, and rectal varices. It typically occurs in patients with preceding conditions such as alcohol use disorder or chronic hepatitis. Incorrect Answers: A, C, D, and E. Decreased cardiac systolic function (Choice A) occurs in heart failure with reduced ejection fraction. Venous congestion caused by reduced forward flow of blood results in increased capillary hydrostatic pressure (Choice C) with consequent interstitial edema. The patient's ejection fraction on echocardiography shows a normal ejection fraction. Increased vascular permeability (Choice D) occurs in acute respiratory distress syndrome from the effect of cytokines on the pulmonary parenchyma caused by direct or indirect lung injury. It may be secondary to numerous causes, including sepsis, aspiration, pneumonia, trauma, and shock. Lymphatic obstruction (Choice E) causes lymphedema, which manifests as peripheral edema cau

179 Exam Section 4: Item 29 of 50 National Board of Medical Examiners® Comprehensive Basic Science Self-Assessment 29. A 30-year-old primigravid woman develops severe uterine bleeding in the hospital after vaginal delivery of a 3827-g (8-lb 7-oz) male newborn at term. Pregnancy was uncomplicated. Delivery lasted 7 hours and was assisted with the use of forceps; delivery of the placenta followed. External uterine massage does not result in decrease of the postpartum hemorrhage. A medication is administered by continuous intravenous infusion to improve uterine contraction. This patient's bleeding is most likely to be decreased via which of the following mechanisms of action? A) Activation of a-adrenoreceptors B) Activation of G coupled receptors C) Decreased prostaglandin E, production D) Inhibition of nitric oxide synthetase activity E) Inhibition of phospholipase A, activity

B. Uterine atony typically presents with postpartum hemorrhage and a boggy uterus on physical examination. Risk factors for uterine atony include macrosomia, multiple gestation, prolonged labor, uterine infection, and spinal anesthesia. Initial management involves intravenous fluid administration, uterine massage, and oxytocin infusion. Oxytocin causes uterine contraction and can also be used to induce or augment labor. The oxytocin receptor is a G protein-coupled receptor with a G, alpha subunit. These proteins are integral membrane proteins with a characteristic structure composed of seven transmembrane domains. Upon activation of the G,coupled receptor, the receptor acts as a guanine nucleotide exchange factor and replaces guanosine diphosphate with guanosine 5'- triphosphate on an associated G protein. The alpha subunit, in this case the G, alpha subunit, diffuses away to activate phospholipase C, which cleaves phosphatidylinositol 4,5-bisphosphate (PIP2) into inositol 1,4,5-triphosphate (IP3) and diacylglycerol (DAG). IP3 then induces calcium release from the endoplasmic reticulum, which activates further downstream signaling and smooth muscle contraction in the uterine myometrium. Incorrect Answers: A, C, D and E. Activation of a radrenoreceptors (Choice A) would lead to vasoconstriction, increased total peripheral resistance, and increased mean arterial pressure through stimulation of a sympathetic G, protein-linked adrenoreceptor located in the vascular smooth muscle. Oxytocin does not stimulate a-adrenoreceptors. Decreased prostaglandin E, production (Choice C) would worsen uterine atony, as prostaglandin E, acts similarly to oxytocin to induce uterine contraction. It also softens the cervix. Its production is decreased by cyclooxygenase (COX) inhibitors but not by oxytocin. Inhibition of nitric oxide syntheta

145 Exam Section 3: Item 45 of 50 National Board of Medical Examiners' Comprehensive Basic Science Self-Assessment 45. A 5-month-old boy is admitted to the hospital because of frequent episodes of pneumonia, otitis media, diarrhea, and failure to thrive since birth. Physical examination shows rachitic, rosary-like abnormalities of the rib cage and wasting. Laboratory findings show an absolute lymphocyte count of 400/mm3 (N=1500-3000), and markedly decreased T- and B-lymphocyte counts. The lymphocyte count does not increase in response to mitogens, and there are decreased concentrations of immunoglobulins that do not increase after immunizations. This patient most likely has an enzymatic defect at which of the following steps in the metabolic pathway shown? Adenine A AMP Adenosine C F IMP Inosine E D Нурохanthine A) B) C) D) E) F)

C. Adenosine deaminase (ADA) deficiency (Choice C) results from autosomal recessive mutations in the ADA gene and is one of several mutations that can lead to the development of severe combined immunodeficiency disease (SCID). SCID is characterized by recurrent viral, bacterial, and fungal infections starting at birth, failure to thrive, and severe lymphopenia with impaired cellular and humoral immunity. ADA deficiency results in the accumulation of a purine derivative called deoxyadenosine within the purine salvage pathway, which is converted to 5'-deoxyadenosine triphosphate and inhibits the action of ribonuclease reductase thereby preventing DNA synthesis in T and B lymphocytes. Additionally, deoxyadenosine is toxic to lymphocytes. The only potentially curative therapy is stem cell transplant. Incorrect Answers: A, B, D, E, and F. Adenine phosphoribosyltransferase (APRTase) salvages and converts adenine to AMP (Choice A). Deficiency results in accumulation of adenine, which is converted to 2,8-dihydroxyadenine (DHA). DHA is poorly soluble in urine and can precipitate to cause nephroliths. 5'-nucleotidase (Choice B) is an enzyme located in the plasma membrane that converts 5'-AMP (which generally cannot cross the plasma membrane) to adenosine, which is able to enter cells. Acquired deficiency is encountered in lead poisoning. It also converts IMP to inosine (Choice F). It is not implicated in the development of SCID. Purine nucleoside phosphorylase (Choice D) converts inosine to hypoxanthine, xanthosine to xanthine, and guanosine to guanine. Deficiency of this enzyme also causes an immunodeficiency syndrome that only affects T lymphocytes while leaving B lymphocytes relatively unaffected. Patients present with recurrent infections that tend to occur slightly later in life. Hypoxanthine-guanine phosphoribosyltransfera

165 Exam Section 4: Item 15 of 50 National Board of Medical Examiners' Comprehensive Basic Science Self-Assessment 15. A 16-year-old girl is brought to the physician because of excessive dieting, constant studying, and social withdrawal during the past 3 weeks. She is 168 cm (5 ft 6 in) tall and weighs 48 kg (106 Ib); BMI is 17 kg/m2. Her blood pressure is 95/66 mm Hg. Physical examination shows dry skin and fine hair growth. Laboratory studies show: Serum 138 mEq/L 2.2 mEq/L 90 mEq/L 36 mEq/L 25 mg/dL 1.2 mg/dL Na+ K+ CI- HCO;- Urea nitrogen Creatinine Arterial blood gas analysis on room air pH Pco2 Po2 7.48 48 mm Hg 97 mm Hg This patient has most likely been abusing which of the following substances? A) Alcohol B) Amphetamines O C) Diuretics D) Laxatives E) Thyroid hormone

C. Anorexia nervosa (binge eating/purging type) is characterized by an intense fear of gaining weight, decreased self-worth related to body weight, and consequent binge eating and purging behavior (eg, vomiting, laxative or diuretic misuse). Patients with anorexia nervosa often have a BMI less than 17 kg/m2 Physical examination can show dry, scaly skin and fine hair or hair loss caused by decreased protein stores. Insufficient nutrition and weight loss may lead to amenorrhea. Diuretics block the reabsorption of sodium, chloride, and potassium in the kidneys, which leads to increased water excretion and consequent hypovolemia, with possible hypotension and tachycardia in severe cases. Hypovolemia can also lead to increased aldosterone secretion, which results in an increased reabsorption of sodium and secretion of potassium and hydrogen ions, leading to hypokalemia and metabolic alkalosis (sodium concentrations may be normal because of balanced reabsorption and secretion). Patients may demonstrate respiratory compensation for the metabolic alkalosis in the form of retained carbon dioxide leading to hypercarbia with a normalization of the pH. Treatment of anorexia requires a combined medical and psychiatric approach and involves correcting fluid and electrolyte derangements along with behavioral and pharmacologic therapy. Incorrect Answers: A, B, D, and E. Alcohol abuse (Choice A) is frequently comorbid with anorexia nervosa and may lead to hyponatremia, but metabolic alkalosis, hypokalemia, and hypochloremia would be atypical. Dry skin and hair loss would also be uncommon. Amphetamines (Choice B) are central nervous system stimulants that cause euphoria and increase sympathetic tone, leading to tachycardia, hypertension, pupillary dilation, and restlessness. Though appetite suppression and weight loss are common, electr

159 Exam Section 4: Item 9 of 50 National Board of Medical Examiners' Comprehensive Basic Science Self-Assessment 9. A 10-year-old boy with cerebral palsy begins benzodiazepine (BZ) therapy with diazepam. The mechanism of action of this drug occurs in the central nervous system where it interacts with the y-aminobutyric acid (GABA) receptor-ion channel macromolecular complex. Which of the following illustrations shown best represents the manner in which diazepam interacts with the macromolecular complex and the nature of the ion flux that subsequently occurs in this patient? A C- GABA & BZ binding site GABA & BZ binding site C CI- GABA binding site BZ binding site INTRACELLULAR INTRACELLULAR INTRACELLULAR CI- D K+ GABA & BZ binding site GABA & BZ binding site F GABA binding site BZ binding site INTRACELLULAR INTRACELLULAR INTRACELLULAR K+ K+ A) B) C) D) E) F)

C. Benzodiazepines are positive allosteric modulators of the ionotropic GABA, receptor, permitting the increased influx of chloride ions. Synaptic GABA binds the active site of the GABA receptor. Synaptic benzodiazepine binds the allosteric benzodiazepine binding site, which increases the frequency that the ion channel opens when GABA is bound. Thus, benzodiazepines enhance GABA activity. Increased influx of chloride ions hyperpolarizes the neuron and decreases the likelihood of neuronal action potentials. Cerebral palsy is a disorder of non-progressive central motor dysfunction resulting from several potential prenatal causes (eg, prematurity, intrauterine growth restriction, intrauterine infection, hypoxia) that typically presents with upper motor neuron signs such as muscle spasticity. Treatment involves physical and occupational therapy and pharmacologic management of spasticity with benzodiazepines and baclofen, a GABA, agonist. These medications increase the activity of GABA receptors, leading to decreased muscle contraction and relief of spasticity. Incorrect Answers: A, B, D, E, and F. Benzodiazepine binding the benzodiazepine site by itself (without GABA binding to the GABA, active site) (Choices A, B, D, and E) is insufficient to activate the GABA, receptor. Benzodiazepine binds the allosteric benzodiazepine binding site, which increases the frequency with which the ion channel opens when GABA is bound. The GABA, receptor is an ionotropic chloride channel, not a potassium channel (Choices D, E, and F). The extracellular chloride concentration is higher than the intracellular concentration, leading to an electrochemical gradient favoring chloride influx, not efflux (Choice B). Educational Objective: Benzodiazepines are positive allosteric modulators of the ionotropic GABAA receptor and thereby enhance the effe

168 Exam Section 4: Item 18 of 50 National Board of Medical Examiners' Comprehensive Basic Science Self-Assessment 18. A 23-year-old woman comes to the physician because of a 3-week history of fatigue and muscle pain shortly after she begins exercising; her symptoms decrease as exercise proceeds. Physical examination shows no abnormalities. Her fasting serum glucose concentration is 90 mg/dL. Following exercise of the left forearm, her serum lactic acid concentration does not increase in blood obtained from the left antecubital vein; however, ammonia concentration does increase. A muscle biopsy specimen is positive for periodic acid-Schiff staining. This patient most likely has a deficiency of which of the following muscle enzymes? A) Acid maltase B) Branching enzyme C) Glycogen phosphorylase D) Hexokinase E) Pyruvate dehydrogenase

C. Glycogen storage disease type V (McArdle disease) presents with fatigue and muscle cramping during exertion, typically in a patient who is otherwise developmentally normal. It arises because of an inability to degrade glycogen, which is necessary to supply the active cell with substrate for making adenosine triphosphate. During initial exercise when free glucose is rapidly consumed, muscle switches to degradation of glycogen for metabolism. An inability to break it down, which requires glycogen phosphorylase to liberate glucose monomers, results in impaired muscle metabolism and resultant cramping, fatigue, and mild rhabdomyolysis. Generally, after the initial fatigue and cramping, patients can resume exertion since alternative forms of energy are utilized via gluconeogenesis and protein catabolism; this may explain this patient's increased local ammonia concentration. Myophosphorylase, the muscle isoform of glycogen phosphorylase, hydrolyzes 1,4-glycosidic bonds of terminal residues in glycogen branches, liberating glucose-1- phosphate. The diagnosis is suspected on history and confirmed by biopsy showing increased muscle glycogen and potentially absent myophosphorylase and genetic testing. An exertional forearm exercise test can also inform the diagnosis, as in this case. Exercising muscle, when deprived of glucose, will break down protein and generate ammonia; the absence of lactate indicates impaired glycolysis from the absence of glucose. Resting creatine kinase activity may be mildly increased. Incorrect Answers: A, B, D, and E. Acid maltase (Choice A), also called 1,4-glucosidase, is an enzyme deficient in Pompe disease, a severe lysosomal and glycogen storage disease that results in hepatosplenomegaly, failure to thrive, and cardiomyopathy. It generally presents before two years of age and is often fatal in

96 Exam Section 2: Item 46 of 50 National Board of Medical Examiners® Comprehensive Basic Science Self-Assessment 46. A 2-month-old girl is brought to the physician for a follow-up examination 2 days after a complete blood count was found to be abnormal. Her blood group is A, Rh-positive. She was born at term to a healthy 23-year-old woman whose blood group is O, Rh- positive. Birth weight was 3890 g (8 lb 9 oz). Pregnancy and delivery were uncomplicated. The patient has been active and feeding wellI. Her temperature is 37.6°C (99.7°F), pulse is 120/min, and respirations are 30/min. Physical examination shows no abnormalities. Laboratory studies done since birth are shown. Patient Age 1 Week 15.7 49 24 Hours 16.9 1 Month Hemoglobin (g/dL) (N=11-15) Hematocrit (%) (N=28-45) Leukocyte count (/mm3) (N=5000-19,500) Segmented neutrophils Eosinophils Basophils Lymphocytes Monocytes Platelet count (/mm3) (N=150,000-400,0

C. Congenital neutropenia refers to the presence of neutropenia at or around the time of birth and is the most likely diagnosis in this case. Neutropenia is defined by an absolute neutrophil count (ANC) of less than 1500/mm3, although most patients with congenital neutropenia have an ANC less than 1000/mm3, at which point the risk for bacterial infection increases. The underlying cause most commonly includes mutations that affect the production of myeloid cells. Bone marrow biopsy, if performed, may show decreased cellularity with cells that are arrested at the promyelocyte or myelocyte stage. Many patients will remain largely asymptomatic. In more severe cases with lower neutrophil counts, patients may develop stomatitis and recurrent mucocutaneous ulcerations, often with abdominal pain from gastrointestinal ulceration. Neutropenia is found in many conditions present at birth including Chédiak-Higashi syndrome, X-linked agammaglobulinemia, and von Gierke disease, and should be distinguished from these by mutational analysis and evaluation for concomitant physical findings. Incorrect Answers: A, B, D, and E. Alloimmune hemolytic disease of the newborn (Choice A) occurs in patients who are Rh-positive and are born to mothers who are Rh-negative but were previously sensitized either by a prior birth or blood transfusion to make anti-Rh antibodies. Both the patient and her mother are Rh-positive, making this diagnosis unlikely. Congenital cytomegalovirus infection (Choice B) causes hearing loss, seizures, a petechial rash, and intracranial calcifications in the neonate when acquired in utero. Hepatomegaly, splenomegaly, and lymphadenopathy are also commonly seen. The virus is transmitted via the placenta from the mother, who often contracts the virus after interacting closely with young children. DiGeorge syndrome (Choic

182 Exam Section 4: Item 32 of 50 National Board of Medical Examiners® Comprehensive Basic Science Self-Assessment 32. A 23-year-old woman comes to the physician for prenatal care. On examination, she appears euthyroid and the thyroid gland is not palpable. Uterine size is consistent with dates, and fetal pulse is normal. Serum thyroid tests show: Thyroxine (T) Thyroid-stimulating hormone 14 ug/dL 2 μυ/mL Changes in which of the following hormones during pregnancy is primarily responsible for the thyroid test results? OA) Androgens B) Corticosteroids C) Estrogens D) Progesterones E) Prolactins

C. Estrogen concentrations increase in pregnancy because of production by the placenta. This increased estrogen concentration increases the concentration of transport proteins, such as thyroid hormone binding globulin, which can also be seen with exogenous estrogen administration. This is because of a decreased metabolism of thyroid hormone binding globulin, which allows more thyroid hormone to be bound by protein in the plasma. In the bound state, the hormone is inactive; it is only the free, unbound portion of the hormone that is biologically active to exert its effects. In this case, the total thyroxine concentration is increased, which includes bound and unbound protein, while the TSH concentration stays within normal limits. This is because the free concentration of thyroxine is unchanged, which keeps TSH within its normal limits as a result of feedback mechanisms. Incorrect Answers: A, B, D, and E. Androgens (Choice A) increase in pregnancy because of increased production by the corpus luteum and placenta. However, they are similarly bound by transport proteins (eg, sex hormone binding globulin). Androgens do not cause changes in the concentration of these proteins. Corticosteroids (Choice B) are produced by the adrenal cortex and are increased during normal pregnancy because of release of corticotropin-releasing hormone by the placenta. Corticosteroids do not affect the plasma concentration of thyroid hormones, although they are also bound by transport proteins. Progesterones (Choice D) increase in pregnancy because of production by the corpus luteum and placenta. They act to maintain the pregnancy, with low concentrations of progesterone leading to miscarriage. Increases in progesterone would not be expected to cause a change in thyroid hormone concentration. Prolactins (Choice E) increase in pregnancy and are

160 Exam Section 4: Item 10 of 50 National Board of Medical Examiners' Comprehensive Basic Science Self-Assessment 10. A previously healthy 26-year-old woman comes to the physician because of a 1-day history of severe rectal pain with no bleeding. She takes no medications and does not smoke cigarettes or drink alcohol. She has been sexually active, and she and her partner use condoms inconsistently. A photograph of the rectal area is shown. Which of the following is the most likely diagnosis? A) Anal carcinoma B) Anal fissure C) External hemorrhoid D) Human papillomavirus infection E) Skin tag

C. Hemorrhoids result from dilated submucosal veins in the anal canal. They are commonly classified as either internal or external based on their relation to the pectinate line of the anal canal. Internal hemorrhoids are typically painless owing to their visceral innervation, while external hemorrhoids are painful as a result of somatic innervation. While hemorrhoids can result in bright red blood per rectum, bleeding is generally mild. A thrombosed external hemorrhoid presents with acute, severe anal pain with an associated tender and swollen bulging, blue-purple nodule. Prolapsed internal hemorrhoids appear as a red or purple mass protruding from the anus during defecation. If severe, hemorrhoids may remain prolapsed following a bowel movement requiring manual reduction. Hemorrhoids are also a potential manifestation of portosystemic shunting in cirrhosis caused by anastomosis of the superior rectal vein with the middle and inferior rectal veins. Initial treatment of external hemorrhoids includes topical analgesics and steroids. If refractory to conservative management, placement of elastic bands is used for the ligation of hemorrhoids. This results in diminished blood flow to the hemorrhoid, which subsequently becomes necrotic and sloughs off. If severe, a thrombosed hemorrhoid may require surgical incision and drainage for immediate pain relief. Incorrect Answers: A, B, D, and E. Anal carcinoma (Choice A) is a form of squamous cell carcinoma induced by infection with high risk subtypes of human papillomavirus 16, 18, 31, or 33. A common presenting symptom is rectal bleeding and it may be misdiagnosed by as a hemorrhoid. However, anal carcinoma usually is preceded by anorectal condyloma which is not apparent in this case. Anal fissure (Choice B) is a common cause of severe rectal pain with defecation and occurs in p

117 Exam Section 3: Item 17 of 50 National Board of Medical Examiners' Comprehensive Basic Science Self-Assessment 17. A 69-year-old woman is brought to the physician by her son because of a 2-month history of memory problems, difficulty walking, and urinary incontinence. He says, "She's always been very sharp, but recently she's had a lot of trouble functioning." She has forgotten on several occasions to turn off the stove when cooking, and she once forgot she had turned on the bathtub until she noticed water coming down the walls from the upstairs bathroom. She also is unsteady on her feet and has fallen down at home on several occasions. She reports that her mood is fine. Physical examination shows mild nystagmus. She has a broad-based gait and is unable to walk in a straight line without assistance. Neurologic examination shows no other focal findings. Mini-Mental State Examination score is 20/30. Which of the fo

C. Normal-pressure hydrocephalus (NPH) refers to a type of communicating (non-obstructive) hydrocephalus in which the ventricles are enlarged, leading to damage of the white matter tracts connecting the frontal lobes and basal ganglia. NPH typically presents in older patients with dementia (eg, frontal executive dysfunction), gait apraxia, and urinary incontinence. The gait apraxia may manifest as a wide-based or magnetic gait. The central vestibular tract, a white matter tract located in the periventricular area, may also be affected by ventriculomegaly, leading to this patient's postural instability and mild nystagmus. Signs of increased intracranial pressure (eg, papilledema, postural headaches) are typically absent. Many cases of NPH are idiopathic though some are secondary to a defect in cerebrospinal fluid drainage from the ventricles caused by scarring or obstruction of the ventricular lining from hemorrhage or meningitis. The diagnosis is primarily clinical, although ventriculomegaly on brain imaging is suggestive. Patients may respond to ventricular shunting. Incorrect Answers: A, B, D, and E. Cerebral neoplasms (Choice A) typically lead to an insidious onset of a focal neurologic deficit rather than the multiple higher-order deficits demonstrated in NPH. Patients with cerebral neoplasms may also demonstrate signs of increased intracranial pressure. Dementia, Alzheimer type (Choice B) is a relatively common neurocognitive disorder that presents with progressive cognitive decline that begins with short-term memory impairment, progresses to language abnormalities and apraxia, and culminates in behavioral and personality changes preventing the patient from performing basic activities of daily living. Early gait dysfunction and urinary incontinence would be atypical. Parkinson disease (Choice D) is a neurologic sy

79 Exam Section 2: Item 29 of 50 National Board of Medical Examiners® Comprehensive Basic Science Self-Assessment 29. A 36-year-old man with a 4-year history of HIV infection comes to the physician for a follow-up examination. Current medications include lopinavir-ritonavir and lamivudine (3TC)-zidovudine (AZT). His CD4+ T-lymphocyte count is 410/mm3 (N2500); 6 months ago, it was 720/mm3 Reverse transcriptase polymerase chain reaction test results show that his remaining CD4+ T lymphocytes are positive for CCR5. Based on the expression of CCR5, which of the following cell types in this patient is most likely also infected with HIV? A) B lymphocytes B) Eosinophils C) Macrophages D) Mast cells E) Natural killer cells

C. Macrophages are the other cell type in addition to CD4+ T lymphocytes that are most likely also infected with HIV as they possess CD4 and CCR5 on their cell surface. To enter the cell, HIV must first bind to CD4 via the HIV gp120 and gp41 subunits. This binding induces a conformational change in the HIV envelope allowing for binding to a coreceptor such as CCR5 or CXCR4. CCR5 is expressed on the surfaces of antigen presenting cells such as macrophages and dendritic cells, in addition to T lymphocytes. Following this, the HIV envelope fuses with the cellular membrane and expels its contents into the host cell. The HIV RNA genome is subsequently reverse transcribed into DNA and inserted into the host cell genome via HIV integrase, followed by transcription of HIV genes and translation into HIV proteins. These proteins are packaged and cleaved to form the mature HIV virus. Following infection with HIV, macrophages may also may serve to perpetuate infection for several reasons: they are present in high numbers at sites of viral entry, are resistant to the cytotoxic effects of HIV infection allowing for HIV persistence and replication, and have a remarkably long life span with the ability to survive for months to years in peripheral tissues. They also distribute widely throughout the body, including the central nervous system. Incorrect Answers: A, B, D, and E. B lymphocytes (Choice A), eosinophils (Choice B), mast cells (Choice D), and natural killer cells (Choice E) do not express both CD4 and CCR5 on their surface, thus HIV is unable to enter and infect these cell types. Educational Objective: HIV binds to CD4 and a coreceptor such as CCR5 to enter CD4+ T lymphocytes. Macrophages also possess CD4 and CCR5, which allows for HIV viral entry. Infection can persist given the long half-life of macrophages, their resistance

189 Exam Section 4: Item 39 of 50 National Board of Medical Examiners' Comprehensive Basic Science Self-Assessment 39. A 60-year-old man comes to the physician for a routine health maintenance examination. Physical examination shows no abnormalities. Urinalysis shows: pH Specific gravity Blood 6.0 1.018 3+ Glucose Protein negative 1+ Microscopic examination of the urine shows atypical cells. A CT scan of the abdomen discloses a lesion of the right kidney as shown in the photograph. Which of the following compounds is the most significant predisposing risk factor for this patient's condition? A) Arsenic B) Beryllium C) 2-Naphthylamine D) Nickel E) Vinyl chloride

C. Papillary transitional cell carcinoma is a common tumor of the urinary tract and typically presents with painless gross hematuria. It can develop in the renal collecting tubules, calyx, renal pelvis, ureter, urethra, or bladder, as all of these structures share the same embryologic origin and are comprised of the same type of epithelium. Hematuria is a common finding and is present in most patients, with ureteral obstruction occurring in about twenty percent of patients as well. Some affected individuals remain asymptomatic until an advanced stage of disease and therefore present with a worse prognosis. Ureteral obstruction occurs in many patients at the site of the primary tumor or in noncontiguous areas with drop metastases. Cigarette smoking is a strong risk factor for transitional cell carcinoma of the urinary tract. Carcinogens known to be associated with urothelial cancer include phenacetin, cyclophosphamide, and 2-naphthylamine. Carcinogens are excreted in the urine thereby causing exposure to the mucosal surfaces of the upper and lower urinary tract. Treatment is based upon the stage of cancer but typically involves partial or complete removal of the affected kidney, ureter, or part of the bladder. Incorrect Answers: A, B, D, and E. Arsenic (Choice A) has also been associated with transitional cell carcinoma. However, cigarette smoking, aniline dyes, 2-naphthylamine, and other carcinogenic compounds have been shown to be the major risk factors for transitional cell carcinoma. Beryllium (Choice B) exposure can lead to berylliosis, characterized by granulomatous inflammation of the lungs. Berylliosis and other pneumoconioses can increase risk for bronchogenic and pulmonary malignancy. Nickel (Choice D) exposure is a common cause of contact dermatitis, which is a T-lymphocyte mediated type IV hypersensitivity r

41 Exam Section 1: Item 41 of 50 National Board of Medical Examiners® Comprehensive Basic Science Self-Assessment 41. A 14-year-old boy is brought to the physician by his parents because of increasingly severe left knee pain during the past week. The pain is exacerbated by running or kneeling. He plays multiple sports and is currently in the middle of soccer season. There has been no recent trauma to the area or related sports injuries. Physical examination shows full range of motion of both lower extremities. There is a discrete area of swelling just below the left patella over the proximal portion of the tibia. Palpation of the area produces pain. Pain is also reproduced when he does a full squat. Examination of the right knee shows no abnormalities. Which of the following is the most likely cause of this patient's condition? A) Chondromalacia B) Ligamental tear C) Osgood-Schlatter disease D) Osteochondritis E) St

C. Osgood-Schlatter disease refers to osteochondrosis or traction apophysitis of the tibial tubercle that typically occurs in adolescent, athletic children. Repetitive tension via the patellar tendon transmits to the tibial tubercle, presenting as pain reported during activities that increase the stress on the tubercle, such as kneeling, squatting, kicking, or similar activities that increase the extensor force transmitted by the quadriceps. Patients localize pain to the anterior aspect of the proximal tibia and knee. Physical examination typically discloses tenderness over an enlarged tibial tubercle. The condition is diagnosed clinically; x-rays, if obtained, may show increased lucency in the area of the tibial tubercle. Treatment is through rest, cryotherapy, and non-steroidal anti-inflammatory medications, as the condition resolves with time and unloading. Incorrect Answers: A, B, D, and E. Chondromalacia (Choice A) as related to the knee refers to deterioration of cartilage along the posterior aspect of the patella. It is common in young athletes, especially runners, and presents with knee pain that is worse with bending. It can be differentiated from Osgood-Schlatter disease by the location of pain, whereas chondromalacia will not present with pain at the tibial tuberosity. Ligamental tear (Choice B) is a broad term that in the context of the knee could describe damage to any of the collateral ligaments, such as the anterior or posterior cruciate or the medial or lateral collateral ligaments. Tear or rupture of these ligaments presents with pain which is worse when placing the affected ligament under strain, tenderness in the area of the injury, and laxity when evoking the motion restricted by the ligament itself. Osteochondritis (Choice D) describes the inflammation of bone or cartilage within a joint; it may al

141 Exam Section 3: Item 41 of 50 National Board of Medical Examiners' Comprehensive Basic Science Self-Assessment 41. A 23-year-old woman who was recently diagnosed with rheumatoid arthritis comes to the physician for a follow-up examination. Two months ago, she began treatment with adalimumab, which binds with tumor necrosis factor-a. The patient says that her symptoms have not improved. Physical examination shows mild swelling of the joints of the wrists, hands, ankles, and feet. Which of the following immune system components is most likely contributing to this patient's symptoms? O A) IgA B) IgE C) Interleukin-1 (IL-1) D) IL-4 E) Vascular endothelial growth factor

C. Rheumatoid arthritis (RA) is an autoimmune disease that causes extensive inflammation in synovial tissues. The clinical course is characterized by joint destruction, periarticular bone loss, synovial hypertrophy, as well as extraarticular manifestations such as rheumatoid nodules and skin rash. Patients often present with symmetric patterns of arthritis particularly in the wrists and hands. Serologic studies generally show a positive rheumatoid factor and anti-cyclic citrullinated peptide antibodies. RA is associated with the HLA-DR4 genotype. During an immune or inflammatory response, many cytokines and chemokines are expressed. In RA, both the classic and alternative pathways of complement are activated, along with a broad cellular and humoral immune response in the synovial space. Autoantibodies and immune complexes contribute to complement activation. Infiltration of immune cells into the synovial space including macrophages, neutrophils, and T lymphocytes results in the production of cytokines and acute phase reactants within the joint space such as interleukins and tumor necrosis factor (TNF). TNF is key in the pathophysiology of RA. It is a potent activator of macrophages and neutrophils, enhancing their cytotoxic effects and their expression of endothelial adhesion molecules to promote migration into peripheral tissues. Because of this central role in RA, it is a key target of disease modifying agents such as adalimumab. Similarly, interleukin-1 (IL-1), also known as osteoclast activating factor, is a potent cytokine; it promotes bone reabsorption and cartilage destruction and is key in the pathophysiology of RA. IL-1 leads to an increase in RANK ligand signaling and subsequent osteoclast-mediated bone resorption. This patient is already taking an appropriate anti-TNF agent but has persistent and progressive

194 Exam Section 4: Item 44 of 50 National Board of Medical Examiners® Comprehensive Basic Science Self-Assessment 44. A 35-year-old man comes to a new physician because of a 3-day history of abdominal pain and daily loose stools. He has a 2-year history of recurrent epigastric pain and a 4-month history of peptic ulcer disease treated with acid suppression medications. He takes no other medications. He was told on three separate occasions that he had ulcers in his duodenum and jejunum but no infection. His blood pressure is 120/78 mm Hg. The abdomen is soft. There is tenderness to palpation of the epigastric area but no rebound tenderness. Which of the following is the most likely cause of this patient's condition? A) Biliary reflux B) Eosinophilic infiltration into the gastric wall C) Increased gastrin production D) Urease production E) Villous atrophy

C. This patient's findings of abdominal pain and recurrent, unexplained gastric and duodenal ulcers are suggestive of Zollinger-Ellison syndrome (ZES). ZES stems from a pancreatic or duodenal gastrin-secreting tumor. Gastrin is typically produced by gastric G cells and stimulates parietal cells to release protons in order to form hydrochloric acid. Excessive production of acid in the setting of ZES leads to recurrent, chronic duodenal or jejunal ulcers, which can present with abdominal pain, diarrhea, malabsorption, and possible hematemesis, melena, or hematochezia. Pancreatic gastrinoma and Zollinger-Ellison syndrome may present as a component of multiple endocrine neoplasia type 1. Incorrect Answers: A, B, D, and E. Biliary reflux (Choice A) occurs when bile secreted into the duodenum is refluxed proximally into the stomach or esophagus. Symptoms include nausea, bilious vomiting, and abdominal pain. Biliary reflux commonly occurs following bariatric surgery. Eosinophilic infiltration into the gastric wall (Choice B) is characteristic of eosinophilic gastroenteritis, a rare cause of chronic diarrhea and malabsorption. Urease production (Choice D) is associated with Helicobacter pylori, a gram-negative rod that infects the stomach and duodenum. It is the most common cause of peptic ulcer disease. Villous atrophy (Choice E) is commonly found in association with Celiac disease, also called gluten-sensitive enteropathy. Celiac disease presents with chronic diarrhea and malabsorption. It does not cause recurrent peptic ulcers. Educational Objective: Zollinger-Ellison syndrome stems from a gastrin-secreting tumor located typically within the pancreas or duodenum that results in excessive production of acid by the gastric parietal cells. Patients develop recurrent, chronic duodenal or jejunal ulcers, which can present with a

42 Exam Section 1: Item 42 of 50 National Board of Medical Examiners® Comprehensive Basic Science Self-Assessment 42. A 40-year-old man is evaluated because his skin is extremely sensitive to sunlight. Exposure to the sun causes the formation of vesicles and blisters on the skin, which frequently take weeks to heal. He is diagnosed with a disorder caused by the increased synthesis of compounds in the skin that are subject to excitation by visible light. Which of the following biochemical pathways is most likely defective in this patient? A) Bile acid synthesis B) Bilirubin degradation C) Heme synthesis D) Melanin synthesis E) Riboflavin metabolism

C. Porphyria cutanea tarda is characterized by severe cutaneous photosensitivity with blistering and hyperpigmentation after exposure to sunlight and is the most common of the porphyrias. It is caused by decreased activity of uroporphyrinogen decarboxylase, an enzyme used to in the production of heme. The initial substrates for heme are glycine and succinyl-CoA. Their conversion to heme begins in the mitochondria with a rate limiting step catalyzed by aminolevulinate synthase. A series of additional steps then occurs which take place in the cellular cytoplasm. In one of these intermediate steps, uroporphyrinogen decarboxylase catalyzes the conversion of uroporphyrinogen III to coproporphyrinogen III. Not only will a deficiency in uroporphyrinogen decarboxylase prevent correct heme synthesis, it will also cause uroporphyrinogen III to accumulate. Accumulated uroporphyrinogen IIl is then deposited in the skin. Upon exposure to light of wavelength 400nm, the molecule enters an excited state and releases photons which in turn create reactive oxygen species within the skin. These reactive oxygen species damage the basement membrane, lipids, and proteins nearby resulting in dermoepidermal separation and blister formation. Incorrect Answers: A, B, D, and E. Bile acid synthesis (Choice A) uses cholesterol as the initial substrate. Cholesterol 7a-hydroxylase is the rate-limiting step in the creation of bile acids. Porphyrins are not intermediate byproducts of this pathway. Impaired bile acid production will lead to decreased intestinal absorption of lipids and fat-soluble vitamins, but not photosensitivity and blister formation. Bilirubin forms as a result of the degradation of heme. Bilirubin degradation (Choice B) is the process by which bilirubin is first conjugated in the liver to become water soluble and then excreted into

121 Exam Section 3: Item 21 of 50 National Board of Medical Examiners' Comprehensive Basic Science Self-Assessment Right Long 21. A 35-year-old man comes to the physician 1 day after he noticed a painless mass on the right side of his scrotum. Examination of the scrotum shows a 2-cm, firm, nontender mass on the right that cannot be separated from the right testis. Ultrasonography of the scrotum is shown. Metastases from this site are most likely to appear in which of the following lymph node groups? A) Common iliac B) Internal iliac C) Para-aortic (lumbar) D) Periumbilical E) Superficial inguinal

C. Testicular cancer most often occurs in young men and presents with a painless, firm nodule in the testis, which is commonly asymptomatic, although it can also present with a dull ache in the surrounding region. The testes are drained by the para-aortic (lumbar) lymph nodes, as are the ovaries in women. These lymph nodes remain at the site of gonadal origination in the embryo, whereas the testes descend into the scrotum later in embryonic development. Therefore, initial nodal metastases will spread to the para-aortic lymph nodes. If even further advanced, it can present with signs of distant metastasis, such as cough (lung metastases) or bone pain (bone metastases). Results of laboratory studies will often indicate increased concentrations of a-fetoprotein or B-hCG, the latter of which can cause gynecomastia depending on the subtype of testicular cancer. Ultrasonography is the initial step in evaluation of a palpable testicular mass and will show a heterogeneous mass with solid or cystic components, including calcifications, in the setting of malignancy. Treatment requires orchiectomy with possible chemotherapy depending on the presenting stage of disease. Incorrect Answers: A, B, D, and E. Common iliac (Choice A) lymph nodes drain lymph from the internal and external iliac lymph nodes. The internal iliac (Choice B) lymph nodes drain the prostate, cervix, corpus cavernosum of the penis, and the proximal vagina, whereas the external iliac lymph nodes drain the body of the uterus and the superior bladder. Neither drain the testes. Periumbilical (Choice D) or mesenteric lymph nodes drain the gastrointestinal system, including the jejunum, ileum, cecum, and parts of the colon. They play no role in the drainage of the testes. Superficial inguinal (Choice E) lymph nodes drain the distal vagina, vulva, scrotum, and distal a

155 Exam Section 4: Item 5 of 50 National Board of Medical Examiners® Comprehensive Basic Science Self-Assessment 5. A 25-year-old man comes to the physician 10 days after injuring his right elbow while trying to lift an 80-lb dumbbell at the gym. When he attempted to lift the weight from the rack, he felt a popping sensation and pain in his right elbow. He uses anabolic steroids. His pulse is 78/min, and blood pressure is 150/95 mm Hg. Examination of the right upper extremity shows swelling of the antecubital fossa and ecchymoses from the antecubital fossa to the mid forearm on the anterior side. There is tenderness to palpation over the proximal aspect of the anterior forearm. He is able to flex the elbow actively, but he is unable to flex the elbow or supinate the forearm against resistance. This patient has most likely injured a muscle-tendon unit that is supplied by which of the following nerves? A) Anterior in

C. The biceps muscle, located in the anterior arm, arises from a long and short head originating from the intra-articular supraglenoid tubercle (ie, long head) and coracoid process (ie, short head). The biceps tendon inserts at the radial tuberosity. Injury to the biceps tendon or biceps muscle, such as in this patient, results in weakness during or complete loss of function of elbow flexion and forearm supination. Tenderness and swelling may occur in the area of injury or rupture, and an abnormally contracted mass of retracted muscle may be palpated in the anterior compartment of the arm. The biceps muscle is innervated by the musculocutaneous nerve, which arises from spinal nerve roots C5 to C7. The musculocutaneous nerve also innervates the brachialis muscle, a flexor of the elbow, and supplies sensation to the anteroinferior arm and lateral forearm via its lateral cutaneous branch. Incorrect Answers: A, B, D, and E. The anterior interosseous (Choice A) nerve is a branch of the median nerve (Choice B). It is a purely motor nerve and supplies the flexor pollicis longus and flexor digitorum profundus muscles of the forearm, along with the pronator quadratus. It does not supply any muscles of the arm. The median nerve, which gives rise to it, is a mixed motor and sensory nerve that supplies the muscles of the anterior forearm, the thenar eminence, and the lateral two lumbrical muscles. It also supplies sensation to the anterior forearm, and anterolateral hand. The radial (Choice D) nerve supplies motor fibers to the posterior arm and forearm, along with sensory fibers to the posterior forearm and hand. The ulnar (Choice E) nerve supplies motor fibers to the flexor carpi ulnaris and the flexor digitorum profundus muscles of the forearm, and most of the intrinsic muscles of the hand. It provides sensory innervation to th

124 Exam Section 3: Item 24 of 50 National Board of Medical Examiners' Comprehensive Basic Science Self-Assessment 24. A prospective study is done to assess the relative efficacy of two different antihypertensive medications, both of which were shown to be efficacious in placebo-controlled trials. In the study, patients with hypertension were randomly assigned to receive one of two antihypertensive medications. At the conclusion of the study, some participants reported inconsistent adherence to their medication regimen, while others reported discontinuing their study medication altogether before the primary outcome (blood pressure measurement after 30 days) was obtained. A few of these patients who discontinued their study medication were prescribed the other group's study medication by their primary care physician and were taking it at the time of the primary outcome measurement. In their primary analysis, which of

C. The investigators should analyze the data of the patients who were not adherent to their medication regimens with the group to which he or she was assigned. This is known as an intention-to-treat analysis, which is used to preserve randomization when attrition or crossover is introduced into a study. According to intention-to-treat analysis, all patients should be analyzed as part of the group to which they were initially randomized. Although randomization is preserved, intention-to-treat analysis leads to problems as it ignores protocol deviations, withdrawal from the study, and noncompliance. In this study, the intention-to-treat analysis ignores the fact that a proportion of each of the placebo and control groups did not adhere to the protocol as prescribed. Because of this, the true effectiveness of the medications compared against each other may be skewed. In this example, intention-to-treat analysis would dictate that the patients who inconsistently adhered to their medication regimen should be analyzed with the group to which they were initially randomized. Incorrect Answers: A, B, and D. Exclude the outcome for only those who inadvertently switched to the other study medication (Choice A) and exclude the outcome for only those who reported stopping the medication altogether (Choice B) ignore intention-to-treat analysis. Such analysis would potentially introduce significant differences between the populations under study, as there may be an underlying difference between patients choosing to switch or stop their medications as compared with the groups as a whole. Include the outcome for those who took more than a prespecified fraction of medication doses (Choice D) would also violate intention-to-treat analysis. Specifying a qualifying fraction of medication taken may skew the participants in the analyzed samp

148 Exam Section 3: Item 48 of 50 National Board of Medical Examiners' Comprehensive Basic Science Self-Assessment 48. A 20-year-old man comes to the emergency department 30 minutes after fainting while playing basketball. He has felt tired for the past 3 months. A systolic murmur is heard at the left sternal border. The murmur increases when he strains against a closed glottis and decreases when he lies supine with his legs elevated. Which of the following is the most likely diagnosis? A) Aortic regurgitation B) Aortic stenosis C) Idiopathic hypertrophic obstructive cardiomyopathy D) Mitral regurgitation E) Mitral stenosis F) Pulmonic regurgitation G) Tricuspid stenosis

C. The patient's presentation is consistent with idiopathic hypertrophic obstructive cardiomyopathy, which classically presents with dyspnea, chest pain, syncope, or sudden cardiac death in an exercising young athlete with a potential family history of a similar event. Physical examination findings include a laterally displaced point of maximal impulse along with a systolic murmur that is loudest in the lower left sternal border; the murmur is quieter with increased preload and afterload and louder with decreased venous return. It is secondary to hypertrophy of the interventricular septum, which can be seen on echocardiography. In addition to left ventricular outflow tract obstruction, the walls of the left ventricle are stiff and noncompliant, which impairs filling of the chamber during diastole. Cardiac output decreases, which becomes more pronounced with tachycardia because of decreased diastolic filling time. Medications that increase diastolic filling by reducing the pulse, such as beta-blockers or non-dihydropyridine calcium channel blockers, are the mainstay of treatment. Incorrect Answers: A, B, D, E, F, and G. Aortic regurgitation (Choice A) presents with an early diastolic decrescendo murmur best heard in the right second intercostal space. It is commonly associated with endocarditis, acute rheumatic fever, and aortic root dilation. Aortic stenosis (Choice B) presents with a crescendo-decrescendo systolic murmur best heard at the upper right sternal border that radiates to the carotid arteries. It classically occurs secondary to age-related fibrotic and calcific changes of the valve but can occur earlier in life in cases of bicuspid aortic valve or chronic rheumatic disease. Mitral regurgitation (Choice D) presents with a holosystolic murmur best heard in the left fourth or fifth intercostal space along the m

186 Exam Section 4: Item 36 of 50 National Board of Medical Examiners' Comprehensive Basic Science Self-Assessment 36. A 53-year-old man comes to the physician to learn the results of a series of tests to evaluate his severe abdominal pain. The testing occurred over a 3-week period and the recent results indicate pancreatic cancer. When the physician reviews the results with the patient and discusses the diagnosis, the patient becomes angry and asks, "Why did it take you so long to figure this out?" Which of the following is the most appropriate response by the physician? A) "Actually, it frequently takes much longer to arrive at a diagnosis of pancreatic cancer." B) "I think all of the tests were done within a reasonable time frame." C) "It must be very difficult to hear this news." D) "Since there is no good treatment available, it really doesn't matter." E) "The most important thing now is to find you a good oncol

C. The physician should validate this patient's emotions. When patients express anger or other strong emotions related to their medical situation, the most effective initial steps are to listen and validate the emotions. Physicians should encourage patients to express their specific concerns while the physician actively listens. There are several methods to validate emotions: being present (actively listening and indicating understanding via nonverbal cues such as nodding), accurate reflection (restating a patient's words, as this physician did), mindreading (inferring the emotional content behind the patient's words), and normalizing (recognizing the patient's reaction as a response any other person would have). Incorrect Answers: A, B, D, and E. Explaining that the diagnostic time frame was reasonable (Choices A and B) or doesn't matter (Choice D) serves to absolve the physician rather than address the patient's emotions. Listening and validating emotions would more likely relieve the patient's distress and secondarily defuse the patient's anger toward the physician. When patients receive a new terminal diagnosis, physicians should approach prognosis-related discussions cautiously, compassionately, and non-defensively. Explaining the next steps in small pieces of information (Choice E) would be appropriate later in the conversation. However, the physician should first validate this patient's emotional response to the bad news. Educational Objective: When patients express anger or other strong emotions related to their medical situation, the most effective initial steps are to listen and validate the emotions. Physicians should avoid defending their own position when patients are angry and instead focus on the patient's emotions. %3D Previous Next Score Report Lab Values Calculator Help Pause

191 Exam Section 4: Item 41 of 50 National Board of Medical Examiners' Comprehensive Basic Science Self-Assessment 41. A 37-year-old woman is found to have a 1-cm thyroid nodule on routine physical examination. There is no adenopathy. Fine-needle aspiration of the nodule confirms papillary thyroid carcinoma. She undergoes thyroidectomy and postoperative radioiodine (131) ablation. When the radioiodine is administered, there is a large concentration of homogenous iodine uptake in the superior mediastinum. Which of the following best explains this finding? A) Extravasation of 131 from the brachiocephalic vein B) Outline of the aortic arch C) Retrosternal thyroid tissue D) Uptake of the 131 by normal lymphoid tissue E) Uptake of the 131 by the thymus

C. The thyroid gland develops as a derivative of the second pharyngeal arch as a diverticulum at the base of the tongue. As the gland develops, it descends through the thyroglossal duct, which traverses through the base of the neck and near the superior mediastinum, before arriving at its final position in the neck. Ectopic thyroid tissue may rarely be found at any location associated with migration of the thyroid gland. While rare, retrosternal ectopic thyroid tissue can be found. Most patients are asymptomatic and euthyroid, although large masses can lead to compression of adjacent structures. Incorrect Answers: A, B, D, and E. Extravasation of 131| from the brachiocephalic vein (Choice A), which is formed by the union of the jugular and subclavian veins, would be accompanied by significant hemorrhage and hemodynamic compromise. The outline of the aortic arch (Choice B) is located in the superior mediastinum but would not be expected to take up radioactive iodine. Uptake of the 1311 by normal lymphoid tissue (Choice D) or by the thymus (Choice E) is not typical. Radioactive iodine is primarily taken up by the thyroid gland, and both lymphoid and thymic tissues are usually not visible on radioactive iodine scintigraphy. Educational Objective: Ectopic thyroid tissue can be rarely found in any location associated with migration of the developing thyroid gland from the base of the tongue through the neck to the upper mediastinum. Most patients are asymptomatic and the ectopic tissue is often discovered incidentally during radioactive iodine scintigraphy. %3D Previous Next Score Report Lab Values Calculator Help Pause

166 Exam Section 4: Item 16 of 50 National Board of Medical Examiners' Comprehensive Basic Science Self-Assessment 16. Virus isolates are obtained from the blood of a man infected with HIV before and after 1 year of treatment with a drug regimen that includes zidovudine (AZT). Sensitivity testing for zidovudine shows: 50% Inhibitory Concentration Before zidovudine After zidovudine .025 µM 1.16 μΜ The viral gene most likely to have mutated during the year to cause this change is the one encoding which of the following proteins? A) Envelope glycoproteins B) Integrase C) Polymerase D) Protease E) Transactivator

C. This patient's 50% inhibitory concentration, or the concentration of drug needed to inhibit 50% of the viral growth, has significantly increased over one year. This indicates that the virus has become less sensitive to zidovudine (AZT). RNA-dependent DNA polymerase (HIV reverse transcriptase) mutations are most likely to account for this patient's decreased sensitivity to AZT. Drugs that block HIV reverse transcriptase are either nucleoside reverse transcriptase inhibitors (NRTIS) or non-nucleoside reverse transcriptase inhibitors (NNRTI). AZT is an NRTI while drugs such as efavirenz are NNRTIS. The process of HIV reverse transcription is error prone and can result in mutations of the HIV DNA caused by inaccurate transcription, but selective pressure garnered by use of NRTIS can lead to mutations of the HIV reverse transcriptase conferring resistance. AZT is a thymidine analog and the mutations that result in resistance to AZT and similar drugs are referred to as thymidine analog mutations (TAMS). Depending upon the specific mutations in HIV reverse transcriptase, the mutation can confer resistance to all drugs within the NRTI class, but usually at least three TAMS are necessary for HIV resistance to emerge. Continued use of the same drug will lead to accumulation of additional mutations caused by selective pressure. Resistance is most often prevented by using a combination of medications from different classes, but patients who were treated in the early days of the HIV pandemic before the wide availability of combination regimens are more likely to have mutations that developed as a result of monotherapy with drugs such as AZT. Incorrect Answers: A, B, D, and E. Attachment of the virus to the CD4+ T lymphocyte surface involves envelope glycoproteins (Choice A) (eg, gp120), CD4, CXCR4, and CCR5 surface proteins. Mut

126 Exam Section 3: Item 26 of 50 National Board of Medical Examiners' Comprehensive Basic Science Self-Assessment 26. A 27-year-old man comes to the physician because he is concerned about his skin. He says that he has had similar problems with hypopigmented skin for 2 years but that the condition has been exacerbated in recent months. A photograph of the affected skin is shown. The remainder of the physical examination shows no other abnormalities. Which of the following is the most likely causal organism? A) Candida albicans B) Epidermophyton floccosum C) Malassezia furfur D) Microsporum canis E) Trichophyton mentagrophytes

C. Tinea versicolor is a superficial infection of the epidermis caused by Malassezia furfur. Clinically, it is characterized by hypopigmented and hyperpigmented macules and patches with a fine scale. The infection has a predilection for sites with increased sebum production such as the chest and back. Patients commonly notice the rash after spending time outdoors, as Malassezia furfur inhibits tanning of the affected areas, making them more noticeable when compared to the unaffected neighboring skin. A potassium hydroxide preparation of a skin scraping can be diagnostic and will show fungal spores and short hyphae along the edges of epidermal cells. Tinea versicolor may be treated using topical antifungal preparations including topical ketoconazole or selenium sulfide preparations. Incorrect Answers: A, B, D, and E. Candida albicans (Choice A) is a fungus that causes a variety of infections in both immunocompetent patients and those with HIVIAIDS or immunocompromise. Candida often causes skin infections in moist intertriginous areas such as the groin and inframammary folds but can also cause invasive infections with high morbidity and mortality, including blood stream and urinary tract infections. Candida is not the cause of tinea versicolor. Epidermophyton floccosum (Choice B) and Trichophyton mentagrophytes (Choice E) are dermatophytes (skin fungus) that can cause superficial infection on any part of the skin, but most commonly affects the trunk or extremities (tinea corporis), groin (tinea cruris), foot (tinea pedis), scalp (tinea capitis), and nail (tinea unguium). Tinea versicolor, in contrast, is caused by the yeast Malassezia furfur. Microsporum canis (Choice D) is a zoophilic dermatophyte in that it typically colonizes the skin of animals. It also is one of the most common fungi that causes tinea capitis in hum

118 Exam Section 3: Item 18 of 50 National Board of Medical Examiners' Comprehensive Basic Science Self-Assessment 18. A 30-year-old woman comes to the physician for a follow-up examination because of a 10-year history of recurrent renal calculi. The calculi have primarily been composed of calcium oxalate. Physical examination shows no abnormalities. Serum calcium and phosphorus concentrations are within the reference ranges. A 24-hour urine collection shows an increased calcium concentration. The physician recommends that the patient increase her consumption of water and decrease her consumption of sodium and protein. The physician also recommends pharmacotherapy with a drug that decreases urinary calcium concentrations. Which of the following is the most likely mechanism of action of this drug? A) Decreases 1,25-dihydroxycholecalciferol synthesis B) Decreases H+ secretion in the distal tubule C) Decreases magnesium

D. Calcium oxalate stones are the most common cause of nephrolithiasis. Patients typically have normal serum calcium concentrations but excrete excessive quantities of calcium in the urine. Calcium oxalate crystals have an octahedral morphology under light microscopy and form radiopaque calculi. Patients can be treated with thiazide diuretics, a low-sodium diet, and supplementation with potassium citrate. Thiazide diuretics, such as hydrochlorothiazide or chlorthalidone, exert diuretic effects by reducing sodium and chloride reabsorption in the DCT through inhibition of the sodium-chloride cotransporter (NCC) channel. Thiazides have the opposite effect on calcium and result in increased reabsorption of calcium in the distal tubule. Patients with calcium oxalate stones can also be treated with oral citric acid, which reduces the formation of calcium oxalate crystals by forming soluble complexes with calcium ions. Incorrect Answers: A, B, C, E, F, and G. Decreased 1,25-dihydroxycholecalciferol synthesis (Choice A) would result in decreased calcium absorption in the intestine and decreased serum calcium concentrations. Most patients with calcium oxalate stones are normocalcemic. Decreased H*secretion in the distal tubule (Choice B) and increased HCO,-secretion in the proximal tubule (Choice E) result from treatment with carbonic anhydrase inhibitors, which leads to alkalinization of the urine. Calcium oxalate stones precipitate under conditions of hypercalciuria and hypocitraturia and are not treated with carbonic anhydrase inhibitors. Decreased magnesium reabsorption in the thick ascending limb of the loop of Henle (Choice C) can occur as a side effect of treatment with gentamicin. Calcium oxalate stones are not precipitated by increased concentrations of urinary magnesium. Increased phosphate reabsorption in the proxima

81 Exam Section 2: Item 31 of 50 National Board of Medical Examiners® Comprehensive Basic Science Self-Assessment 31. The table reports survival of patients who had an operation for a particular form of cancer: Number of Patients at the Number of Patients Who Percent of Patients Surviving This Interval 80 Interval Beginning of the Interval Died During the Interval 500 0-1 year 1-2 years 400 2-3 years 350 3-4 years 315 100 50 35 31 87.5 90 90 Which of the following is the probability of a patient surviving 4 years? A) (.875 - .80) x 90 x .90 B).875 C) .90 D) .80 x .875 x 90 x .90 E) (.875 - .80) x .90

D. A survival analysis is used to convey the fraction of a group of patients who are alive at a given time point after an intervention has been made. Graphically, this is often done with a Kaplan-Meier survival curve, the data for which is shown in the table. For this particular cancer the time interval of 0 to 1 year is accompanied by a 20% death rate, or 80% survival rate. This means that if 100 patients were given the same diagnosis and treated with the same operation, 80 of them would be living at one year. After surviving to 1 year, there is then a likelihood of 87.5% that they would survive to 2 years, and so on. This analysis allows prognostication for a patient newly diagnosed with this cancer when that patient's outcome is yet unknown. The probability that a patient with a new diagnosis who undergoes treatment survives until 4 years is the combined probability of surviving until 1 year, then 2 years, then 3 years, and finally 4 years. This is represented by 0.80 x 0.875 × 0.90 x 0.90. Incorrect Answers: A, B, C, and E. Calculating the 4-year survival rate does not require calculating the difference between two probabilities. Neither (.875 - 80) x .90 x .90 (Choice A) nor (.875 - .80) x .90 (Choice E) will correctly calculate the combined probability of survival through all four years. .875 (Choice B) is the survival rate of patients between the 1st and 2nd year after diagnosis. It does not take into account the probability of the first, third or fourth years. Likewise, .90 (Choice C) is the survival rate of patients between either the 2 nd and 3rd year or 3rd and 4th year after diagnosis. The probability of surviving from diagnosis to the end of the fourth year is the product of the probabilities of surviving each year. Educational Objective: Kaplan-Meier estimates are used to provide prognostic data to patie

142 Exam Section 3: Item 42 of 50 National Board of Medical Examiners' Comprehensive Basic Science Self-Assessment 42. A 44-year-old man with a 25-year history of alcoholism is brought to the emergency department by his wife because of a 1-day history of confusion. The wife reports that the patient's personality has recently changed and that he vomited a small amount of blood several days ago. He last consumed alcohol 2 weeks ago. Liver function tests 3 years ago showed abnormal findings. He is alert and disheveled. His pulse is 82/min, respirations are 20/min, and blood pressure is 100/64 mm Hg. Physical examination shows signs of mild dehydration, mild jaundice, spider angiomata over the face and chest, and asterixis. There are no tremors or ophthalmoplegia. The abdomen is distended with bulging flanks and shifting dullness. The spleen tip is palpated 3 cm below the left costal margin. There is testicular atrophy.

D. Cirrhosis can present with edema, ascites, increased bilirubin, jaundice, spider angiomata, and sequelae of portal hypertension such as esophageal varices, splenomegaly, caput medusae, rectal varices, and hepatic encephalopathy. It typically occurs in patients with preceding conditions such as alcohol use disorder or chronic hepatitis. In the setting of portal hypertension, retrograde blood flow between tributaries of the portal system and systemic veins may occur, forming portosystemic anastomoses. This leads to the formation of esophageal, periumbilical, and rectal varices. A consequence of portosystemic shunting is hepatic encephalopathy (HE). HE occurs as a result of ammonia (NH) build up in the body caused by decreased NH3 metabolism in the urea cycle. In cirrhosis, the blood carrying NH3 does not entirely pass through the liver but rather enters directly into the systemic circulation as a result of portosystemic shunting. When NH3 accumulates, it causes neuropsychiatric disturbances ranging from cognitive defects such as impaired attention and memory to altered mental status and coma. Asterixis, a flapping tremor in which the hands move up and down when the arms are outstretched, is a clinical manifestation of hyperammonemia. While the measurement of serum ammonia is commonly performed, venous concentrations do not reliably correlate with the degree of encephalopathy. Lactulose is used to prevent uptake of NH, Degradation of lactulose by colonic bacteria generates hydrogen ions within the gut. The abundance of hydrogen ions converts ammonia (NH) to ammonium (NH,), which cannot be absorbed by the bowel, causing it to be excreted in the feces. Lactulose is given prophylactically to many patients with cirrhosis to prevent HE and is titrated to a target number of bowel movements per day. 4 Incorrect Answers: A, B,

107 Exam Section 3: Item 7 of 50 National Board of Medical Examiners' Comprehensive Basic Science Self-Assessment 7. A 44-year-old woman comes to the physician because of a 3-day history of a persistent nosebleed and a diffuse rash. She has no history of major medical illness and takes aspirin occasionally for headaches. Her vital signs are within normal limits. A photograph of her left foot is shown. Similar findings are on the torso and upper and lower extremities. Laboratory studies show: Hemoglobin Hematocrit 13 g/dL 38% Mean corpuscular volume 85 µm3 Leukocyte count Platelet count Serum Urea nitrogen Creatinine 6200/mm3 <5000/mm3 10 mg/dL 0.8 mg/dL Which of the following is the most likely diagnosis? A) Aspirin toxicity B) Bernard-Soulier syndrome C) Hemolytic uremic syndrome D) Immune thrombocytopenic purpura E) Thrombotic thrombocytopenic purpura

D. Immune thrombocytopenic purpura (ITP) is the result of an autoimmune process leading to the destruction of circulating platelets with subsequent development of thrombocytopenia. Antibodies bind to platelets, which results in their opsonization. Macrophages within the spleen phagocytose the platelet-antibody complex, resulting in thrombocytopenia. ITP is diagnosed in the setting of a low platelet count in the absence of other explanatory causes for thrombocytopenia. Any other cytopenias, including anemia or leukopenia, warrants evaluation for an alternative diagnosis. Bone marrow biopsy will classically show an increased number of megakaryocytes indicating adequate platelet production but increased peripheral destruction of platelets. It may present with petechiae and purpura which occur as a result of microhemorrhages into the skin, and prolonged bleeding time on laboratory analysis. Severe ITP may lead to uncontrolled hemorrhage. Acute ITP often follows an infection, and generally is self-limited. Chronic ITP is first treated with steroids, which may be required long-term, intravenous immunoglobulin , or immunomodulators. Refractory ITP that is non-responsive to such therapies is treated with splenectomy, which results in remission of thrombocytopenia in the majority of cases as a result of the spleen's primary role in the underlying pathophysiology. Incorrect Answers: A, B, C, and E. Aspirin toxicity (Choice A) is often a result of ingestion and presents with tinnitus, nausea, vomiting, and vertigo. Laboratory analysis may show metabolic acidosis and respiratory alkalosis. Aspirin inhibits the function of platelets but does not affect the number of circulating platelets, so thrombocytopenia is not a feature of aspirin toxicity. Bernard-Soulier syndrome (Choice B) is an autosomal recessive disorder resulting in dys

195 Exam Section 4: Item 45 of 50 National Board of Medical Examiners' Comprehensive Basic Science Self-Assessment 45. A 52-year-old man with acute myelogenous leukemia comes to the physician because of a 3-day history of fever and a 2-week history of nonproductive cough. He received ablative chemotherapy and received a bone marrow transplant 2 months ago. He has had neutropenia since the transplant. His temperature is 38.8°C (101.8°F), pulse is 96/min, respirations are 20/min, and blood pressure is 100/65 mm Hg. Crackles are heard over the right lung base. His serum 1,3-B-D-glucan concentrations are increased. A chest x-ray shows a cavitary lesion in the right lower lobe. This patient's serum study result most likely indicates the presence of which of the following? A) Atypical mycobacteria B) Bacterial flagella C) Bacterial spores D) Fungal cell wall E) Fungal ribosomes

D. Immunocompromised and neutropenic patients are at risk for opportunistic and fungal infections such as aspergillus. Aspergillomas are the most common presentation of pulmonary aspergillosis caused by infection of the Aspergillus species, which is characterized by septated hyphae with acute angle branching. Pulmonary aspergillosis can result in many clinical presentations such as aspergilloma, allergic bronchopulmonary aspergillosis, chronic necrotizing aspergillosis, and airway invasive or angioinvasive aspergillosis. Aspergillomas (the formation of fungal masses) arise within existing cavitary lesions of the lung, such as those caused by previous infection with tuberculosis, emphysematous blebs or bullae, or from cavitary squamous cell lung carcinoma. Patients typically present with a persistent, sometimes bloody cough, with a history of a prior cavitary lung disease. Chest x-rays should be obtained, which will show a round, solid nodule or mass within a larger cavity. 1,3-B-D-glucan is a polysaccharide component of fungal cell walls and can be measured in the serum of infected patients. Incorrect Answers: A, B, C, and E. Atypical mycobacteria (Choice A) such as Mycobacterium kansasii can cause chronic pulmonary infections that resemble tuberculosis, presenting with fever, cough, night sweats, and weight loss, often in patients with immunocompromised states. This patient's symptoms are more likely caused by aspergillus. Bacterial flagella (Choice B) function to provide motility and are composed of the protein flagellin. Flagella do not contain 1,3-B-D-glucan. Bacterial spores (Choice C) are formed by some gram-positive bacteria such as Bacillus anthracis, Bacillus cereus, and Clostridium botulinum, difficile, perfringens, and tetani. Spores are able to resist extreme conditions such as dehydration, heat, and chemic

61 Exam Section 2: Item 11 of 50 National Board of Medical Examiners® Comprehensive Basic Science Self-Assessment 11. A 43-year-old man comes to the physician because of a 1-month history of chronic sinus congestion, occasional bleeding from the nose, and intermittent shortness of breath with exertion. He has had no fever or chills but has felt very "run down." His temperature is 37.4°C (99.3°F), pulse is 88/min, respirations are 16/min, and blood pressure is 160/100 mm Hg. Physical examination shows bilateral maxillary and frontal sinus tenderness to palpation and poor transillumination. Isolated crackles are heard in the left mid lung. Cardiac examination shows no abnormalities. Laboratory studies show: 9.6 g/dL 28% Hemoglobin Hematocrit Serum Na+ K+ CI- 132 mEq/L 5 mEq/L 100 mEq/L 21 mEq/L 48 mg/dL 2.4 mg/dL negative positive negative HCO,- Urea nitrogen Creatinine Antinuclear antibody Cytoplasmic antineutrophi

D. Granulomatosis with polyangiitis is a systemic necrotizing granulomatous vasculitis that affects small and medium vessels. It is associated with c-ANCA and primarily affects the sinopulmonary tract, kidneys, and skin. Patients typically present with constitutional symptoms such as fever, weight loss, and fatigue, with additional symptoms based on specific organ involvement. Sinopulmonary manifestations include recurrent sinusitis, otitis media, mastoiditis, nasal ulcerations with epistaxis, hemoptysis, cough, and dyspnea. The kidneys are typically involved, and patients often present with hematuria with an increased risk for developing rapidly progressive glomerulonephritis and renal failure. Skin lesions are common, especially lower extremity purpura. Incorrect Answers: A, B, C, and E. Eosinophilic vasculitis (Choice A) is associated with allergic granulomatous angiitis (Churg-Strauss syndrome), which is a small vessel vasculitis that may present with asthma, nasal inflammation, and lung, gastrointestinal, cardiac, or kidney involvement. Pathology shows a granulomatous, necrotizing vasculitis and eosinophilia. Leukocytoclastic vasculitis (Choice B) refers to small vessel vasculitis that primarily involves the skin. It affects women more than men, and typically presents with a painful, burning rash as well as systemic symptoms of fever, weight loss, fatigue, and myalgias. Palpable purpura is commonly noted on physical examination. Lymphomatoid granulomatosis (Choice C) is a rare lymphoproliferative disorder that most commonly affects the lungs. Patients may also develop a patchy rash and subcutaneous nodules. In this patient, sinopulmonary involvement is more suggestive of granulomatosis with polyangiitis. Temporal arteritis (Choice E) is an autoimmune vasculitis that affects large and medium sized arteries. It is m

157 Exam Section 4: Item 7 of 50 National Board of Medical Examiners' Comprehensive Basic Science Self-Assessment 7. A 30-year-old woman has diarrhea and crampy abdominal pain. Which of the following historical features most strongly suggests that the cause is inflammatory bowel disease rather than irritable bowel syndrome? A) Cramping left lower quadrant pain B) Lactose intolerance C) More than three bowel movements daily D) Nighttime awakening caused by pain and diarrhea E) Stressful family relationships

D. Inflammatory bowel disease comprises both Crohn disease and ulcerative colitis. Crohn disease is characterized by transmural inflammation that can involve any portion of the gastrointestinal tract, manifesting with skip lesions. It particularly involves the terminal ileum and typically spares the rectum. Clinically, it can present with chronic abdominal pain, low-grade fever, weight loss, and watery or bloody diarrhea. Irritable bowel syndrome presents with recurrent abdominal pain, which often improves with defecation, along with a change in frequency and consistency of stool. Associated alterations in bowel habits can occur with intermittent episodes of diarrhea and/or constipation. Emotional stress and certain foods may exacerbate symptoms of irritable bowel syndrome. Both inflammatory bowel disease and irritable bowel syndrome can present with abdominal pain and diarrhea. While decreased sleep quality can increase the severity of symptoms of irritable bowel syndrome during the day, nighttime awakening caused by pain and diarrhea is more indicative of an organic cause such as inflammatory bowel disease. Features that are suggestive of underlying disease rather than irritable bowel syndrome include occult blood or melena, nocturnal diarrhea, worsening abdominal pain, weight loss, and a family history of inflammatory bowel disease or colon cancer. Incorrect Answers: A, B, C, and E. Abdominal pain and cramping left lower quadrant pain (Choice A) is nonspecific and can occur with both inflammatory bowel disease and irritable bowel syndrome. In Crohn disease, inflammation of the terminal ileum can cause right lower quadrant pain. Left lower quadrant pain may be associated with diverticulitis. Irritable bowel syndrome has been associated with lactose intolerance (Choice B). Individuals with lactose intolerance can also

95 Exam Section 2: Item 45 of 50 National Board of Medical Examiners® Comprehensive Basic Science Self-Assessment 45. A 54-year-old woman comes to the physician because of a 10-day history of episodes of severe stabbing pain of her left cheek and jaw. She says that the pain occurs suddenly and can be precipitated by chewing, speaking, or brushing her teeth; it lasts several seconds and occurs 20 to 30 times daily. She adds, "It feels like lightning is striking my cheek." Touching the left cheek reproduces her symptoms. Which of the following is the most likely cause of this patient's symptoms? A) Compression of the trigeminal nerve by an acoustic neuroma (vestibular schwannoma) B) Inflammation of the temporomandibular joint C) Inflammation of the trigeminal nerve by a multiple sclerosis plaque D) Microvascular compression of the trigeminal nerve E) Recurrent transient ischemia in the vertebrobasilar system

D. Trigeminal neuralgia is a neuropathic pain disorder that presents with severe shooting or burning pain within the innervated regions of the sensory branches of cranial nerve V (the trigeminal nerve). Cranial nerve V divides into three sensory branches, V, to V3, which innervate the forehead (V,), the skin overlying the zygomatic and maxillary bone (V2), and the skin overlying the mandible (V). The pathophysiology of the pain is believed to be related to compression of the trigeminal nerve by adjacent blood vessels or bone as it exits the skull base. Symptoms may be triggered by light touch, shaving, swallowing, or may occur without any precipitant. The pain is generally described as severe. Treatment includes carbamazepine or surgical decompression. Incorrect Answers: A, B, C, and E. Compression of the trigeminal nerve by an acoustic neuroma (vestibular schwannoma) (Choice A) would be unlikely, as acoustic neuromas generally occur in association with cranial nerve VIII and generally cause sensorineural deafness, vertigo, and disequilibrium. Pain would be an unusual presentation, as would a lesion large enough to also affect cranial nerve V. Inflammation of the temporomandibular joint (Choice B) would cause pain with chewing and be specifically located in the area of the temporomandibular joint. Patients may also complain of locking of the jaw. Inflammation of the trigeminal nerve by a multiple sclerosis plaque (Choice C) is a known cause of trigeminal neuralgia, however this patient demonstrates no symptoms of multiple sclerosis such as neurologic dysfunction affecting multiple body systems over time (eg, urinary incontinence, weakness, or optic neuritis). Recurrent transient ischemia in the vertebrobasilar system (Choice E) presents with functional loss affecting the brainstem and cerebellum, such as vertigo, ataxi

65 Exam Section 2: Item 15 of 50 National Board of Medical Examiners® Comprehensive Basic Science Self-Assessment 15. A 48-year-old man comes to the physician because he is concerned about a pigmented lesion that has enlarged and changed color during the past 3 months. Physical examination shows a 2-cm oval lesion. Histologic examination of an excisional biopsy specimen shows melanoma. If a biopsy of the lesion had been done prior to the malignant transformation, which of the following would have been the most likely diagnosis? A) Acanthosis nigricans B) Actinic keratosis C) Freckle D) Nevocellular nevus E) Seborrheic keratosis

D. Malignant melanoma is likely to be present when a lesion demonstrates asymmetry, irregular-appearing borders, variable coloration, a diameter greater than 6 mm, and rapid evolution in characteristics. It is a neoplastic proliferation of melanocytes and can develop within a giant congenital nevus (nevocellular nevus). Compared to acquired nevi, congenital nevi tend to be larger, darker in color, thicker, and more likely to contain hair. These nevi should be monitored by the patient for development of any features concerning for malignant degeneration. Malignant melanoma has the ability to rapidly invade and metastasize, which carries a poor prognosis when diagnosed late. Subtypes include superficial spreading, nodular, lentigo maligna, and acral lentiginous. Any lesion with features suggestive of malignant melanoma should be surgically excised with negative margins and pathologically examined for the depth of dermal invasion. Incorrect Answers: A, B, C, and E. Acanthosis nigricans (Choice A) is characterized by hyperpigmented, velvety patches seen on the neck, upper back, breasts, and axillae which is a marker of metabolic syndrome and diabetes mellitus. It is neither premalignant nor malignant and there are no atypical cells on histopathologic examination. Actinic keratosis (Choice B) is a premalignant lesion that may progress to squamous cell carcinoma. They present as chronic rough, scaly patches of skin in areas of prolonged sun exposure (eg, face, ears, hands). Freckle (Choice C), or ephelis, is a benign, light tan macule which darkens with sun exposure. It is caused by an increase in melanin production and is neither premalignant nor malignant. Seborrheic keratosis (Choice E) is a benign proliferation of the epidermis; lesions exhibit a greasy, adherent appearance. While seborrheic keratoses are often brown, th

26 Exam Section 1: Item 26 of 50 National Board of Medical Examiners® Comprehensive Basic Science Self-Assessment LDS 1 cm 26. A 65-year-old woman dies 6 months after the onset of severe headaches. Her brain as seen at autopsy is shown in the photograph. Which of the following is the most likely cell of origin of this neoplasm? A) Astrocyte O B) Endothelial OC) Melanocyte D) Meningeal E) Squamous epithelial

D. Meningiomas are the most common primary brain tumors in adults and arise from the meninges (the protective coverings of the brain). This patient's meningioma is pictured along the inferior aspect of the forebrain within the anterior interhemispheric fissure between the frontal lobes, resulting in significant mass effect and compression of the adjacent frontal lobes and olfactory bulbs. Patients may be asymptomatic, or experience seizures or focal neurologic deficits related to the tumor location. Meningiomas are typically benign but can be malignant. Malignant meningiomas may grow rapidly, leading to symptoms of increased intracranial pressure (postural headache, nausea, papilledema), or may metastasize. Meningiomas appear as well-circumscribed, extra-axial (outside of the brain parenchyma) masses that compress the parenchyma, as in this patient. Treatment may include surgical resection to alleviate the tumoral mass effect if significant. Incorrect Answers: A, B, C, and E. Astrocytes (Choice A) proliferate to form glial tumors in adults, for example, glioblastoma. These highly malignant tumors appear as intra-axial masses in the cerebral hemispheres, which typically demonstrate areas of necrosis and can spread to the opposite hemisphere across the corpus callosum. Endothelial cells (Choice B) may proliferate to form hemangioblastomas.Hemangioblastomas typically arise in the posterior fossa as intra-axial tumors. Tumors arising from melanocytes (melanomas) (Choice C) or squamous epithelial cells (squamous cell carcinomas) (Choice E) are primarily extracranial and would not compress the brain. Brain metastases from these malignancies would most likely be intra- axial rather than extra-axial. Educational Objective: Meningiomas are the most common primary brain tumors in adults. Malignant meningiomas may grow rapidly an

163 Exam Section 4: Item 13 of 50 National Board of Medical Examiners' Comprehensive Basic Science Self-Assessment 13. A3-year-old boy has progressive coarsening of his facial features, hirsutism, hepatomegaly, and developmental delay. X-rays of the skeletal system show dysostosis multiplex. Which of the following is primarily involved in this condition? A) Cilia O B) Cytosol C) Endoplasmic reticulum D) Lysosome E) Mitochondrion F) Peroxisome G) Plasma membrane H) Ribosome

D. Mucopolysaccharidoses are a type of lysosomal storage disorder caused by an enzyme deficiency that prevents the degradation of glycosaminoglycans in the lysosome. The buildup of atypical substrates leads to varying concentrations of cellular dysfunction based on the enzyme affected. The diagnosis is not often clinically apparent at birth as the abnormal deposition of metabolites has not had time to occur. Mucopolysaccharidosis I and II (Hurler and Hunter syndromes) are the two primary mucopolysaccharidoses and are caused by deficiencies in a-iduronidase and iduronate-2-sulfatase, respectively, with both resulting in the deposition of heparan sulfate and dermatan sulfate. Hurler syndrome is an autosomal recessive disorder and presents in the first year of life with coarse facial features, corneal clouding, hepatosplenomegaly, skeletal abnormalities (eg, dysostosis multiplex), joint contractures, intellectual disability, and developmental delay, as well as heart failure, hydrocephalus, hearing loss, and recurrent respiratory tract infections. Hunter syndrome is an X-linked recessive disorder and presents with milder features of Hurler syndrome with the absence of corneal clouding and addition of aggressive behavior and pearly papular skin lesions. Diagnosis is made with urinary glycosaminoglycans and an analysis of enzyme activity; treatment is largely supportive. Average lifespan is five years of age in Hurler syndrome and 13 to 20 years of age for Hunter syndrome. Incorrect Answers: A, B, C, E, F, G, and H. Cilia (Choice A) abnormalities are involved in primary ciliary dyskinesia, which presents with recurrent respiratory infections, situs inversus, and infertility. It is also not often clinically apparent at birth. Cytosol (Choice B) abnormalities are not involved in the development of Hurler syndrome. Other geneti

197 Exam Section 4: Item 47 of 50 National Board of Medical Examiners' Comprehensive Basic Science Self-Assessment 47. A72-year-old man with chronic obstructive pulmonary disease comes to the physician for a follow-up examination. Current medications include albuterol and ipratropium. He smokes 1 pack of cigarettes daily. He does not drink alcohol. His respirations are 24/min. Pulse oximetry on room air shows an oxygen saturation of 87%. Breath sounds are decreased bilaterally. His baseline FEV, is 0.88 (38% of predicted). Which of the following hematologic abnormalities is most likely in this patient? A) Eosinophilia B) Lymphocytosis C) Lymphopenia D) Polycythemia E) Thrombocytopenia OF) Thrombocytosis

D. Secondary polycythemia is the most likely hematologic abnormality present in this patient with severe chronic obstructive pulmonary disease (COPD) (FEV, of 30-50%) and resting hypoxia with oxygen saturation of 87%. Polycythemia refers to an abnormally high hematocrit or hemoglobin concentration, which in men is usually above 18 g/dL. Polycythemia may occur as a result of primary myeloproliferative neoplasms such as polycythemia vera or essential thrombocytosis, or it may be secondary to an alternative cause. Common causes of secondary polycythemia include COPD, obstructive sleep apnea, and cyanotic heart disease, as decreased peripheral tissue oxygenation causes increased secretion of erythropoietin (EPO) from the kidneys. Increased concentrations of EPO stimulate production of hemoglobin and erythrocytes. In these instances, patients will have increased serum EPO concentrations and evidence of hypoxia. Alternative causes of secondary polycythemia include EPO-producing tumors such as renal cell carcinoma, hepatocellular carcinoma, and pheochromocytoma. In younger men with polycythemia and no clear explanation, anabolic steroid use should always be considered. Incorrect Answers: A, B, C, E, and F. Eosinophilia (Choice A) can sometimes be seen in patients with COPD-asthma overlap syndromes and can be predictive of clinical response to inhaled corticosteroids. However, the eosinophil count is not affected by blood oxygen saturation. Lymphocytosis (Choice B) refers to an increased number of circulating lymphocytes and can occur in malignancies of lymphoid cells such as acute lymphoblastic leukemia; it may also occur as a result of viral infections. A finding of lymphocytosis in this patient with severe COPD would be unexpected in the absence of another underlying cause. Lymphopenia (Choice C) refers to a decreased numbe

74 Exam Section 2: Item 24 of 50 National Board of Medical Examiners® Comprehensive Basic Science Self-Assessment 24. Two days after admission to the hospital because of a sickle cell disease crisis, a 24-year-old man suddenly develops blood in his urine and flank pain. Laboratory studies at the time of admission showed that his hematocrit was decreased to 11% from his normal baseline of 25%. Urinalysis shows gross blood, rare WBCS, and no WBC casts. Which of the following is the most likely cause of the hematuria in this patient? OA) Glomerulonephritis B) Nephrolithiasis C) Prostatitis D) Renal papillary necrosis E) Transitional cell carcinoma

D. Renal papillary necrosis (RPN) occurs following ischemic, inflammatory, infectious, or toxin-mediated damage to the renal papilla and describes the sloughing and loss of the papillae including substructures such as the distal collecting tubule. RPN can be triggered by infections (eg, acute pyelonephritis), diabetes mellitus, sickle cell disease, or nonsteroidal anti-inflammatory medications (NSAIDS). In sickle cell disease, renal papillary necrosis is common and occurs secondary to renal medullary ischemia and infarction. It typically presents with hematuria and acute flank pain. It can also be complicated by urinary tract obstruction and/or infection secondary to necrotic tissue sloughing into the renal collecting system and ureters. It characteristically presents with gross hematuria and proteinuria on urinalysis. Incorrect Answers: A, B, C, and E. Glomerulonephritis (Choice A) refers to a variety of glomerular diseases, including nephritic and nephrotic syndromes. Nephritic syndromes typically present with acute renal failure with associated hematuria, red blood cell urine casts, and hypertension. Nephrotic syndrome typically presents with excessive proteinuria (greater than 3g per day) hyperlipidemia, hypoalbuminemia, and edema. It would not typically cause ureteral obstruction. Nephrolithiasis (Choice B) typically presents with unilateral flank pain that is colicky and sometimes radiates to the groin with associated hematuria. In a patient with sickle cell crisis, renal papillary necrosis is a more likely cause of flank pain and hematuria. Prostatitis (Choice C) can present with dysuria, frequency, and urinary urgency similar to a urinary tract infection. On examination, a tender, boggy, enlarged prostate can be palpated and treatment is with antibiotics. Urinalysis would show WBCs and would be unlikely to indi

120 Exam Section 3: Item 20 of 50 National Board of Medical Examiners® Comprehensive Basic Science Self-Assessment 20. A 4-year-old girl is conscious but unable to breathe spontaneously for 3 hours following uncomplicated removal of a branchial cleft cyst. The procedure was done under general anesthesia with sevoflurane. Succinylcholine was administered for intubation. Which of the following is the most likely cause of the prolonged apnea in this patient? A) Idiosyncratic reaction to sevoflurane anesthesia B) Intraoperative vagus nerve injury during the operation C) Overdose with midazolam used as preoperative sedation D) Pseudocholinesterase deficiency E) Ryanodine receptor defect

D. Succinylcholine is a depolarizing skeletal muscle relaxant commonly used to facilitate rapid sequence tracheal intubation. The generally short duration of action is because of its rapid metabolism by pseudocholinesterase. Succinylcholine is a potent acetylcholine receptor agonist that causes depolarization of the muscle fibers, seen clinically as fasciculations, followed by muscle relaxation. It is structurally similar to acetylcholine but is not metabolized by acetylcholinesterase. Its alternative metabolism causes it to persist in the synaptic cleft for a longer duration, leading to prolonged depolarization and prevention of further skeletal muscle activation by acetylcholine. As serum concentration falls, it diffuses away from the neuromuscular junction. It is then hydrolyzed by pseudocholinesterase in the plasma and liver. Therefore, a genetic deficiency of pseudocholinesterase would prevent succinylcholine from being metabolized, which would lead to an abnormally slow recovery from neuromuscular blockade. Incorrect Answers: A, B, C, and E. An idiosyncratic reaction to sevoflurane anesthesia (Choice A) would more likely present with an altered level of consciousness, given sevoflurane's hypnotic and sedative effects. It would be less likely than pseudocholinesterase deficiency in this patient with muscle (diaphragmatic) weakness following succinylcholine administration. Intraoperative vagus nerve injury during the operation (Choice B) would be expected to present with unilateral diaphragmatic weakness or paralysis, as well as changes in voice quality caused by the vagus nerve's laryngeal branches. Following vagus nerve injury, the patient would be able to breathe spontaneously but would be unlikely to obtain adequate tidal volumes. Overdose with midazolam used as preoperative sedation (Choice C) would manifest a

192 Exam Section 4: Item 42 of 50 National Board of Medical Examiners' Comprehensive Basic Science Self-Assessment 42. A 35-year-old woman, gravida 1, para 1, comes to the physician because of a 2-month history of generalized weakness and fatigability that are significantly exacerbated by exercise. The symptoms began during her pregnancy. Physical examination shows diplopia. There is reduction in arm muscle strength with repetitive movement, but preservation of deep tendon reflexes. A therapeutic trial of oral pyridostigmine provides symptomatic relief. Serum studies will most likely show antibodies to which of the following proteins? A) Acetylcholinesterase B) Muscarinic acetylcholine receptor C) Myeloperoxidase D) Nicotinic acetylcholine receptor

D. This patient with symptoms of myasthenia gravis likely possesses antibodies to the nicotinic acetylcholine receptor (NACHR) on skeletal muscle cells. When acetylcholine binds the NACHR, there is a resultant influx of sodium (and a lesser degree of potassium efflux), which depolarizes the muscle cell and increases the frequency of action potentials. Myasthenia gravis is an autoimmune disorder of neuromuscular transmission. Patients with myasthenia gravis possess antibodies that block or destroy NACHRS, which interferes with neuromuscular junction signaling. Many patients with myasthenia gravis have thymic hyperplasia or thymoma, which may be involved in the pathogenesis of autoimmunity. Patients present with progressively worsening skeletal muscle weakness at rest and fatigability on activation of focal or generalized muscle groups. The ocular, bulbar, and neck muscles are commonly affected. Most concerningly, respiratory muscle weakness can lead to respiratory insufficiency, and in an acute flare, these patients may require mechanical ventilation. Management includes symptomatic treatments such as acetylcholinesterase inhibitors (eg, pyridostigmine), which increase the synaptic availability of acetylcholine, along with immune modulating therapies. In patients with or without thymoma, complete thymectomy may be curative. Some women experience myasthenia gravis exacerbations during pregnancy. In pregnant patients, the nAChR antibodies may be transmitted transplacentally, necessitating that neonates be monitored for transient symptoms of myasthenia gravis. Incorrect Answers: A, B, and C. Antibodies to acetylcholinesterase (Choice A) would be atypical for myasthenia gravis. Antibodies to acetylcholinesterase would decrease acetylcholine degradation and therefore facilitate neuromuscular junction signaling. Antibodies to

9 Exam Section 1: Item 9 of 50 National Board of Medical Examiners® Comprehensive Basic Science Self-Assessment 9. A 66-year-old man with type 2 diabetes mellitus and hypertension is brought to the emergency department 30 minutes after the sudden onset of left eyelid drooping, double vision, and mild weakness of the right hand and leg. His pulse is 88/min and regular, and blood pressure is 159/99 mm Hg. Examination of the head shows a substantially droopy left eyelid, and in primary gaze, the left eye is exotropic and somewhat lower than the right. He has slowed finger movements on the right, a pronator drift with the right hand, and mild hyperreflexia on the right. Which of the following is the most likely site and diagnosis of the patient's lesion? A) Angular gyrus (Gerstmann syndrome) B) Dorsolateral thalamus (Dejerine-Roussy syndrome) C) Lower medulla (Wallenberg syndrome) D) Lower midbrain (Weber syndrome) E) U

D. This patient likely has a lesion of the lower midbrain that affects the cerebral peduncle and the oculomotor nerve, which causes contralateral hemiparesis and ipsilateral oculomotor palsy, known as Weber syndrome. The cerebral peduncle refers to the anterior portion of the midbrain and includes the crus cerebri, which contains the corticospinal tract and is supplied by paramedian branches of the posterior cerebral artery. Upper motor neurons of the corticospinal tract originate in the primary motor cortex, descend ipsilaterally through the internal capsule and midbrain (within the crus cerebri), decussate in the caudal medulla, and then descend contralaterally in the spinal cord to synapse with the contralateral lower motor neuron. A brainstem lesion of the corticospinal tract leads to contralateral weakness in an upper motor neuron pattern of dysfunction (eg, spastic paralysis, pronator drift, and hyperreflexia). The oculomotor nerve (cranial nerve III) is located anteriorly between the two cerebral peduncles and may also be affected by strokes of the paramedian branches of the posterior cerebral artery, leading to ipsilateral dysfunction of the extraocular muscles and consequent diplopia along with ptosis and a deficit in pupillary constriction. CVAS occur because of ischemic or hemorrhagic loss of blood supply to the brain. Approximately 80-85% of CVAS are ischemic, commonly arising from thromboembolic disease (eg, middle cerebral artery occlusion from a thrombus), whereas 15-20% of CVAS are hemorrhagic and caused by blood vessel rupture (eg, hypertension-related intraparenchymal hemorrhage from a perforating artery). Risk factors for CVAS include smoking, hypertension, diabetes mellitus, carotid or intracranial atherosclerotic disease, history of hypercoagulability, atrial fibrillation, and advanced age. Classic

94 Exam Section 2: Item 44 of 50 National Board of Medical Examiners® Comprehensive Basic Science Self-Assessment 44. A 3-month-old boy is brought to the physician because of yellow eyes and skin and weakness since birth. Physical examination shows jaundice, large fontanels, a flat midfacial area, hypotonia, and hepatomegaly. Serum studies show: Total bilirubin (mainly direct) AST ALT increased increased increased increased Very-long-chain fatty acids A liver biopsy specimen shows foamy, lipid-filled hepatocytes, necrosis, and absence of a specific organelle. This organelle is most likely which of the following? O A) Golgi complex B) Lysosomes C) Mitochondria D) Peroxisomes E) Smooth endoplasmic reticulum

D. Zellweger syndrome is caused by a genetic mutation that leads to an absence of peroxisomes. Peroxisomes synthesize cholesterol and bile acids, as well as cell membrane substrates. As peroxisomes are also responsible for metabolizing very long chain fatty acids (VLCFA), phytanic acid, pipecolic acid, pristanic acid, hydrogen peroxide, and ethanol, their absence leads to an accumulation of these products. Zellweger syndrome classically presents with seizures, intellectual disability, hypotonia, hepatomegaly, jaundice, kidney disease, cataracts, hearing loss, and craniofacial abnormalities. Physical examination will show an absence of reflexes, and results of laboratory studies will show increased concentrations of VLCFA, hepatic transaminases, and direct bilirubin, as observed in this patient. There is no effective treatment, and life expectancy is less than six months of age. Incorrect Answers: A, B, C, andE. The golgi complex (Choice A) sorts, modifies, and transports proteins within the cell and to the cell membrane for export. Deficiency of the golgi complex in hepatocytes would likely lead to cell death, although it would not result in an increased concentration of VLCĒAS. Lysosomes (Choice B) are responsible for the degradation of a variety of substrates, including those brought into the cell through endocytosis and materials of the cell itself (eg, autophagy). Deficiencies in lysosomal enzymes lead to lysosomal storage disorders, which present broadly with neurocognitive decline, skeletal abnormalities, and craniofacial abnormalities. Mitochondria (Choice C) are responsible for producing energy for the cell in the form of adenosine triphosphate. Disorders of mitochondrial structure or function include MELAS (mitochondrial encephalomyopathy, lactic acidosis, and stroke-like episodes) syndrome and myoclonic epil

169 Exam Section 4: Item 19 of 50 National Board of Medical Examiners' Comprehensive Basic Science Self-Assessment 19. A 24-year-old man comes to the physician because his wife has been unable to conceive for the past 2 years. Previous evaluation of his wife showed no abnormalities. He has no history of major medical illness. He does not smoke cigarettes. Physical examination shows a left testicle that is smaller than the right and an enlarged left spermatic cord that increases in size with Valsalva maneuver. Laboratory studies show oligospermia. Which of the following is the most likely diagnosis? A) Cryptorchidism B) Epididymitis C) Hypospadias D) Testicular torsion E) Varicocele

E. A varicocele refers to dilated veins of the pampiniform plexus caused by increased venous pressure. Varicoceles are commonly located on the left side because of increased resistance to venous flow as the left gonadal vein drains into the left renal vein. On the right side, the right gonadal vein drains directly into the inferior vena cava and is less likely to become obstructed. The dilation of the pampiniform plexus causes a bluish appearance of the overlying skin. Palpation of the dilated plexus demonstrates varicosities, which are often described as having a vermiform (worm-like) feeling. Unlike a hydrocele, which is caused by failure of the processus vaginalis to close leading to accumulation of fluid around the testicle, varicocele will not transilluminate with application of light to the scrotum. Infertility is a potential complication of varicoceles, as the dilated veins result in increased temperature of the testicle which impairs spermatogenesis. Treatment for varicoceles may require surgical embolization or ligation to manage infertility. Incorrect Answers: A, B, C, and D. Cryptorchidism (Choice A) is the failure of one or both testes to descend into the scrotum. An undescended testis may be found anywhere along the typical path of descent and, if left uncorrected for a prolonged duration, will caused infertility as a result of suboptimal spermatogenesis caused by increased temperature of the testicular environment. In this patient, both testes are palpated in the scrotum. Epididymitis (Choice B) is a common cause of painful scrotal swelling and refers to acute infection and inflammation of the epididymis. In younger men, this is commonly secondary to sexually transmitted infections such as Chlamydia trachomatis or Neisseria gonorrhoeae. In older patients, Escherichia coli is more common. Hypospadias (Choi

187 Exam Section 4: Item 37 of 50 National Board of Medical Examiners' Comprehensive Basic Science Self-Assessment 37. An 18-year-old man has had temperatures to 38.3°C (101°F), a sore throat, and cervical lymph node enlargement for 8 days. A latex agglutination test result for Epstein-Barr virus antibody is positive. Atypical cells with abundant lacey cytoplasm in the peripheral blood smear are most likely derived from which of the following types of cells? A) B lymphocytes B) Basophils C) Neutrophils D) Plasma cells E) T lymphocytes

E. Atypical cells seen on the peripheral smear in patients with mononucleosis are T lymphocytes. Infectious mononucleosis is a viral illness caused most often by the Epstein-Barr virus (EBV). EBV is transmitted through respiratory secretions and saliva, causing the illness to be commonly acquired by teenagers and young adults. EBV infects B lymphocytes through CD21. While B lymphocytes are the cell type infected, atypical lymphocytes seen on peripheral blood smear are actually cytotoxic, CD8+ T lymphocytes, which are reacting to the viral infection rather than infected B İymphocytes. As a result, patients often have leukocytosis with a lymphocytic predominance. Cytomegalovirus (CMV) is another less common cause of infectious mononucleosis and an acute HIV infection may also present with a similar syndrome. Clinically, the syndrome commonly presents with fever, lymphadenopathy (typically involving the posterior cervical Tymph nodes), and hepatosplenomegaly along with pharyngitis. Patients with infectious mononucleosis will typically test positive on the monospot test, which detects heterophile antibodies through the agglutination of sheep or horse erythrocytes. Treatment is supportive and patients should be counseled to avoid any contact sports until the splenomegaly has resolved because of the risk for splenic rupture. Incorrect Answers: A, B, C, and D. B lymphocytes (Choice A) are impossible to distinguish from T lymphocytes on the peripheral smear. Flow cytometry is required to identify specific surface markers that differentiate the two. Basophils (Choice B) seen in great quantities on the peripheral smear are associated with chronic myelogenous leukemia (CML), but additional findings would be expected, including the presence of a wide spectrum of granulocytes and hyperleukocytosis. CML would be uncommon in a young

131 Exam Section 3: Item 31 of 50 National Board of Medical Examiners' Comprehensive Basic Science Self-Assessment 31. A 45-year-old woman has a 2-week history of a heart murmur and hepatomegaly. She has a 3-month history of flushing with hypotension and crampy diarrhea. Which of the following is the most likely site of a primary neoplasm in this patient? A) Adrenal gland B) Bone marrow OC) Kidney D) Ovary E) Small intestine

E. Carcinoid syndrome is a rare disorder that results from the production of high concentrations of serotonin by a carcinoid tumor. Carcinoid tumors are neuroendocrine cell neoplasms that often occur in the gastrointestinal tract, most commonly in the small bowel, though they can occur as a primary lung lesion as well. Normally, the liver eliminates active serotonin secreted by a primary gastrointestinal carcinoid tumor; however, if the cancer has metastasized or if the primary lesion has developed outside of the gastrointestinal tract, physiologically active serotonin enters the systemic circulation and manifests as carcinoid syndrome. Symptoms include weight loss, flushing, diarrhea, bronchospasm, right-sided cardiac valvular disease, and abdominal pain. Physical examination may disclose erythema, wheezing, and a cardiac murmur dependent on which valve is involved (tricuspid valve is commonly affected). Diagnosis can be confirmed by measuring urine concentration of 5-hydroxyindoleacetic acid (5-HIAA). 5-HIAA is the primary metabolite of serotonin, and increased urine concentrations suggest increased concentrations of serotonin in the blood. Additional diagnostic information can be obtained by CT scan and octreotide scintigraphy to localize the tumor and evaluate for metastatic disease. Treatment consists of surgical resection and medical therapy with octreotide (a somatostatin analog). Patients may suffer from carcinoid heart disease if the cardiac valves are affected, with the eventual consequent development of right heart failure. Incorrect Answers: A, B, C, and D. Adrenal glands (Choice A) are the site of origin of pheochromocytoma. Pheochromocytoma presents with episodes of increased blood pressure, headache, and palpitations, commonly because of a tumor of the adrenal medulla. Bone marrow (Choice B) is the prima

178 Exam Section 4: Item 28 of 50 National Board of Medical Examiners' Comprehensive Basic Science Self-Assessment 28. A 6-year-old boy who recently emigrated from Russia is brought to the physician by his parents because of unstable gait and incoordination for 2 weeks. He has had frequent pale, bulky stools for 4 years and two episodes of bacterial pneumonia and chronic cough since the age of 1 year. He is below the 3rd percentile for height and weight. Increased rhonchi are heard over both lung fields. Neurologic examination shows ataxia, absence of deep tendon reflexes, and loss of proprioception. Stool analysis shows an increased fat concentration. Which of the following vitamins is most likely deficient in this patient? A) Biotin B) Niacin C) Vitamin C D) Vitamin D E) Vitamin E

E. Cystic fibrosis is an autosomal recessive disorder as a result of a defect in the CFTR gene, leading to a deficiency in a chloride channel that secretes chloride in the lungs and gastrointestinal tract and reabsorbs chloride in sweat glands. This abnormal chloride transport causes decreased chloride and water secretion and increased water reabsorption, leading to abnormally viscous mucous in the lungs and gastrointestinal tract. The thick mucus in the gastrointestinal tract contributes to pancreatic insufficiency, steatorrhea, and malabsorption, leading to a deficiency of fat-soluble vitamins A, D, E, and K. Vitamin E is an antioxidant that protects cells from free radical damage. Deficiency may present with hemolytic anemia, generalized muscle weakness, and posterior column and spinocerebellar tract demyelination. This causes a similar presentation to vitamin B12 deficiency with an ataxic gait and impaired proprioception in the lower extremities. This patient's recurrent pneumonia, chronic cough, rhonchi, and small height and weight are all additional manifestations of cystic fibrosis. Incorrect Answers: A, B, C, and D. Biotin (Choice A) deficiency is characterized by dermatitis, alopecia, and enteritis. It is relatively rare but may be caused by ingestion of large amounts of raw egg whites, as they bind biotin and prevent its absorption. Niacin (nicotinic acid, vitamin B) (Choice B) deficiency can result in glossitis, diarrhea, neuropsychological disturbances such as dementia and hallucinations, and dermatitis. All B vitamins are water soluble, not fat soluble, so absorption is intact even in the case of intestinal fat malabsorption. Vitamin C (Choice C) is found in fruits and vegetables and is necessary for collagen synthesis, iron absorption, immune function, and conversion of dopamine to norepinephrine. Deficie

100 Exam Section 2: Item 50 of 50 National Board of Medical Examiners® Comprehensive Basic Science Self-Assessment 50. A 16-month-old girl is brought to the physician because she has refused to move her right arm since her older brother grabbed her right hand 1 hour ago to stop her from running into the street. On examination, she is holding the right arm in pronation. Any motion of the right elbow produces pain. There is no visible edema. Which of the following is the most likely explanation for these findings? A) Dislocation of the shoulder B) Fracture of the distal radius and ulna C) Salter-Harris type I fracture of the humerus D) Septic arthritis E) Subluxation of the radial head

E. Dislocation or subluxation of the radial head is a common pediatric injury, known as a nursemaid elbow. It results from traction on the radial head, which displaces it from within its ligamentous annulus. This most often occurs in children of age 12-36 months, and often arises from defensive maneuvers by parents or caretakers when grabbing the child's arm to prevent running into traffic or similarly endangering oneself. It can also arise from routine play if a parent, older sibling, or caregiver is swinging the child by his or her arms. Sometimes a pop may be heard, or clunk may be felt, usually followed by immediate crying. Children are generally consolable following the injury, and they often present with the arm held in a neutral, extended, pronated position as this is generally the position of best comfort. They will generally grimace or cry with palpation of the radial head. Diagnosis is clinical, and radiographs are seldom necessary; even if performed, they may not show misalignment as this is a ligamentous injury and pediatric ossification of the elbow occurs between ages 2-12 years making the structures poorly visible on radiograph. Treatment involves reducing the radial head back into the annulus through physical manipulation by hyper-pronation, supination-flexion maneuvers, or both. Seldom is operative reduction necessary, nor is sedation to achieve appropriate reduction. A successful reduction is determined by the child reusing his or her arm. Recurrent injuries are common. Incorrect Answers: A, B, C, and D. Dislocation of the shoulder (Choice A) would be uncommon following a traction maneuver on the arm. Shoulder dislocation may result from abduction, extension, and rotation, or as a consequence of trauma or muscle spasm (such as in seizure or electrocution). Examination shows a flattened depression infe

147 Exam Section 3: Item 47 of 50 National Board of Medical Examiners' Comprehensive Basic Science Self-Assessment 47. A 27-year-old man comes to the physician because of a 3-hour history of nausea, cramping abdominal pain, and diarrhea. His symptoms developed 24 hours after he ate at a wedding reception; several other people who ate at the reception have similar symptoms. A culture of the stool grows Salmonella enterica. He is able to maintain oral hydration. His symptoms begin to resolve within 36 hours but persist in a milder form for several more days. Which of the following is most likely to occur if this patient is treated with antibiotics? A) Anaphylaxis as a result of antibiotic hypersensitivity B) Decreased risk for endocarditis C) Decreased risk for hemolytic uremic syndrome D) Establishment of a chronic carrier state in the spleen E) Prolonged fecal excretion of the organism

E. Ingestion of food contaminated with Salmonella enterica can lead to Salmonella gastroenteritis. Salmonella enteritis classically presents with mucosal inflammation and invasion, leading to nausea, vomiting, cramping, and diarrhea. Associated symptoms can include fever and bloody stools if the infection is severe or invasive. Symptoms are typically self-limited, with resolution beginning within 48-72 hours. Uncommonly, individuals with Salmonella gastroenteritis can develop bacteremia and complications such as endocarditis, mycotic aneurysm, abscesses, and osteomyelitis, though these complications are atypical. Most patients only require symptomatic management and supportive care with hydration. Salmonella enteritis can be treated with antibiotics when indicated by severe illness (eg, prolonged fever, severe diarrhea, dehydration, or need for hospitalization), or in immunocompromised individuals, young infants, or elderly individuals at higher risk for complications. After resolution of the illness, asymptomatic shedding of Salmonella in the stool can continue for a short time, and further antibiotic treatment is not indicated. Antibiotic therapy has been associated with a longer duration of asymptomatic bacterial shedding through prolonged fecal excretion of the organism. Incorrect Answers: A, B, C, and D. Anaphylaxis as a result of antibiotic hypersensitivity (Choice A) is rare and would be unlikely with treatment with antibiotics in this patient unless he had a known allergy or unknown previous sensitization. Decreased risk for endocarditis (Choice B) may occur with prevention of Salmonella bacteremia, and antibiotics can decrease this risk, however, the development of Salmonella bacteremia is rare. Fecal excretion of Salmonella is a common and known sequelae of Salmonella gastroenteritis. Hemolytic uremic syndrom

153 Exam Section 4: Item 3 of 50 National Board of Medical Examiners' Comprehensive Basic Science Self-Assessment 3. A full-term female newborn is born with a symmetrically enlarged neck. The swelling is composed of a mass of endothelial cell-lined, cystically dilated spaces filled with lymph. The intervening stroma is scanty, and there are occasional lymphoid aggregates. Which of the following disorders is most likely in this newborn? A) Amniotic bands B) Cystic fibrosis C) Cystic teratoma D) Down syndrome E) Gonadal dysgenesis 45,X (Turner syndrome) F) Nephroblastoma (Wilms tumor)

E. Gonadal dysgenesis 45,X (Turner syndrome) presents with short stature, cystic hygroma or webbed neck, swelling in the hands/feet, coarctation of the aorta, bicuspid aortic valve, fused kidney, and infertility. It is a common cause of primary amenorrhea, which is secondary to gonadal (ovarian) dysgenesis in the setting of a 45, XO genotype. Additionally, patients fail to develop secondary sexual characteristics because of decreased estrogen concentrations. Vital signs will be normal, and diagnosis is made through a karyotype analysis. Other laboratory studies that may aid in diagnosis are increased concentrations of luteinizing hormone (LH) and follicle-stimulating hormone (FSH) because of the lack of negative feedback from estrogen, but these findings are nonspecific. Histologic evaluation of a cystic hygroma will show a lymphangioma or cystically dilated spaces filled with lymph fluid caused by obstruction of the lymphatic system. Šince it arises from the lymph vessels, it is lined with endothelium and does not contain a significant amount of connective tissue. Incorrect Answers: A, B, C, D, and F. Amniotic bands (Choice A) are bands of mesoderm that extend from the chorion to or across the embryo, with consequent constriction and/or vascular disruption. This constriction can result in congenital anatomic anomalies, such as amputation, craniofacial abnormalities, and neural defects. Amniotic bands are not associated with a cystic hygroma. Cystic fibrosis (Choice B) presents in a newborn with meconium ileus or the failure to pass stool within the first 48 hours of life, as well as the development of recurrent respiratory infections, pancreatic insufficiency, and failure to thrive. It is not commonly associated with congenital anatomic abnormalities. Cystic teratomas (Choice C) are tumors of the ovary that are deriv

139 Exam Section 3: Item 39 of 50 National Board of Medical Examiners' Comprehensive Basic Science Self-Assessment 39. A 55-year-old man with type 1 diabetes mellitus comes to the physician because of intermittent burning pain of his feet during the past 4 months. Examination of the feet shows allodynia bilaterally. Sensation to pinprick is decreased. Motor strength, deep tendon reflexes, joint position, and vibration sense are normal. Which of the following is the most likely cause of the pain in this patient? A) Increased activation of glutamate receptors in the dorsal root ganglia B) Increased activity of presynaptic y-aminobutyric acid receptors in the dorsal horn C) Increased activity of voltage-gated K+channels in the thalamus D) Inhibition of vanilloid receptors in the dorsal root afferents E) Persistent activation of voltage-gated Na+ channels in the nociceptor

E. In the setting of poorly controlled diabetes mellitus, chronic hyperglycemia results in the formation of advanced glycation end products in vascular endothelium, leading to capillary damage throughout the body, including within the vasa nervorum. Ischemic damage to peripheral nerves leads to diabetic neuropathy, which may cause neuropathic pain. A variety of changes that mediate nociception occur within ischemic nerves, including the upregulation and persistent activation of voltage-gated Na+ channels in the nociceptor, which lead to increased production of action potentials that cause the sensation of pain. Many pharmacologic agents that are effective for treating neuropathic pain, such as tricyclic antidepressants and carbamazepine, demonstrate sodium channel blocking properties. Incorrect Answers: A, B, C, and D. Increased activation of glutamate receptors in the dorsal root ganglia (Choice A) may underlie the pathogenesis of neuropathic pain in some types of nerve injuries. However, this mechanism does not underlie neuropathic pain that results from peripheral nerve microvascular damage. Increased activity of presynaptic y-aminobutyric acid receptors in the dorsal horn (Choice B) would result in the attenuation of somatosensory input. y-aminobutyric acid (GABA) is an inhibitory neurotransmitter. Increased activity of voltage-gated K+channels in the thalamus (Choice C) would result in increased central processing of nociceptive stimuli. However, this would not result from peripheral nerve microvascular damage. Inhibition of vanilloid receptors in the dorsal root afferents (Choice D) would result in reduced nociception. Vanilloid receptors are activated by noxious stimuli, such as capsaicin, and play a role in inflammatory pain. Educational Objective: Upregulation and persistent activation of voltage-gated Na chan

190 Exam Section 4: Item 40 of 50 National Board of Medical Examiners' Comprehensive Basic Science Self-Assessment 40. Restriction of valine in the diet is most likely to be effective in treatment of patients with which of the following? A) Cystinosis B) Cystinuria C) Galactosemia D) Homocystinuria E) Maple syrup urine disease F) Orotic aciduria

E. Maple syrup urine disease (MSUD) is treated by restriction of dietary valine, isoleucine, and leucine. MSUD is caused by a deficiency in branched-chain a-keto acid dehydrogenase. Branched-chain amino acids include isoleucine, leucine, and valine, and are important in physiologic processes such as gluconeogenesis and cholesterol synthesis. To break down branched-chain amino acids, they must first be converted to a-ketoacids by branched-chain aminotransferases (BCAT) with specific enzymes for each amino acid. This is followed by decarboxylation by branched-chain a-keto acid dehydrogenase complex (BCKDC). These amino acids are converted to acetoacetate, acetyl-CoA, and succinyl-CoA, respectively. MSUD occurs as a result of deficiency in BCKDC and leads to increased circulating concentrations of these branched-chain amino acids and their keto-acids. The transport of neutral amino acids into the central nervous system is required for synthesis of neurotransmitters, and increased concentrations of leucine affect this process. Clinically, the disease is characterized by severe neurologic defects, intellectual disability, vomiting, and poor feeding. The urine characteristically has an odor similar to maple syrup, which is from increased concentrations of isoleucine. Incorrect Answers: A, B, C, D, and F. Cystinosis (Choice A) is a lysosomal storage disease that results in increased concentrations of cystine and its accumulation in tissues, which can lead to severe organ dysfunction and failure. It is characterized by the impaired reabsorption in the proximal tubules of the kidney (Fanconi syndrome), leading to losses of sodium, potassium, glucose, amino acids, phosphate, and bicarbonate. Valine restriction will not affect the disease course. Cystinuria (Choice B) occur as a result of impaired cystine resorption in the proxim

116 Exam Section 3: Item 16 of 50 National Board of Medical Examiners' Comprehensive Basic Science Self-Assessment 16. A female newborn is delivered at 26 weeks' gestation and develops grunting and chest retractions shortly after birth. She is intubated and mechanically ventilated. A chest x-ray 8 hours later shows bilateral pulmonary infiltrates. The newborn dies the following day, and an autopsy is done. Which of the following findings is most likely on microscopic examination of the lungs? A) Cytomegalovirus inclusions and vascular necrosis B) Dense lymphocytic infiltrates and interstitial edema C) Platelet and fibrin thrombi D) Prominent type II pneumocyte hyperplasia and fibrosis E) Proteinaceous alveolar debris and atelectasis

E. Neonatal respiratory distress syndrome (NRDS) commonly occurs in premature infants and is caused by an inadequate production of pulmonary surfactant. Surfactant produced by type II pneumocytes functions to decrease surface tension of the alveolar membrane. Without sufficient quantities of surfactant, high surface tension leads to alveolar collapse. A fetus' risk for NRDS may be estimated by amniocentesis and calculation of the ratio of lecithin to sphingomyelin in the amniotic fluid. Surfactant is rich in phospholipids, such as lecithin and sphingomyelin, and an increase in the lecithin to sphingomyelin ratio correlates with increasing fetal lung development. Surfactant reaches normal levels of production in the third trimester, and infants who are born prior to 34 weeks are at an increased risk for NŘDS. NRDS presents with tachypnea, hypoxia, accessory muscle use, and cyanosis, and may lead to death. Chest x-ray commonly shows diffuse, bilateral airspace and interstitial opacities. Biopsy is not required for diagnosis, but if performed at autopsy demonstrates atelectasis with deposition of proteinaceous alveolar debris and hyaline material. The treatment of NRDS is supportive. NRDS may also be both prevented and treated with glucocorticoids, which stimulate surfactant production. Incorrect Answers: A, B, C, and D. Cytomegalovirus inclusions and vascular necrosis (Choice A) may occur in infants with congenital CMV infection. Infants with congenital CMV typically demonstrate characteristic features, including microcephaly, hepatosplenomegaly, and rash. Dense lymphocytic infiltrates and interstitial edema (Choice B) may occur in the setting of viral pneumonia. This is a less likely cause of the respiratory failure in this premature infant. Platelet and fibrin thrombi (Choice C) deposition may occur in disseminated in

101 Exam Section 3: Item 1 of 50 National Board of Medical Examiners' Comprehensive Basic Science Self-Assessment 100 1. A pharmaceutical company develops a new opioid drug (Drug X) that binds to the same receptor as morphine does. Drug X is compared with morphine to test its analgesic effect. The graph shows the log dose- response curves for morphine and Drug X. Based on these results, which of the following best describes Drug X? Morphine A) Competitive antagonist B) Full agonist C) Inverse agonist Drug X D) Noncompetitive antagonist E) Partial agonist ++ -8 10 -7 10 Concentration

E. Partial agonists are drugs that bind to and activate a receptor at the same site as a full agonist but have decreased efficacy relative to the full agonist. In this case, Drug X is a partial agonist that binds to the same opioid receptors as morphine (mu, delta, and kappa), and exhibits a similar effect. However, its efficacy is reduced, and thus it never reaches the same percentage of maximum response as morphine. This is the mechanism of action of buprenorphine at mu-opioid receptors, which can be used to treat opioid use disorder or chronic pain because of its long half-life and decreased risk for withdrawal. Buprenorphine is occasionally dosed in combination with naloxone, which is not absorbed orally but is absorbed if the medication is injected intravenously. This causes the opioid effect to be negated if the medication is not taken as prescribed. Incorrect Answers: A, B, C, andD. A competitive antagonist (Choice A) directly competes with an agonist for a receptor binding site, blocking its action. The competitive antagonist can be overcome by increasing the concentration of agonist to achieve full efficacy. A competitive antagonist would not typically result in a similar response as the direct agonist, as can be seen in the graph; rather, it affects the response of the direct agonist (in this case, morphine) when co-administered. A full agonist (Choice B) leads to the same maximal response as an agonist at the receptor site. It would be expected to reach the same maximum response as morphine, rather than be less efficacious. An inverse agonist (Choice C) binds to the same receptor site as a full agonist, but with the opposite effect of the full agonist. This is in contrast to a competitive agonist, which inhibits the efficacy of the response but does not cause the reverse effect. An inverse agonist would be e

149 Exam Section 3: Item 49 of 50 National Board of Medical Examiners' Comprehensive Basic Science Self-Assessment 49. A healthy 30-year-old woman participates in a study of cardiovascular function as it relates to posture. Placed in a room with a comfortable temperature (72°F) and humidity (40%), she moves from a standing to a supine position. Which of the following sets of changes is most likely to be observed in this woman's cardiovascular status? Stroke Volume Left Ventricular End-Diastolic Volume Pulse OA) ↑ ↑ ↑ B) ↑ C) ↑ ↑ D) ↑ E) ↑ ↑ F) ↑ G) ↑ H)

E. Postural change from standing to supine results in an influx of 500 to 1000 mL of blood from the lower body to the upper body caused by gravity. The sudden increase in venous return results in increased preload (left ventricular end-diastolic volume), which generates an increased stroke volume. The redistribution of blood volume also results in increased arterial pressure as sensed by baroreceptors in the aortic arch and carotid sinus, and a reflexive decrease in pulse results. In contrast, rising from supine to standing results in decreased venous return, decreased stimulation of carotid sinus baroreceptor activity, and decreased cerebral blood flow. The body normally responds to these changes with increased mean arterial and diastolic pressure, and increased stroke volume, pulse, and cardiac output to maintain perfusion to the brain and organs of the upper body. Incorrect Answers: A, B, C, D, F, G, and H. Choices A, B, C, and D are incorrect as the pulse decreases with a change from standing to supine rather than increasing. The pulse rate increases with the reverse action of standing from supine in order to maintain cardiac output. Choices C, D, G and H are incorrect as stroke volume increases with the increase in venous return as a result of increased contractility via Frank-Starling mechanics. Choices B, D, F, and H are incorrect as the left ventricular end-diastolic volume, or preload, is increased as a significant volume of blood in the lower extremities is returned to the heart when changing from standing to supine. Educational Objective: Changing position from standing to supine results in a large influx of venous return from the lower extremities to the heart with a subsequent increase in preload. This results in greater stroke volume via Frank-Starling mechanics. There is also a reflexive decrease in puls

112 Exam Section 3: Item 12 of 50 National Board of Medical Examiners' Comprehensive Basic Science Self-Assessment 12. A 42-year-old woman comes to the physician 2 weeks after she found a nodule in the left side of her neck. She is otherwise asymptomatic. Physical examination shows no other abnormalities. She undergoes a thyroid scan with 1231, a nondestructive isotope. After 1 week, the largest amount of circulating 123 would most likely be in which of the following? A) Free thyroxine (FT) B) Free triiodothyronine (FT3) C) Triiodothyronine (T3) bound to thyroxine (T)-binding globulin D) T3 bound to transthyretin E) T, bound to T binding globulin F) T, bound to transthyretin

E. Thyroid hormone is produced in the follicular cells of the thyroid gland under the regulation of thyroid stimulating hormone. Tyrosine molecules are assembled into protein-containing chains known as thyroglobulin. Concurrently, iodine is transported into the follicular cells and is linked to the tyrosine residues in thyroglobulin by thyroid peroxidase, forming thyroxine (T) or triiodothyronine (T). The T4 and T3 molecules are then cleaved from thyroglobulin. After administration of radioactive iodine, the iodine atoms will be incorporated into thyroid hormone. In circulation, the vast majority of thyroid hormone exists as T, bound to Tbinding globulin, rather than T3 Incorrect Answers: A, B, C, D, and F. Free thyroxine (FT) (Choice A) and free triiodothyronine (FT) (Choice B) collectively represent less than 0.5% of circulating thyroid hormone and will not contain a large amount of circulating radioactive iodine. Triiodothyronine (T2) bound to thyroxine (T)-binding globulin (Choice C) is significantly less abundant than T4 bound to Tbinding globulin. T3 or T4 bound to transthyretin (Choices D and F) is significantly less abundant than the amount bound to thyroid binding globulins, which are the predominant proteins involved in thyroid hormone transport in circulation. Educational Objective: The majority of thyroid hormone exists at T bound to Tbinding globulin. Radioactive iodine is incorporated into thyroid hormone in thyroid follicular cells before being released into the circulation. 4 %3D Previous Next Score Report Lab Values Calculator Help Pause

103 Exam Section 3: Item 3 of 50 National Board of Medical Examiners® Comprehensive Basic Science Self-Assessment 3. A 43-year-old man comes to the physician because of progressive shortness of breath during the past year. He is a stockbroker and has had no environmental exposure to toxins. When asked about his smoking habits, he says he smoked marijuana in college and afterwards, until the age of 28 years. He has not smoked any substance since then. He was adopted as a child and does not know his family history. His respirations are 29/min. Diffuse wheezing is heard on auscultation of the chest. Abdominal examination shows mild hepatomegaly. Pulmonary function tests show a decreased FEV, and decreased diffusing capacity. A chest x-ray shows hyperinflation consistent with air trapping mostly in the lung bases. A CT scan of the chest is shown. Which of the following is the most likely cause of these findings? O A) As

E. The most likely cause of the findings in this patient is a-antitrypsin deficiency, an inherited protease inhibitor deficiency. Please note that on subsequent iterations of reviewing this test item, we recognize that choice E should read 'Inherited protease inhibitor deficiency,' and that, as written, the correct answer could be unclear. a-Antitrypsin normally regulates neutrophil elastase, which is a serine protease secreted by neutrophils and macrophages as part of a localized inflammatory response. a-Antitrypsin deficiency is an inherited disorder caused by mutations in the SERPINA1 gene, which encodes a,-antitrypsin. Reduced concentrations of effective protein result in host tissue damage from unopposed neutrophil elastase, especially in the lungs. Pulmonary manifestations classically include panlobular emphysema (especially in the lung bases). The diagnosis should be suspected in a young patient presenting with evidence of chronic lung disease with bibasilar emphysematous changes. Patients are also at risk for chronic liver inflammation and cirrhosis as a result of the accumulation of misfolded a-antitrypsin polymers in hepatocytes. Incorrect Answers: A, B, C, and D. Asthma (Choice A) is a reversible obstruction to airflow that typically presents with shortness of breath, tachycardia, tachypnea, hypoxia, poor air movement, wheezing, and accessory muscle use. Chest imaging generally shows no abnormalities. Continuous marijuana use (Choice B) and a history of smoking marijuana sprayed with a toxic herbicide (Choice D) are unlikely to be the cause of this patient's chronic respiratory disease. Hepatic involvement, bibasilar emphysema, and an obstructive pattern on pulmonary function testing are more suggestive of a-antitrypsin deficiency. Diffuse idiopathic interstitial fibrosis (Choice C) is characterized by archi

132 Exam Section 3: Item 32 of 50 National Board of Medical Examiners' Comprehensive Basic Science Self-Assessment 32. A 30-year-old man comes to the physician because of a 2-month history of constant numbness of his right hand. He says he has had intermittent numbness of his right hand for the past 2 years. He says that sometimes the numbness awakens him from sleep. He is a right-handed carpenter. Occasionally at work he needs to rest for a few minutes when the numbness increases. Neurologic examination shows mild atrophy of the right thenar eminence. There is mild weakness of the right abductor pollicis brevis muscle. Sensation to touch is most likely to be decreased or abnormal in which of the following areas? A) Dorsal surface of the lateral hand B) Lateral hand and forearm C) Medial hand D) Medial hand and forearm E) Palmar surface of the lateral hand

E. The palmar surface of the lateral hand (the median nerve dermatome) is most likely to be affected in carpal tunnel syndrome (CTS), which results from compression of the median nerve within the carpal tunnel of the wrist. The carpal tunnel is located anterior to the carpal bones and is bound by the flexor retinaculum. The flexor tendons of the fingers course through the tunnel, along with the median nerve. The space is rigid and does not generally permit compression, thus, any inflammatory, traumatic, or space-filling lesion may result in CTS. Patients generally present with pain and paresthesia in the median nerve distribution, along with weakness involving the muscles of the thenar eminence (as in this patient's atrophic abductor pollicis brevis). Symptoms are generally worse at night. Repetitive trauma (as seen in this carpenter) is a known risk factor for CTS, because of chronic trauma and inflammation within the carpal tunnel leading to nerve compression. ČTS is frequently associated with pregnancy, hypothyroidism, rheumatoid arthritis, acromegaly, amyloidosis, ganglion cysts, obesity, and acute trauma to the wrist with impingement, or chronic remodeling and scar formation following a wrist injury. Management includes treating the underlying cause, splinting of the wrist, and decompressive surgery to relieve pressure and prevent irreversible median nerve palsy. Incorrect Answers: A, B, C, and D. Dorsal surface of the lateral hand (Choice A) sensation is transmitted via the radial nerve, which courses in the dorsal forearm and is not compressed or affected in carpal tunnel syndrome. The radial nerve also innervates the extensors of the wrist and fingers. Lateral hand and forearm (Choice B) sensation is transmitted by the radial, median, and musculocutaneous nerves. A lesion affecting this large of a territory wo

84 Exam Section 2: Item 34 of 50 National Board of Medical Examiners® Comprehensive Basic Science Self-Assessment 34. During a study, a healthy 35-year-old woman and a 35-year-old woman with Crohn disease are given an oral solution of radiolabeled lactulose. The appearance of radioactivity in their urine is monitored over the next 24 hours. Urinary excretion of radioactivity in the patient with Crohn disease is much greater than that of the healthy subject. This finding is most likely due to inflammation-associated changes in the integrity of which of the following epithelial structures? A) Adherens junctions (zonulae adherentes) B) Basement membrane C) Desmosomes (maculae adherentes) D) Gap junctions E) Tight junctions (zonulae occludentes)

E. Tight junctions (zonulae occludentes) are a type of epithelial cell connection composed of proteins called claudins and occludins. Tight junctions are located between two cells, toward the apical surface, and create a watertight seal to prevent the paracellular movement of solutes or water through an epithelial or endothelial lining. Inflammation of an epithelial lining causes it to become more permeable to substances via impairment of tight junctions. As seen in this patient with Crohn disease, inflammation of the wall of the small intestine impairs tight junctions and allows substances which would not normally be absorbed by the enterocytes to be transmitted paracellularly and then diffuse into the interstitial blood vessels. Another example of this same phenomenon is seen in an exudative pleural effusion. Inflammation from pneumonia, for example, impairs tight junctions and allows for solutes to move paracellularly from the intravascular space or alveoli into the pleural space. The lack of tight junctions explains the movement of large molecules which could not normally diffuse across cells. Incorrect Answers: A, B, C, and D. Adherens junctions (zonulae adherentes) (Choice A) are epithelial cell junctions which connect the actin cytoskeletons of adjacent cells to each other. A defect in adherens junctions, such as when E-cadherin expression is lost in breast cancer, allows for cells to metastasize. Basement membrane (Choice B) is a barrier which separates an epithelial lining from the interstitial tissue surrounding it. For example, the basement membrane separates the epidermis from the dermis in the skin. The basement membrane is made of type IV collagen, laminin, heparan sulfate, and fibronectin among other molecules. Type IV collagen is defective in Alport syndrome, causing dysfunction of the glomerular baseme

183 Exam Section 4: Item 33 of 50 National Board of Medical Examiners' Comprehensive Basic Science Self-Assessment 33. A 60-year-old man comes to the physician for an examination prior to employment. Physical examination shows no abnormalities. Laboratory studies show: Hemoglobin Hematocrit 14 g/dL 42% Leukocyte count Segmented neutrophils Small lymphocytes Monocytes Platelet count 12,000/mm3 45% 50% 5% 250,000/mm3 Flow cytometry analysis of peripheral lymphocytes shows the following phenotype: CD3 CD4 CD8 CD20 Surface kappa Surface lambda 50% 40% 10% 50% 47% 3% Which of the following is most predictive of a clonal lymphoid proliferation in this patient? A) Absolute CD3+ T-lymphocyte count B) Absolute CD20+ B-lymphocyte count C) Absolute lymphocyte count O D) CD4:CD8 ratio E) Surface kappa:surface lambda ratio

E. The surface kappa:lambda ratio showing high expression of kappa (47%) and low expression of lambda (3%) is the feature of this flow cytometry suggestive of a clonal lymphoid proliferation. CD20 is found on the surface of all B lymphocytes. Additionally, CD3, CD4, and CD8 are found on T lymphocytes regardless of clonality so these findings are not specific. All mature B lymphocytes express a unique immunoglobulin molecule that contains both a light chain and a heavy chain with constant and variable regions. The variable regions are unique to a specific B lymphocyte while the constant regions are conserved among an immunoglobulin class. Light chains may either be kappa or lambda, and the distribution in normal lymphocytes is typically about equal. The presence of peripheral Iymphocytes expressing a preponderance of one of these light chains (kappa in this case) suggests the expansion of a clonal population of lymphocytes, all of which are expressing the same immunoglobulin light chain. This is potentially consistent with chronic lymphocytic leukemia (CLL). Other common findings in CLL include a peripheral smear with numerous lymphocytes with large blue nuclei and scant cytoplasm, many of which may appear ruptured, and diffuse lymphadenopathy. Treatment with ibrutinib is initiated if patients have profound cytopenia or disabling symptoms such as malaise and bulky lymphadenopathy. Incorrect Answers: A, B, C, and D. Absolute CD3+ T-lymphocyte count (Choice A) in this patient may be slightly below the normal range, but this is a nonspecific finding. All T lymphocytes express CD3+, and a reduced percentage can be seen in a variety of conditions including both malignancy and infections such as HIV. Absolute CD20+ B-lymphocyte count (Choice B) in this patient is above the normal range and is likely consistent with CD20 expre

146 Exam Section 3: Item 46 of 50 National Board of Medical Examiners' Comprehensive Basic Science Self-Assessment 46. A 53-year-old woman is brought to the emergency department because of double vision for 8 hours. Her temperature is 36.1°C (97°F), pulse is 88/min, and blood pressure is 218/116 mm Hg. She has smoked 3 packs of cigarettes daily for 30 years. Neurologic examination shows a right-sided ptosis and inability to move the right eye medially or vertically. The right pupil is dilated and nonreactive to light. An MRI of the brain stem shows a lesion of the right cerebral peduncle. Which of the following additional physical findings is most likely in this patient? A) Complete paralysis of the face on the left side B) Complete paralysis of the face on the right side C) An involuntary movement disorder D) Left-sided ataxia E) Left-sided hemiparesis F) Right-sided hemiparesis

E. This patient who experienced a cerebrovascular accident (CVA, or stroke) of the right cerebral peduncle will most likely demonstrate left-sided hemiparesis on physical examination. The cerebral peduncle refers to the anterior portion of the midbrain and includes the crus cerebri, which contains the corticospinal tract and is supplied by paramedian branches of the posterior cerebral artery. Upper motor neurons of the corticospinal tract originate in the primary motor cortex, descend ipsilaterally through the internal capsule and midbrain (within the crus cerebri), decussate in the caudal medulla, and then descend contralaterally in the spinal cord to synapse with the contralateral lower motor neuron. A brainstem lesion of the corticospinal tract leads to contralateral weakness in an upper motor neuron pattern of dysfunction (eg, spastic paralysis, Babinski sign, and hyperreflexia). The oculomotor nerve (cranial nerve III) is located anteriorly between the two cerebral peduncles and may also be affected by strokes of the paramedian branches of the posterior cerebral artery, leading to ipsilateral dysfunction of the extraocular muscles and consequent diplopia along with ptosis and a deficit in pupillary constriction. The combination of ipsilateral cranial nerve IIl palsy and contralateral hemiparesis is called Weber syndrome. CVAS occur because of ischemic or hemorrhagic loss of blood supply to the brain. Approximately 80-85% of CVAS are ischemic, commonly arising from thromboembolic disease (eg, middle cerebral artery occlusion from a thrombus), whereas 15-20% of CVAS are hemorrhagic and caused by blood vessel rupture (eg, hypertension-related intraparenchymal hemorrhage from a perforating artery). Risk factors for CVAS include smoking, hypertension, diabetes mellitus, carotid or intracranial atherosclerotic disease,

89 Exam Section 2: Item 39 of 50 National Board of Medical Examiners® Comprehensive Basic Science Self-Assessment 39. A 21-year-old woman who is a college student comes to the physician because of a severe left-sided headache, nausea, and vomiting for 1 hour. Her symptoms began with flashing lights in both eyes while she was studying for final exams. Physical examination shows no abnormalities. The most appropriate treatment is a drug that acts on a receptor for which of the following endogenous mediators? A) B-Endorphin B) Histamine C) Neuropeptide Y D) Nitric oxide E) Serotonin

E. This patient with a migraine headache may be treated with triptan medications, which are serotonin agonists. Migraine is a relatively common condition that presents with recurrent primary headaches. Migraine headaches are typically severe, unilateral, throbbing, and associated with nausea, photophobia, and phonophobia. Some patients additionally experience an aura before or during the headache, which may present with vision loss followed by the perception of scintillating shapes or lines and/or paresthesias. Migraine triggers include stress, menstruation, weather changes, fasting, and alcohol. Migraines are postulated to arise from deficient serotonin inhibition of descending pain pathways. Acute migraine treatment includes triptans (serotonin agonists), dopamine antagonists, antiemetics, acetaminophen, and/or nonsteroidal anti-inflammatory drugs (NSAIDS). For patients with disabling or frequent migraines, prophylactic treatment is also recommended and includes B-adrenergic blockers, antidepressants (such as tricyclic antidepressants and serotonin-norepinephrine reuptake inhibitors), and anticonvulsants. Incorrect Answers: A, B, C, and D B-Endorphin (Choice A) is an endogenous opioid that is released during intense exercise and stress. Opioid medications mimic endogenous B-endorphin but are not recommended for migraines because of the risk for opioid overuse and the potentially resultant worsening of headaches. Histamine (Choice B) release causes vasodilation and increased vascular permeability and contributes to environmental allergies. Antihistaminergic medications (eg, diphenhydramine, cetirizine) are utilized to address allergies and are used off-label to treat anxiety and insomnia. Antihistaminergic medication does not necessarily play a role in acute migraine treatment, however the anticholinergic and sedating

174 Exam Section 4: Item 24 of 50 National Board of Medical Examiners' Comprehensive Basic Science Self-Assessment 24. A 40-year-old woman comes to the physician because of a 3-month history of continuous watery diarrhea that has gradually increased in severity; she has had an 11-kg (24-lb) weight loss during this period. Physical examination shows skin tenting and sunken eyes, and orthostatic hypotension. Tests of the stool are negative for ova, parasites, and Clostridium difficile. A CT scan of the abdomen shows a mass in the pancreas. Treatment with standard antidiarrheal medications is ineffective. Administration of a somatostatin analogue, octreotide, results in improvement of the diarrhea. This patient most likely has a tumor secreting which of the following agents? A) Cholecystokinin B) Gastrin C) Histamine D) Motilin E) Vasoactive intestinal polypeptide

E. VIPoma is a rare tumor often associated with multiple endocrine neoplasia that produces vasoactive intestinal peptide (VIP). Through its physiologic mechanism of action, VIP causes chronic and profound watery diarrhea, electrolyte disturbances (eg, hypokalemia, hypercalcemia), achlorhydria, alkalosis, flushing, and vasodilation. This syndrome is often termed WDHA syndrome (watery diarrhea, hypokalemia, achlorhydria). Diagnostic evaluation includes CT scan or MRI of the abdomen, somatostatin receptor nuclear scintigraphy, or PET scan to assist in localization of the tumor, and biopsy for tissue evaluation. Treatment includes correction of the associated fluid, electrolyte, and acid-base derangements, along with surgery and/or chemotherapy. Octreotide, a somatostatin analogue, directly antagonizes the effects of VIP and is useful for the improvement of symptoms. Incorrect Answers: A, B, C, and D. Cholecystokinin (Choice A) is an enteroendocrine hormone that is important in the regulation of biliary and pancreatic secretions and in delaying gastric emptying. It is not commonly secreted by neuroendocrine tumors. Gastrin (Choice B) is secreted by gastrinomas and results in the excessive production of acid by the gastric parietal cells. This can lead to recurrent, chronic duodenal or jejunal ulcers, which can present with abdominal pain, diarrhea, malabsorption, and possible hematemesis, melena, or hematochezia. Histamine (Choice C) can be secreted by carcinoid tumors, which may also secrete serotonin. Symptoms of metastatic carcinoid tumor include weight loss, diarrhea, bronchospasm, right-sided cardiac valvular disease, and abdominal pain. Cutaneous flushing is also a typical finding. Motilin (Choice D) is an enteroendocrine hormone that is important in promoting intestinal motility. It is not commonly secreted by neuro

125 Exam Section 3: Item 25 of 50 National Board of Medical Examiners® Comprehensive Basic Science Self-Assessment 25. An 18-year-old man comes to the physician 12 hours after the sudden onset of fever, chills, intense muscle aches, and headache. He has recently been on a hunting trip in Arkansas. His temperature is 39.4°C (103°F), pulse is 110/min, respirations are 18/min, and blood pressure is 110/80 mm Hg. Physical examination shows diaphoresis but no rash. A diagnosis of ehrlichiosis is made. The most appropriate treatment for this patient is an antibiotic with which of the following mechanisms of action? A) Binding to folic acid B) Binding to glycoprotein C) Binding to hydrogen ion D) Inhibition of glucose synthesis E) Inhibition of peptidoglycan synthesis F) Inhibition of protein synthesis G) Inhibition of water uptake

F. Inhibition of protein synthesis describes the mechanism by which doxycycline works. Ehrlichia chaffeensis is a tick-borne, intracellular bacterium that is endemic in the southeast, mid-Atlantic, and central United States. Patients often present with fever, headache, myalgias, and rarely with a rash. Laboratory studies commonly show thrombocytopenia, leukopenia, and increased transaminase concentrations. Doxycycline is the treatment of choice and is usually given for a period of 7 to 14 days. Doxycycline is a tetracycline antibiotic that binds to the 30S subunit of the bacterial ribosome and prevents translation of bacterial proteins. It is a well-tolerated medication with adverse effects that include nausea and photosensitivity. Incorrect Answers: A, B, C, D, E, and G. Binding of folic acid (Choice A) is not the correct answer. Medications such as trimethoprim bind to the dihydrofolate reductase enzyme and prevent conversion of folic acid to dihydrofolate and tetrahydrofolate to halt bacterial growth. Binding to glycoprotein (Choice B) is a feature of vancomycin, which prevents bacterial cell wall synthesis by binding to D-alanyl-D-alanine in the bacterial cell wall, thereby blocking polymerization of glycopeptides. Binding to hydrogen ions (Choice C) could theoretically increase the intracellular pH of bacteria and contribute to cell death but this is not the mechanism by which doxycycline works. Additionally, inhibition of glucose synthesis (Choice D) and of water uptake (Choice G) is not the mechanism of doxycycline. Inhibition of peptidoglycan synthesis (Choice E) is a feature of B-lactam antibiotics, which bind to penicillin binding proteins to inhibit the final step of peptidoglycan synthesis. Bacterial cells have peptidoglycan layers that encapsulate their inner membrane, although Gram-positive bacterium have

115 Exam Section 3: Item 15 of 50 National Board of Medical Examiners' Comprehensive Basic Science Self-Assessment 15. A 60-year-old man comes to the physician because of a 6-month history of nonproductive cough, increased skin pigmentation, and swollen feet; he has had a 10-kg (22-lb) weight loss during this period. Physical examination shows hyperpigmentation, muscle wasting, and pedal edema. Laboratory studies show a decreased serum potassium concentration, increased serum concentrations of adrenocorticotropic hormone and cortisol, and metabolic alkalosis. Administration of high-dose dexamethasone has no effect on the serum cortisol concentration. Which of the following is the most likely cause of the findings in this patient? A) Adrenal carcinoma B) Carcinoid tumor C) Multiple myeloma D) Pituitary adenoma E) Prostate carcinoma F) Small cell lung carcinoma

F. Small cell lung carcinoma is a neoplasm of neuroendocrine cells and is associated with numerous paraneoplastic syndromes, including Cushing syndrome, syndrome of inappropriate antidiuretic hormone, Lambert-Eaton myasthenic syndrome caused by presynaptic calcium channel antibody production, paraneoplastic limbic encephalitis, and subacute cerebellar degeneration. Histologic features include small, dark blue tumor cells lacking nucleoli with a high nuclear to cytoplasm ratio. This patient has developed paraneoplastic Cushing syndrome, marked by proximal muscle wasting, hyperpigmentation, hypertension, edema, increased blood glucose, and hypokalemia caused by an increased secretion of ectopic adrenocorticotropic hormone (ACTH) and leading to chronic adrenal hyperstimulation and unregulated overproduction of cortisol. In small cell lung cancer, the ACTH is typically released from the malignancy itself, with no appreciable response to a dexamethasone suppression test (no change in serum cortisol concentration with dexamethasone administration). By contrast, ACTH secreted by the pituitary may still respond to negative feedback, and therefore concentrations may fall after the administration of high-dose dexamethasone. Incorrect Answers: A, B, C, D, and E. Adrenal carcinoma (Choice A) is a rare and aggressive malignancy that may cause Cushing syndrome secondary to excess cortisol production by the tumor. ACTH-independent cortisol-producing adrenal carcinomas are less common in adults than small cell lung carcinoma. Results of laboratory studies would show a decreased ACTH concentration. Carcinoid tumor (Choice B) is a neuroendocrine cell neoplasm that most commonly occurs in the gastrointestinal tract. It produces high concentrations of serotonin, and if metastatic, may result in carcinoid syndrome characterized by weight l

171 Exam Section 4: Item 21 of 50 National Board of Medical Examiners' Comprehensive Basic Science Self-Assessment 21. A 25-year-old woman comes to the physician because of a 7-year history of 5- to 10-minute episodes of increased heart rate, sweating, trembling, dizziness, derealization, and intense fear that occur almost daily. She feels well between attacks except for apprehension about when the next attack will occur. Physical examination and laboratory studies show no abnormalities. Which of the following is the most likely diagnosis? A) Alcohol withdrawal B) Diencephalic epilepsy C) Generalized anxiety disorder D) Hyperthyroidism E) Intermittent atrial fibrillation F) Panic disorder G) Pheochromocytoma

F. This patient is likely experiencing recurrent panic attacks consistent with panic disorder. Panic attacks feature acute fear or anxiety that peaks within minutes and is associated with four additional physical symptoms or associated mental states. These additional symptoms may include heart palpitations or tachycardia, shortness of breath, chest pain, dizziness, the sensation of choking, gastrointestinal distress, paresthesias, sweating, chills, trembling, derealization, the fear of dying, and/or the fear of losing control. Panic disorder is a relatively common psychiatric disorder characterized by recurrent panic attacks that are unexpected and associated with worry about future panic attacks or avoidance of panic attack triggers. Treatment typically includes psychotherapy, long-term selective serotonin reuptake inhibitors, and short-term benzodiazepines. Incorrect Answers: A, B, C, D, E, and G. Alcohol withdrawal (Choice A) typically involves hours to days of symptoms of sympathetic hyperactivity such as anxiety, tremors, nausea, diaphoresis, hypertension, and tachycardia and, in severe cases, may also result in hallucinations, seizures, and delirium tremens. Daily episodes of minutes of sympathetic hyperactivity with apprehension about future episodes is more consistent with panic disorder. Diencephalic epilepsy (Choice B) is a rare condition resulting from closed head injuries, tumors, or corpus callosum agenesis that involves episodic sympathetic hyperactivity. Intense fear would be atypical, and panic disorder is much more prevalent than diencephalic epilepsy. Generalized anxiety disorder (Choice C) features excessive and persistent worrying that may be associated with sympathetic hyperactivity. Panic attacks may occur in generalized anxiety disorder, though anxiety symptoms are typically more constant and per

31 Exam Section 1: Item 31 of 50 National Board of Medical Examiners® Comprehensive Basic Science Self-Assessment 31. A 50-year-old man is brought to the emergency department because of a 2-hour history of vomiting blood. He is in acute distress. His temperature is 37.5°C (99.5°F), pulse is 110/min, respirations are 18/min, and blood pressure is 80/50 mm Hg. Despite appropriate treatment, he dies shortly thereafter, and an autopsy is done. The gross and microscopic appearances of a section of the esophagus are shown. Which of the following best explains these autopsy findings? A) Angiosarcoma B) Candidiasis C) Cytomegalovirus infection SPECIMEN DATE D) Esophageal carcinoma E) Gastric metaplasia OF) Hemangioma G) Miliary tuberculosis H) Portal hypertension

H. The microscopic findings taken from the esophageal sample demonstrate dilation of the submucosal venous plexus. This correlates with the dilated vessels seen on the gross specimen. The inferior esophageal veins drain into the left gastric vein and then into the portal vein. They are part of the portal venous system, a series of interconnected veins that drain blood from the colon, small intestines, spleen, liver, stomach, and inferior esophagus. While this blood eventually makes its way to the systemic circulation via the inferior vena cava, it must first pass through the liver. In cirrhosis, obliteration of the hepatic sinusoids through progressive fibrosis increases the resistance to blood flow through the liver, which is transmitted to the portal venous system causing portal hypertension (PH). Increased venous pressure is transmitted to all the veins which drain into the portal vein, including the inferior esophageal veins, resulting in esophageal varices after they are engorged. In addition to esophageal varices, manifestations of PH include gastric varices, caput medusae, hemorrhoids, splenomegaly, and ascites. The risk for acute gastrointestinal bleeding from esophageal varices is high. It presents with large volume hematemesis followed by hemodynamic collapse if untreated. Incorrect Answers: A, B, C, D, E, F, and G. Angiosarcoma (Choice A) and hemangioma (Choice F) are both vascular neoplasms caused by proliferation of endothelial cells. Angiosarcoma is a malignant vascular neoplasm which is characterized by large, atypical endothelial cells and vascular slit-like spaces. In contrast, hemangioma is a proliferation of benign endothelial cells creating duplicated vascular channels. This histopathology does not demonstrate cellular atypia and the number of vascular channels is normal, only they are dilated. Esop

162 Exam Section 4: Item 12 of 50 National Board of Medical Examiners' Comprehensive Basic Science Self-Assessment 12. A 56-year-old woman comes to the emergency department because of a 1-month history of a painful, enlarging mass on the right side of her jaw. She has not noticed any drainage from the area. She has had no fever or other systemic symptoms. She appears thin. Her vital signs are within normal limits. Physical examination shows a tender, 3-cm, firm mass on the right mandible with mild overlying erythema and warmth but no exudate. Oral examination shows multiple dental caries. There is shotty anterior right cervical lymphadenopathy. A CT scan of the head shows a heterogeneous, 3-cm, soft-tissue mass with associated bony changes indicative of osteomyelitis. A photomicrograph of a specimen obtained via needle aspiration of the mass is shown; aerobic cultures grow no organisms. Which of the following is the

A. Actinomyces israelii is a gram-positive, branching, anaerobic rod that classically causes oral and facial abscesses which may progress to osteomyelitis of the jaw, often in patients with poor dentition and multiple caries. A. israelii can be identified by its branching, filamentous pattern on gram stain. It resembles fungi but is a bacterium and is a component of oral flora. Lesions caused by A. israelii often ulcerate or drain to the skin via sinus tracts, which grossly appear to have yellow granules resembling the element sulfur. Lesions have also been described involving the female genital tract, however facial lesions are the classic presentation. Biopsy demonstrates the branching, gram-positive, filamentous rods, as shown in this photomicrograph. Histologically, A. israelii closely resembles Nocardia asteroides. The two organisms must be differentiated, as the treatment differs. Treatment of A. israelii is with penicillin, whereas sulfonamides are first line for N. asteroides. Incorrect Answers: B, C, D, and E. Fusobacterium necrophorum (Choice B) is a gram-negative, anaerobic rod. It is a part of normal gastrointestinal flora. In pathology, it causes pharyngitis, peritonsillar abscess, and Lemierre syndrome: a septic thrombophlebitis of the internal jugular vein, often in a healthy, young patient. Nocardia asteroides (Choice C) is a gram-positive, branching, aerobic rod. Like A. israelii, it also resembles fungi on histology. However, N. asteroides is not a normal component of oral flora and more commonly causes pneumonia in immunocompromised persons. In immunocompetent patients, it generally causes mild cutaneous abscesses or localized cellulitis. Propionibacterium acnes (Choice D) is a commensal, gram-positive, anaerobic rod that classically causes the infectious comedonal condition referred to as acne. It i

123 Exam Section 3: Item 23 of 50 National Board of Medical Examiners' Comprehensive Basic Science Self-Assessment 23. A 23-year-old man comes to the physician because of pain in his left leg for 3 days. He has a 9-year history of alcoholism. He has a court hearing scheduled the next day for driving while under the influence and another court hearing scheduled the following week for "fighting in a bar." He blames "overzealous police" for his legal problems and says that most people are stupid and don't see things his way, which is the right way. Physical examination shows no abnormalities of the left lower extremity. This patient most likely has which of the following personality disorders? A) Antisocial B) Avoidant C) Histrionic D) Paranoid E) Schizoid

A. Antisocial personality disorder is a cluster B personality disorder (the emotional or dramatic cluster) that arises from a deficit in empathy, resulting in pervasive violations of others' rights, aggression, and a hostile and manipulative attitude toward others. Similar to other cluster B personality disorders, patients with antisocial personality disorder struggle to regulate their emotions because of lower frustration tolerance, leading to frequent impulsive behavior. These patients may also be chronically bored and sensation seeking, leading to a high rate of substance abuse and gambling. These personality traits lead to deficits in personal (extreme ego-centrism) and interpersonal (empathy, intimacy) functioning. Personality disorders are persistently maladaptive ways of relating to the self and to society that typically appear by early adulthood. Incorrect Answers: B, C, D, and E. Avoidant personality disorder (Choice B) is a cluster C personality disorder (the anxious or fearful cluster) that features the avoidance of interpersonal contact caused by feelings of social inadequacy and hypersensitivity to rejection. Alternatively, patients with antisocial personality disorder are apathetic about the opinions of other people. Histrionic personality disorder (Choice C), a cluster B personality disorder, is characterized by theatrical, superficial expressions of emotion that unconsciously serve to garner attention from others to fulfill emotional needs. These patients may dress in a seductive way for the same purpose. Patients with antisocial personality disorder try to violate, not seduce, other people. Paranoid personality disorder (Choice D) is a cluster A personality disorder (the odd or eccentric cluster) characterized by a pervasive mistrust of others and viewing others as possessing malicious intent. These pa

196 Exam Section 4: Item 46 of 50 National Board of Medical Examiners' Comprehensive Basic Science Self-Assessment 46. A 65-year-old man receives a liver transplant for primary biliary cirrhosis. To prevent organ rejection, immunosuppressive therapy that inhibits T-lymphocyte activation with cyclosporine is planned. The activity of which of the following molecules involved in T- lymphocyte activation is most likely to be inhibited by this treatment? A) Calcineurin B) Protein kinase C C) STAT4 D) STAT6 E) T-bet

A. Calcineurin inhibitors, such as tacrolimus and cyclosporine, impair T-lymphocyte activation and are commonly employed in patients who have undergone solid organ transplants to prevent the development of acute and chronic cellular allograft rejection. More intensive regimens given early after transplant are gradually tapered to a maintenance dose and are given indefinitely. Cyclosporine binds to cyclophilin proteins in the cytoplasm of T lymphocytes. Calcineurin is a phosphatase that is required for the transcription and expression of interleukin-2 (IL-2), IL-3, IL-4, interferon-y, and tumor necrosis factor-a (TNF-a). The cyclosporine-cyclophilin complex binds to calcineurin and prevents it from upregulating transcription of these genes. These cytokines, specifically IL-2, are necessary for T-lymphocyte proliferation and action, so inhibition attenuates the T-lymphocyte response. Of the various types of circulating T lymphocytes, calcineurin inhibitors preferentially inhibit helper T lymphocytes. Incorrect Answers: B, C, D, and E. Protein kinase C (Choice B) is a ubiquitous enzyme involved in numerous signaling cascades throughout the body. Inhibitors of this enzyme include ruboxistaurin, which is currently being investigated for use in diabetic peripheral neuropathy and retinopathy. STAT4 (Choice C) is a transcription factor that upregulates expression of IL-2 and contributes to activation of T helper 1 (Th1) T-lymphocytes. There are not currently any medications in clinical use outside of trials that inhibit this transcription factor. STAT6 (Choice D) is a transcription factor involved in IL-4 signaling and Th, lymphocyte function. Calcineurin inhibitors do not affect this signaling. T-bet (Choice E) is a transcription factor expressed in CD4+ T lymphocytes that will ultimately become Th, cells. It also has a role

63 Exam Section 2: Item 13 of 50 National Board of Medical Examiners® Comprehensive Basic Science Self-Assessment 13. A 32-year-old man begins to laugh while eating dinner with his friends. A small particle of food irritates his larynx and provokes him to cough. Which of the following best describes the position of his vocal cords throughout this sequence? ITTT Immediately After Laryngeal Irritation While Swallowing While Coughing A) Closed closed open B) Closed оpen closed C) Closed оpen open D) Оpen closed closed E) Оpen closed оpen OF) Оpen open closed

A. Coughing can be a voluntary or reflexive action that protects the airway from aspiration, foreign bodies, and environmental irritants, and clears bronchial secretions and debris. The forceful expulsion of air requires coordinated mechanics between the respiratory muscles and laryngeal structures. While swallowing, the vocal cords are closed to prevent inadvertent introduction of food into the airway and lungs. Immediately after laryngeal irritation, the vocal cords and epiglottis remain closed while the respiratory muscles contract to generate increased pressure in the airways. While coughing, the vocal cords open to allow forceful exhalation of air to remove the irritant. Respiratory muscle weakness and neuromuscular disorders that interfere with the coordinated mechanics of cough are risk factors for aspiration and associated pneumonia. Incorrect Answers: B, C, D, E, and F. Choices D, E, and F are incorrect as the epiglottis is inverted and the vocal cords are closed during swallowing to protect the airway and lungs. Choices B, C, and F are incorrect as the vocal cords remain closed as the cough reflex arc is initiated by laryngeal irritation in order to generate pressure with the trapped air in the lungs. Choices B, D, and F are incorrect as the vocal cords must open during the cough to allow air to escape the lungs and remove the offending irritant. Educational Objective: Coughing is a protective mechanism for clearing material from the airways. To generate high positive pressure for expelling air from the lungs, the vocal cords close as the respiratory muscles contract, trapping air in the lungs. Opening of the vocal cords then allows a forceful expulsion of air to occur. %3D Previous Next Score Report Lab Values Calculator Help Pause

88 Exam Section 2: Item 38 of 50 National Board of Medical Examiners® Comprehensive Basic Science Self-Assessment 38. A 62-year-old man is brought to the physician by his wife because of increasing confusion during the past 6 months. His wife says that he has become lost twice in the past month when going to work, even though he has been going to the same office for 12 years. She adds that he often has difficulty finding objects such as his glasses and keys, sometimes cannot recall his grandchildren's names, and has become very critical of her cooking, which he used to enjoy. When asked, he can name only the current president and none of the candidates for an upcoming presidential election, although he and his wife watch the television news together each night. Neurologic examination shows no motor or sensory abnormalities. His Mini-Mental State Examination score is 19/30. Treatment with a cholinesterase inhibitor i

A. Dementia, Alzheimer type, results from decreased cholinergic signaling in the cerebral cortex and basal forebrain. Cholinergic neurons (which primarily originate from the locus coeruleus and basal forebrain) project widely to areas including the cortex, mediating attention, learning, and memory. Alzheimer dementia is the most prevalent type of dementia, presenting with progressive cognitive decline that begins with short-term memory impairment, progresses to apraxia and language abnormalities, and culminates in behavioral and personality changes preventing the patient from performing basic activities of daily living. Acetylcholinesterase normally hydrolyzes acetylcholine in synaptic clefts. Donepezil and other cholinesterase inhibitors non-competitively and reversibly inhibit acetylcholinesterase activity and thereby increase the amount of synaptic acetylcholine available for neurotransmission. Though these medications may slow the rate of cognitive decline and modestly improve functionality, they are not curative. Incorrect Answers: B, C, D, and E. The dentate nucleus and thalamus (Choice B) mediate output from the cerebellum. These structures are not typically affected in Alzheimer dementia. The fornix and neurons in the mammillary bodies (Choice C) constitute output pathways from the hippocampus and mediate episodic memory. Lesions may result in amnesia. Patients with Alzheimer dementia demonstrate hippocampal atrophy and decreased cholinergic signaling in the hippocampus, but the fornix and mammillary bodies are less commonly affected. The substantia nigra and the neurons of the globus pallidus (Choice D) together modulate voluntary movement. These brain regions are affected in Parkinson disease, not Alzheimer dementia. The thalamus and neurons in layer 4 of the cerebral cortex (Choice E) mediate the transmissio

167 Exam Section 4: Item 17 of 50 National Board of Medical Examiners' Comprehensive Basic Science Self-Assessment 17. Within the Ashkenazi Jewish population, the frequency of Tay-Sachs disease is approximately 1 in 3600. Assuming that this population is in Hardy-Weinberg equilibrium, which of the following is the expected frequency of heterozygous carriers within this population? A) 1/30 B) 1/60 C) 1/120 D) 1/1000 E) 1/3600

A. The Hardy-Weinberg principal, known as Hardy-Weinberg equilibrium, proposes that allele frequencies will remain constant across generations in the absence of evolutionary change. This model of population genetic stability requires a number of assumptions, including that natural selection does not occur, that there are no random mutations, that no movement in or out of the population occurs in equilibrium, and that mating is completely random. With these assumptions, the frequencies of alleles can be calculated as they will remain constant over time. The Hardy-Weinberg equilibrium equation is: p2 + 2pq + q2 = 1, where p is the frequency of the dominant allele and q is the frequency of the recessive allele. The sum of the allele frequencies p and q is also equal to 1 (p+ q = 1). 2pq is the probability of heterozygosity, p2 is the probability of dominant homozygote, and q2 is the probability of being a recessive homozygote. In the case of Tay-Sachs disease, the disease frequency of this autosomal recessive condition is 1 in 3600, which is the probability of being a recessive homozygote, q2. Therefore, solving for q, the recessive allele frequency, yields 1/60. By subtraction, p equals 59/60. The expected frequency of heterozygous carriers is 2pq, or, 2*(59/60)*(1/60), which is equal to approximately 1/30. Incorrect Answers: B, C, D, and E. 1/60 (Choice B), is the frequency of recessive alleles that cause Tay-Sachs disease. Since two alleles are required for expression of the phenotype, this number must be multiplied by the dominant allele frequency and factored by two based on Mendelian inheritance to arrive at the correct answer. 1/120 (Choice C) is half of the recessive allele frequency. 1/1000 (Choice D) reflects a mathematical error or assumption error. 1/3600 (Choice E) is the frequency of patients carrying two co

138 Exam Section 3: Item 38 of 50 National Board of Medical Examiners' Comprehensive Basic Science Self-Assessment 38. A healthy 21-year-old woman is participating in a study of an experimental drug. Various cardiac measurements before and after administration of the drug are shown: Before Administration 82/min After Administration 82/min Pulse Central venous pressure Mean arterial pressure Pulmonary capillary wedge pressure Aortic pressure 1 cm H,0 (N=5-8) 100 mm Hg mm Hg (N=5-16) 134/83 mm Hg 6 сm Н,0 90 mm Hg 12 mm Hg 110/80 mm Hg Assuming that no reflex adjustments have occurred, which of the following best describes the mechanism of action of this drug? A) Decreased cardiac contractility B) Decreased compliance of the aorta C) Increased total peripheral resistance D) Increased ventricular compliance E) Increased ventricular conduction

A. The cardiac measurements suggest the experimental drug most likely results in decreased cardiac contractility. There is no change in pulse. The increase in central venous pressure and pulmonary capillary wedge pressure and the decrease in mean arterial pressure suggest a reduced stroke volume and decreased forward flow of blood. The aortic systolic pressure is decreased with preservation of the diastolic pressure, which also argues for a reduced volume of blood ejected during systole without impairment of vascular tone. Incorrect Answers: B, C, D, and E. Decreased compliance of the aorta (Choice B) would result in an increased aortic systolic pressure after administration of the drug as the volume of blood entering the aorta remains the same. Compliance is equal to the change in volume divided by the change in pressure. Increased total peripheral resistance (Choice C) would result in increased aortic systolic and diastolic pressures and an increased mean arterial pressure as flow is equal to driving pressure divided by resistance. If flow remains equal, pressure increases as resistance increases. Increased ventricular compliance (Choice D) would result in decreased central venous pressure and pulmonary capillary wedge pressure after drug administration as it would take less driving pressure to fill the ventricles with the same volume of blood. Increased ventricular conduction (Choice E) would be expected to alter the pulse after administration of the experimental drug. Educational Objective: A reduction in cardiac contractility results in decreased cardiac output, mean arterial pressure, and aortic systolic pressure, along with an increase in central venous pressure and pulmonary capillary wedge pressure caused by the reduced forward flow of blood. %3D Previous Next Score Report Lab Values Calculator Help Pause

119 Exam Section 3: Item 19 of 50 National Board of Medical Examiners' Comprehensive Basic Science Self-Assessment 19. A 43-year-old man comes to the physician for a follow-up examination. Ten years ago, he underwent cardiac valve replacement. Chest x-rays are shown. Based on these findings, which of the following valves was most likely replaced in this patient? A) Aortic B) Mitral C) Pulmonic D) Tricuspid

A. The cardiac valves are contained within the fibrous skeleton of the heart, and their location may be identified on chest imaging if extensive calcification is present or an artificial valve has been placed. The aortic and mitral valves are the most commonly replaced valves. Cardiac valve location can be determined by analyzing a lateral plain film of the chest and drawing a line from the carina to the cardiac apex. The pulmonic and aortic valves are superior to this line while the tricuspid and mitral valves are inferior. Additional information can be gained by drawing a line from the left atrial appendage to the right cardiophrenic recess on the posterior-anterior film. The aortic valve lies superior to this line. Incorrect Answers: B, C, and D. The mitral (Choice B) valve is located between the left atrium and left ventricle. It lies posterior and inferior to the aortic valve. The pulmonic (Choice C) valve separates the right ventricle from the pulmonary trunk. It is located at the same vertical level as the aortic valve and to the right. The tricuspid (Choice D) valve is located between the right atrium and the right ventricle. It resides anteriorly and to the right on chest imaging. Educational Objective: The locations of the cardiac valves can be identified on chest imaging by drawing a diagonal line from the carina to the cardiac apex on a lateral film and from the left atrial appendage to the right cardiophrenic recess on the posterior- anterior film. The aortic valve lies above these two lines. %3D Previous Next Score Report Lab Values Calculator Help Pause

170 Exam Section 4: Item 20 of 50 National Board of Medical Examiners' Comprehensive Basic Science Self-Assessment 20. An experimental animal has a blood flow of 4 mL/sec in a small arterial segment of the mesenteric circulation. Which of the following best describes the blood flow (in mL/sec) if the diameter of the vessel is decreased by one half with no change in the perfusion pressure gradient? A) 0.25 B) 0.50 C) 1.0 D) 2.0 E) 16.0

A. The flow of blood through a vessel is equal to the driving pressure divided by the resistance to flow (flow = pressure/resistance). The driving pressure in the normal circulatory system is the pressure difference across the capillary beds (the perfusion pressure gradient), represented by mean arterial pressure (MAP) - right atrial pressure (RAP). Resistance to flow is a function of vessel diameter, length, and fluid viscosity if laminar flow through a cylindrical vessel is assumed. This is represented by the Poiseuille equation, resistance = (8 x cylinder length x fluid viscosity)/(TT x radius4). If the vessel length and blood viscosity remain constant, a change in diameter of the vessel by a factor of 2 results in an inverse change in resistance (and direct change in flow) by a factor of 16. In this case, a flow rate of 4 mL/sec through the mesenteric vessel will decrease to 0.25 mL/sec if the diameter of the vessel is decreased by half. Incorrect Answers: B, C, D, and E. 0.50 (Choice B) and 1.0 (Choice C) are incorrect as these answers imply calculating flow as a function of radius to the third and second power, respectively. 2.0 (Choice D) is incorrect as it does not take into account that flow is a function of radius to the fourth power. 16.0 (Choice E) is incorrect as decreasing the diameter of a vessel will greatly increase the resistance to flow. Flow rate would decrease, not increase. Educational Objective: The resistance to flow through a vessel is a function of the vessel length, radius, and fluid viscosity. A change in radius has the greatest effect on the resistance and flow rate. %3D Previous Next Score Report Lab Values Calculator Help Pause

198 Exam Section 4: Item 48 of 50 National Board of Medical Examiners' Comprehensive Basic Science Self-Assessment 48. A 1-week-old male newborn has the lesion shown. Which of the following is the most likely diagnosis? A) Acquired melanocytic nevus B) Café au lait spot C) Ecchymosis D) Hemangioma E) Malignant melanoma F) Slate gray nevus

B. A café au lait spot is characterized by a tan to dark brown, uniformly pigmented macule or patch often seen at birth or noticed in early childhood. It may be asymmetrical or have an irregular border. It represents a focal increase in melanin production rather than an increased number of melanocytes or any form of malignancy. While numerous café au lait spots suggest an underlying genetic disease, a single lesion is a normal finding. If numerous café au lait spots are present, neurofibromatosis type I (NF1) and McCune- Albright syndrome should be considered. The diagnostic criteria for neurofibromatosis type I include six or more café au lait spots which measure >5 mm before puberty or >15 mm after puberty. Other findings including neurofibromas, axillary freckling, optic glioma, iris hamartoma, bony dysplasia, and a family history of NF1. In contrast, McCune-Albright syndrome presents with precocious puberty, irregularly spaced café au lait spots with irregular borders, and bony dysplasia. In a newborn, a single café au lait spot is most likely benign and a solitary finding. Incorrect Answers: A, C, D, E, and F. Acquired melanocytic nevus (Choice A) is a benign proliferation of melanocytes. They are common but not typically present at birth. Acquired melanocytic nevi should not display asymmetry, border irregularity, or multiple colors. Ecchymosis (Choice C), or bruise, is caused by extravasation of red blood cells. The color of an ecchymosis comes from the extravasated hemoglobin rather than melanin, which is seen in a café au lait spot. Hemangioma (Choice D) is a benign proliferation of blood vessels which can either be present at birth or develop shortly after birth. They are bright red in appearance and do not have a pigmented component. Malignant melanoma (Choice E) clinically appears as a brown to black

102 Exam Section 3: Item 2 of 50 National Board of Medical Examiners' Comprehensive Basic Science Self-Assessment 2. A 32-year-old woman comes to the physician because of a 7-day history of sneezing, nasal stuffiness, and watery eyes. She has a history of similar symptoms each spring while gardening. Her temperature is 37°C (98.6°F). Which of the following types of cells are most likely to be increased in her nasal secretions as a result of this reaction? A) Basophils B) Eosinophils C) Lymphocytes D) Mast cells E) Monocytes

B. Allergic rhinitis is characterized by IgE mediated inflammation (type I hypersensitivity) of the nasal mucosa. Inciting allergens result in the cross-linking of IgE on the surface of basophils and mast cells, leading to their immediate degranulation and release of histamine, prostaglandins, and leukotrienes, which results in vasodilation, vascular pooling, and increased vascular permeability. This is followed by the recruitment of a variety of inflammatory cells, predominantly eosinophils, to the nasal mucosa, which can be found in nasal secretions. Common inciting allergens include plant pollens, animal dander, dust, and mold. Patients commonly experience nasal or sinus congestion, rhinorrhea, sneezing, nasal pruritus, conjunctivitis, and tearing. Patients with allergic rhinitis may display signs of atopic disease processes, such as asthma or eczema. Treatment includes inhaled nasal corticosteroids, inhaled leukotriene receptor antagonists, and oral antihistamines. Removal or avoidance of the offending allergen is essential to prevent further exacerbations and improve symptoms. Incorrect Answers: A, C, D, andE. Basophils (Choice A) and mast cells (Choice D) are important for the initiation of the allergic response. However, they are typically outnumbered by eosinophils in nasal secretions in allergic rhinitis. Lymphocytes (Choice C), especially T lymphocytes, play a role in the potentiation of the allergic response, but are not as numerous as eosinophils. Monocytes (Choice E) are important cells in phagocytosis and antigen presentation. They do not play a prominent role in the allergic response. Educational Objective: Allergic rhinitis is characterized by IgE mediated inflammation (type I hypersensitivity) of the nasal mucosa. Inciting allergens result in the cross-linking of IgE on the surface of basophils and mas

122 Exam Section 3: Item 22 of 50 National Board of Medical Examiners' Comprehensive Basic Science Self-Assessment 22. The effects of drug X alone, drug X and cholinesterase, and drug X and atropine on an isolated gastrointestinal muscle are shown in the bar chart. Drug X is most likely which of the following? A) Acetylcholine B) Bethanechol C) Nicotine D) Succinylcholine E) Trimethaphan Drug X Drug X and cholinesterase Drug X and atropine

B. Bethanechol is a direct cholinergic agonist that works at muscarinic receptors but not at nicotinic receptors. It can be used to treat urinary retention and gastric dysmotility. However, unlike acetylcholine which is also an agonist at muscarinic receptors, it is not metabolized by cholinesterase. Therefore, its effects are not affected by cholinesterase administration, as can be seen in this graph. In contrast, atropine is a direct competitive antagonist at the muscarinic receptor, which would prevent bethanechol from binding to its receptor site and exerting its effects when co-administered. Incorrect Answers: A, C, D, and E. Acetylcholine (Choice A) is a direct agonist at muscarinic and nicotinic receptors. It increases the actions of the parasympathetic nervous system and causes muscular contraction. However, it is metabolized by cholinesterase and, thus, its ability to promote gastrointestinal muscle contraction would be inhibited when co-administered with cholinesterase. Nicotine (Choice C) is a direct agonist at nicotinic receptors but not at muscarinic receptors, which are responsible for gastric smooth muscle contraction. Therefore, the effects of nicotine would be absent in this graph, regardless of whether or not it was administered with cholinesterase or atropine. Succinylcholine (Choice D) is a direct competitive agonist of nicotinic receptors in the neuromuscular junction. It is metabolized by pseudocholinesterase. However, it plays no role in the contraction of the smooth muscle of the gastrointestinal system. Trimethaphan (Choice E) is an anticholinergic medication that acts on ganglionic nicotinic receptors to inhibit output from the autonomic nervous system, including both sympathetic and parasympathetic components. It is rarely used clinically. Its administration would be expected to decrease gast

134 Exam Section 3: Item 34 of 50 National Board of Medical Examiners' Comprehensive Basic Science Self-Assessment 34. A 43-year-old woman undergoes ultrasonography for suspected cholecystitis. Ultrasonography shows that the right kidney is absent; the left kidney appears normal. The most likely cause of this anomaly is failure of which of the following tissues to respond to the developmental stimulus provided by the growth of the ureteric bud? O A) Mesonephros B) Metanephros C) Pronephros D) Somatic mesoderm E) Splanchnic mesoderm

B. During embryologic development, several primitive renal structures form, the pronephros, mesonephros, and metanephros. Only the metanephros contributes to the ultimate formation of the kidney, while the two more primitive structures undergo degeneration. The ureteric bud forms as an outpouching of the mesonephric duct and induces the development of mature renal tissue from the surrounding metanephros. Normal renal embryologic development is dependent on reciprocal inductive signaling between the ureteric bud and the metanephros. The ureteric bud reacts to signaling from the metanephros and undergoes branching, with the subsequent development of the collecting ducts, while the metanephros mesenchyme reacts to signals from the uretic bud to form the renal tubules. When these two tissues are separated in culture, they are incapable of developing completely in the absence of the other. This interaction occurs either via diffusible compounds, direct cell-to-cell contact, or through communication via gap junctions. Incorrect Answers: A, C, D, and E. The mesonephros (Choice A) is the second embryologic precursor of the kidney and forms primitive nephron-like structures. Like the pronephros, the mesonephros undergoes degeneration in utero and does not contribute to the mature kidney. The pronephros (Choice C) is the first embryologic precursor of the kidney and consists of several primitive duct-like structures called nephrotomes. The pronephros degenerates in utero and does not contribute to the mature kidney. The somatic mesoderm (Choice D) derives from the lateral plate mesoderm and contributes to the structures of the body wall. The splanchnic mesoderm (Choice E) is also a derivative of the lateral plate mesoderm and contributes to a variety of visceral organs, including the heart, vasculature, and gastrointestinal trac

83 Exam Section 2: Item 33 of 50 National Board of Medical Examiners® Comprehensive Basic Science Self-Assessment 33. A study is conducted to assess the effectiveness of a registry for identifying patients 50 years old and older eligible for colon cancer screening. Medical practices are selected based on equal and proportional representation of both genders and socioeconomic and ethnic groups, including those who have medical insurance and those who do not. Half of the practices will use the registry for a 15-year period and the other half will provide usual care without the registry during the same period. Results show that the colon cancer mortality rate is decreased in the practices using the registry, and the researchers recommend use of a patient registry to decrease the number of deaths due to colon cancer. Which of the following study characteristics most directly supports the researchers' recommendation? A)

B. External validity determines whether an experiment can be generalized and its conclusions applied to groups beyond those considered in the study population. In other words, will the conclusions hold for different populations at different times? In this example, a registry that identifies patients over age 50, but then broadly selects practices for inclusion that have representation of gender, socioeconomic and ethnic breadth, and wide financial and insurance status will likely capture a reasonable sample of the population as a whole presenting to medical practices. The study period was also quite long, 15 years, which is likely sufficiently long to capture outcomes related to colorectal carcinoma, a condition with a long latency period. Given that the study showed reduced mortality rate through the use of the registry, the researchers make the recommendation to use it broadly, a recommendation based on proximate similarity of practices that will use it. In other words, the study results should apply to many medical practices beyond those purely with the characteristics of those included. Incorrect Answers: A, C, D, and E. Accuracy (Choice A) describes the absence of error or bias and reflects how correct the test measurements are as compared to actuality. In other words, if a patient actually weighs 50 kg, and the test measures the patient's weight, will the test show a weight of 50 kg? If so, the test may be considered accurate. Face validity (Choice C) is a relatively weak form of a validity assessment that subjectively asks whether the test or experiment measures what it is supposed to measure. In other words, is the test suitable to its aims? This experiment appears to be valid at face value - it is designed to measure whether a registry will reduce the mortality from colorectal carcinoma and compares its use ov

14 Exam Section 1: Item 14 of 50 National Board of Medical Examiners® Comprehensive Basic Science Self-Assessment 14. A 27-year-old man with AIDS comes to the physician because of a 6-week history of progressive memory loss, imbalance, clumsiness, and difficulty finding words. The symptoms began with an inability to concentrate 3 months ago. He is not adherent to combination antiretroviral therapy and recently recovered from Pneumocystis jirovecii (formerly P. carinii) pneumonia. An MRI shows diffuse, poorly defined, bilateral hyperintensities within cerebral white matter. A stereotactic biopsy specimen shown in the photomicrograph depicts cells scattered predominantly along the perivascular spaces. Which of the following is the most likely diagnosis? A) Cryptosporidiosis B) HIV encephalopathy C) HIV protease inhibitor toxicity O D) Lymphoma E) P. jirovecii infection

B. HIV encephalopathy is the most likely diagnosis in this patient with AIDS, progressive memory loss, and motor deficits who does not adhere to antiretroviral therapy. HIV encephalopathy is a diagnosis of exclusion in patients with HIV, and other causes of encephalopathy such as toxoplasmosis, meningitis, encephalitis, bacterial abscess, progressive multifocal leukoencephalopathy (PML), or primary central nervous system (CNS) lymphoma should be ruled out. The classic triad of HIV encephalopathy involves movement disorders, psychomotor impairment, and memory deficits. While it can occasionally be confused with PML, patients with PML tend to have a more rapid disease progression, focal deficits, and different findings on MRI. In HIV encephalopathy, the MRI demonstrates multiple, symmetric, and poorly demarcated T2 hyperintense lesions scattered in the subcortical white matter. Brain biopsy characteristically demonstrates microglial nodules with multinucleated giant cells as in this case. Incorrect Answers: A, C, D, and E. Cryptosporidiosis (Choice A) is caused by Cryptosporidium parvum that presents with severe diarrhea in patients with AIDS. Symptoms include fever, weight loss, symptoms of dehydration and orthostasis, severe, watery diarrhea, cramping abdominal pain, nausea, and vomiting. Disseminated infection can involve the lungs and liver but encephalopathy or encephalitis does not occur. HIV protease inhibitor toxicity (Choice C) can present acutely with nausea and vomiting, but long-term complications commonly include lipodystrophy and increased cardiovascular risk. They do not cause encephalopathy. Lymphoma (Choice D) in patients with AIDS can take on many forms, but primary CNS lymphoma is common. It often presents with seizures, lethargy, subacute memory loss, and headache. Physical examination may show neurol

28 Exam Section 1: Item 28 of 50 National Board of Medical Examiners® Comprehensive Basic Science Self-Assessment 28. A 23-year-old woman comes to the physician because of a 1-week history of intermittent episodes of fever and chills, followed by a rash. She has had four operations to correct scoliosis, in addition to undergoing a tonsillectomy, an appendectomy, removal of a lipoma from her scalp, and dermabrasion to correct acne scarring. She was admitted to the hospital 1 year ago for 2 weeks because of nausea and vomiting of unexplained origin. Her temperature is 37°C (98.6°F). Physical examination shows approximately 12 evenly spaced punctate marks in a linear pattern on the abdomen and upper and lower extremities. There are no marks on the face or back. Which of the following is the most likely diagnosis? A) Acute myelogenous leukemia B) Factitious disorder C) Immune thrombocytopenic purpura D) Lyme disease E

B. In factitious disorder, patients consciously produce symptoms (ie, purposely injure themselves) for primary gain. Primary gain is the motivation to be cared for, which constitutes an unconscious motivator for the patient's conscious production of symptoms. Early losses, neglect, or abuse may disrupt patient identity and relationships, and adopting the sick role may represent an attempt to cope with stress and stabilize their identity and relationships. This young patient's extensive history of elective surgeries and hospital admissions illuminate a pattern of care seeking, and linearly spaced punctate marks are consistent with self-inflicted puncture wounds rather than a known disease. Treatment includes regular follow-up with one physician who oversees management (as opposed to symptom-triggered appointments) and possible psychotherapy. Incorrect Answers: A, C, D, and E. Acute myelogenous leukemia (Choice A) is a hematologic cancer resulting from the malignant transformation of myeloid precursor cells. Proliferated myeloid precursor cells may deposit in soft tissues and present as nodules or plaques. Punctate lesions would be atypical. Immune thrombocytopenic purpura (Choice C) is an acquired thrombocytopenia resulting from autoantibodies against platelet antigens. Patients may present with a petechial or purpuric rash in dependent body regions (eg, feet, legs, and hands) rather than punctate marks on the abdomen and legs. Lyme disease (Choice D) is a tick-borne illness that may initially present with erythema migrans, a lesion that forms around the site of the tick bite. The lesion may develop central clearing. Multiple punctate lesions would be inconsistent with Lyme disease. Patients with syphilis (Choice E), a sexually transmitted spirochete infection, may demonstrate a diffuse macular or papular rash that invo

62 Exam Section 2: Item 12 of 50 National Board of Medical Examiners® Comprehensive Basic Science Self-Assessment 12. A 25-year-old man is brought to the emergency department because of a 2-hour history of nausea, vomiting, severe abdominal pain, and shortness of breath. His respirations are 32/min. Abdominal examination shows guarding and rigidity. An x-ray of the chest and upper abdomen with the patient seated upright is shown. Based on these findings, this patient most likely has which of the following? A) Diaphragmatic hernia B) Perforated duodenal ulcer C) Rectovesical fistula D) Tension pneumothorax E) Traumatic injury to the liver

B. Perforation of a duodenal ulcer commonly presents with severe, diffuse abdominal pain, nausea, vomiting, fever, tachycardia, and septic shock. Bowel contents in the peritoneal space cause rapid infection in the form of peritonitis, marked by abdominal rigidity, rebound tenderness, and guarding. An upright chest x-ray may show air under the diaphragm (pneumoperitoneum) caused by leakage of bowel contents into the peritoneal cavity, as in this patient. Perforated viscus (and resultant peritonitis) constitutes a surgical emergency requiring exploratory laparotomy to identify and repair any perforation, lavage the peritoneum and prevent or control sepsis, manage exsanguination if vessels are involved, and resect any necrotic bowel. Broad spectrum antibiotics are also indicated. Incorrect Answers: A, C, D, and E. Diaphragmatic hernia (Choice A) describes the presence of abdominal contents within the thorax, and can present with chest pain, shortness of breath, and bowel sounds within the lung fields. It most commonly occurs in the left hemithorax and may be visualized as intestinal loops on radiographs. Rectovesical fistula (Choice C) is a condition in which there is an abnormal connection between the rectum and the urinary bladder. This can lead to complications of urine leakage through the rectum and urinary tract infections plus pneumaturia. It would not cause pneumoperitoneum. Tension pneumothorax (Choice D) occurs when air accumulates within the pleural cavity to an extent that results in compression and shifting of the mediastinal structures. Such structural compromise results in impaired venous return leading to diminished cardiac output. It typically presents as severe respiratory distress, jugular venous distention, tracheal deviation, diminished breath sounds, hyperresonance to percussion, and hemithorax hyperi

181 Exam Section 4: Item 31 of 50 National Board of Medical Examiners' Comprehensive Basic Science Self-Assessment 31. A 29-year-old woman is brought to the emergency department 1 hour after being injured in a motor vehicle collision. Abdominal examination shows tenderness with rebound. Vital signs are stable. A CT scan of the abdomen shows no abnormalities except for a small amount of intraperitoneal fluid. After 6 hours of observation, an exploratory laparotomy is done because of continuing severe abdominal pain. No abnormalities are seen until the surgeon gently slides his hand behind the liver; there is a sudden massive hemorrhage from the region of the hand, and the patient's blood pressure becomes undetectable within 1 minute. Which of the following underlying lesions is most likely associated with the hemorrhage? A) Avulsion of celiac trunk from the aorta B) Avulsion of hepatic veins from the inferior vena cav

B. The source of occult and slow bleeding posterior to the liver in this patient is likely caused by an avulsion injury of the hepatic veins from the inferior vena cava (IVC). The hepatic veins function to drain venous blood from the liver into the inferior vena cava back to the heart. The hepatic veins and IVC lie posterior to the liver and blunt traumatic injury such as in a motor vehicle collision can cause avulsion of the hepatic veins from the IVC, leading to intraperitoneal hemorrhage posterior to the liver. The patient's probable supine positioning throughout her hospital course may have permitted auto-tamponade of the avulsion injury as the dependent effect of the weight of the liver may have slowed blood loss and exceeded venous pressure. On elevating the liver in the operating theater, traction would be placed on the avulsion injury, separating previously abutting vessels and permitting rapid intraperitoneal exsanguination. Management requires immediate pressure, packing, clamping, and vascular repair, plus emergent autotransfusion or administration of blood products. Large vessel injury may be immediately fatal because of the potential for loss of several liters of blood volume within seconds, leading to transfusion-refractory hypotension and hemorrhagic shock. Incorrect Answers: A, C, D, and E. Avulsion of the celiac trunk from the aorta (Choice A) would result in hemorrhage because of its central location and arterial-level pressure. The celiac trunk is a large artery that branches off the aorta and supplies the foregut of the intestinal tract. Rapid hemoperitoneum or retroperitoneal hemorrhage would be expected. Traumatic transection of the aorta (Choice C) can occur in penetrating trauma, or in deceleration injuries such as motor vehicle collisions or falls. However, it typically causes near-immediate fa

64 Exam Section 2: Item 14 of 50 National Board of Medical Examiners® Comprehensive Basic Science Self-Assessment 14. A 25-year-old man comes to the physician because of abdominal pain and vomiting for 1 day. He has had greasy yellow plaques on his skin and recurrent abdominal pain following ingestion of fatty meals since infancy. Physical examination shows xanthomas over the trunk and lipemia retinalis. There is tenderness over the epigastrium, and bowel sounds are decreased. Laboratory studies show leukocytosis and an increased serum amylase activity. The plasma appears milky. Which of the following is most likely increased in this patient's serum? A) Apo A-II B) Chylomicrons C) Free cholesterol D) HDL-cholesterol E) LDL-cholesterol

B. The patient is presenting with characteristic features of familial hyperchylomicronemia, a genetic lipid disorder caused by lipoprotein lipase deficiency. Lipoprotein lipase catalyzes the conversion of triglycerides carried by circulating chylomicrons into fatty acids in extrahepatic tissue. The fatty acids can then be absorbed by cells for use in metabolism. Lipoprotein lipase deficiency is marked by increased serum concentrations of chylomicrons and triglycerides. Patients typically present with abdominal pain following ingestion of fatty meals and steatosis. Physical examination may disclose eruptive xanthomas and lipemia retinalis (cream-colored discoloration of the retinal vessels associated with hyperchylomicronemia). Build-up of chylomicrons and triglycerides in the serum results in a milky appearance and increased amylase activity. Incorrect Answers: A, C, D, and E. Apo A-II (Choice A) is a protein that is found in HDL-cholesterol and is not involved in chylomicron transport and processing. Apo C-Il is a component of VLDL-cholesterol and chylomicrons, and it activates lipoprotein lipase. Deficiency of Apo C-Il can also result in familial hyperchylomicronemia. Free cholesterol (Choice C) concentrations are not typically increased in hyperchylomicronemia, and the condition is not associated with an increased risk for atherosclerosis, as with other lipid disorders. HDL-cholesterol (Choice D) is composed of the apolipoproteins ApoA-1, ApoC-II, and ApoE. HDL concentrations are not increased in hyperchylomicronemia. LDL-cholesterol (Choice E) is primarily composed of the apolipoprotein ApoB-100, which mediates binding to the LDL-receptor on hepatocytes. LDL concentrations are increased in familial hypercholesterolemia and combined hyperlipidemia. Educational Objective: A deficiency in lipoprotein lipase results in

47 Exam Section 1: Item 47 of 50 National Board of Medical Examiners® Comprehensive Basic Science Self-Assessment 47. A 12-year-old girl is brought to the emergency department 15 minutes after she accidentally sliced her left palm with a knife. Physical examination shows a 2-cm laceration over the left palm. The wound is cleansed and sutured. One week later, the sutures are removed. At this time, which of the following factors is most instrumental in the migration of fibroblasts into the area of wound healing? A) Collagen B) Fibronectin C) Нераrin D) Immunoglobulin O E) Plasminogen

B. Wound healing occurs via a staged process. In early wound healing, platelet aggregation and platelet plug formation occur to achieve hemostasis. In the subsequent 1 to 7 days, neutrophils and macrophages infiltrate the area and release growth factors and cytokines that stimulate fibroblast proliferation. Fibronectin is essential for fibroblast migration by providing a pathway for migration during wound healing. Fibroblasts bind to peptide sequences within fibronectin, which guide them to the site of healing. Granulation tissue forms as collagen is deposited into the area by fibroblasts and neovascularization begins to occur. During this time, wound edges contract from the action of myofibroblasts. Epidermal cells migrate across the newly deposited collagen matrix to reconstitute normal skin appearance. In the following weeks and months, scar formation and remodeling occur via metalloproteinase-mediated collagen breakdown. Incorrect Answers: A, C, D, and E. Collagen (Choice A) is synthesized by fibroblasts and contributes to scar formation in the process of wound healing. Dermal collagen fibers increase as a result of scarring. While the majority of collagen fibers in healthy skin are type I collagen, scarring is initially created by type III collagen. Heparin (Choice C) is a common anticoagulant that potentiates the action of antithrombin III to inhibit multiple coagulation factors. It does not relate to fibroblast migration in wound healing. Immunoglobulins (Choice D) are found on B lymphocyte membranes or are secreted into the serum by plasma cells to recognize antigens and activate the immune system in response to a pathogen. Plasminogen (Choice E) is converted to plasmin, which subsequently degrades fibrin clots leading to clot dissolution. Plasminogen is made in the liver and its role is to degrade and prevent

200 Exam Section 4: Item 50 of 50 National Board of Medical Examiners' Comprehensive Basic Science Self-Assessment 50. A 25-year-old woman comes to the physician because of palpitations and insomnia for 5 months; she also has had a 5-kg (11-lb) weight loss during this period. Her pulse is 125/min. Physical examination shows exophthalmos, thyromegaly, and hand tremors. Serum studies show a thyroid-stimulating hormone concentration of less than 0.1 pU/mL, thyroxine (T) concentration of 18 µg/dL, and triiodothyronine (T) concentration of 250 ng/dL. The signs and symptoms in this patient are most likely caused by hormones acting on which of the following receptors? A) Cytokine-linked kinase B) G protein-coupled receptor C) Nuclear/retinoid X binding to DNA D) Serine/threonine kinase E) Tyrosine kinase

C. Graves disease is the most common cause of hyperthyroidism. It is caused by an autoantibody that activates the thyroid stimulating hormone receptors (TSH-R) on thyroid follicular cells. Both T3 and T are hormones that act on nuclear receptors, requiring them to enter the target cell to exert effects. Unlike other lipophilic hormones, thyroid hormones contain charged amino acids that prevent passive diffusion across the cellular membrane and thus, must enter the cell by facilitated diffusion. Thyroid hormone transporters transport both T3 and T4 into the cell to reach their receptors. The thyroid hormone receptors are nuclear receptors that contain DNA-binding domains. Nuclear receptors can initially be located in either the cytosol or nucleus. Once nuclear receptors bind their respective hormones, they translocate into the nucleus, if not already there, where they act as DNA transcription factors to regulate the expression of target genes, often with the aid of retinoid X receptors. Excessive stimulation of TSH-R leads to follicular thyroid hyperplasia and a diffusely enlarged thyroid. Serum laboratory studies typically show increased concentrations of T3 and T4 decreased TSH, and the presence of TSH-R stimulating antibodies. Graves disease presents with symptoms of hyperthyroidism, including heat intolerance, weight loss, tremor, hyperreflexia, warm, moist skin, and pretibial myxedema. Some patients with Graves disease also develop thyroid ophthalmopathy, which can cause diplopia, proptosis, and restrictive strabismus. Incorrect Answers: A, B, D, and E. Cytokine-linked kinases (Choice A) are important for the downstream signaling of cytokine receptors. Many cytokine receptors rely on signaling via Janus kinase (JAK) tyrosine kinases and serine/threonine kinases. The thyroid hormone receptor does not have a role in

69 Exam Section 2: Item 19 of 50 National Board of Medical Examiners® Comprehensive Basic Science Self-Assessment 19. A 55-year-old homeless man comes to the emergency department because of a 2-month history of intermittent fever, cough productive of blood-tinged sputum, night sweats, and fatigue; he has had a 3.2-kg (7-lb) weight loss due to decreased appetite during this period. On arrival, he is cachectic and has a chronic cough. His temperature is 38°C (100.4°F). Rhonchi are heard over the right upper lobe.. A chest x-ray shows areas of cavitation in the right upper lobe. Which of the following is the most likely cause of tissue injury in this patient's condition? A) Antigen-antibody complex deposition B) Arthus reaction C) Delayed-type hypersensitivity D) Pseudomembrane formation E) Wheal and flare reaction

C. Delayed-type hypersensitivity reaction, also known as a type IV hypersensitivity reaction, most likely accounts for this patient's lung injury in the setting of pulmonary tuberculosis. Type IV hypersensitivity is characterized by a cell-mediated response which involves maturation of antigen specific CD4+ or CD8+ T lymphocytes, which in this case, are antigens specific to Mycobacterium tuberculosis (MTB). When the antigen is encountered, CD4+ T lymphocytes release cytokines leading to inflammation and macrophage activation, while CD8+ T lymphocytes directly kill cells expressing the antigen. Activated macrophages become histiocytes and form granulomas, which in the case of MTB, may be caseating granulomas. Local inflammation at the site of MTB infection is an attempt to encapsulate and/or destroy MTB, but the local immune response results in substantial damage to the surrounding lung parenchyma. Incorrect Answers: A, B, D, and E. Antigen-antibody complex deposition (Choice A), also called a Type lII hypersensitivity reaction, is a result of immune complex (antigen-antibody) deposition in tissues resulting in a local inflammatory response and complement fixation with subsequent damage to tissues. Common diseases in which this plays a role include polyarteritis nodosa, systemic lupus erythematosus, and serum sickness. Arthus reaction (Choice B) is a localized type III (antigen-antibody complex) hypersensitivity reaction whereby complexes that fix complement deposit in blood vessels and cause local necrosis. It occasionally occurs with hepatitis B or tetanus vaccines, although this is uncommon. Pseudomembrane formation (Choice D) occurs in infection with Clostridium difficile, a bacterium that causes colitis in patients with recent exposure to antibiotics. Colonoscopy classically shows pseudomembranes on the surface of

193 Exam Section 4: Item 43 of 50 National Board of Medical Examiners' Comprehensive Basic Science Self-Assessment 00 43. A previously healthy 71-year-old man comes to the emergency department because of a 12-hour history of fever and cough. Current medications include ramelteon nightly. His temperature is 39.6°C (103.2°F), pulse is 100/min, respirations are 20/min, and blood pressure is 100/60 mm Hg. Crackles are heard over the right lower lung with dullness to percussion. Laboratory studies show: Hemoglobin Hematocrit 14 g/dL 42% Leukocyte count Platelet count 52,000/mm3 460,000/mm3 A chest x-ray shows a patchy infiltrate in the right lower lobe. A Gram stain of sputum shows numerous segmented neutrophils containing gram-positive cocci. A photomicrograph of a Wright-stained peripheral blood smear is shown. Which of the following is the most likely explanation for these findings? A) Essential thrombocythemia B) In

C. Leukemoid reaction in the setting of probable Streptococcus pneumoniae lobar pneumonia and sepsis is the most likely explanation of the abnormal peripheral blood smear. A leukemoid reaction is a response to stress, infarction, or infection that results in a significant rise in peripheral circulating leukocytes. In states of excessive physiologic stress, the bone marrow releases a large number of neutrophil precursors into the circulation including myelocytes and metamyelocytes, with more severe states of stress leading to release of progressively more immature cells. It can sometimes be confused with acute leukemia but is differentiated by the fact that in a leukemoid reaction, leukocytes are mature, functional, and polyclonal as opposed to immature and monoclonal. If confusion persists, flow cytometry and bone marrow biopsy can be helpful diagnostic tools to differentiate the two entities, as can measuring leukocyte alkaline phosphatase. Incorrect Answers: A, B, D, and E. Essential thrombocythemia (Choice A) is a myeloproliferative neoplasm characterized frequently by mutations in genes such as JAK2, MPL, and CALR that can cause unregulated production of platelets in the bone marrow. Patients often present with a platelet count exceeding 500,000/mm3. This patient's thrombocytosis is likely a reaction to his acute infection and is unlikely to represent an underlying myeloproliferative disorder. Infectious mononucleosis (Choice B) is commonly caused by Epstein-Barr virus or cytomegalovirus and presents with fever, lymphadenopathy (typically involving the posterior cervical lymph nodes), hepatosplenomegaly, and pharyngitis. A peripheral blood smear would demonstrate atypical lymphocytes, not the immature neutrophils seen in the presented case. Myelodysplasia (Choice D) is frequently encountered in older individuals or

104 Exam Section 3: Item 4 of 50 National Board of Medical Examiners® Comprehensive Basic Science Self-Assessment 4. A 54-year-old man comes to the physician because of a 3-day history of low back pain. His temperature is 38.3°C (101°F), pulse is 75/min, and blood pressure is 120/80 mm Hg. Physical examination shows right-sided flank tenderness to percussion. A CT scan of the abdomen shows a lesion consistent with an abscess along the posterior medial aspect of the right kidney. Extension of this process into this patient's pelvis is most likely to occur along which of the following surfaces? A) Internal oblique B) Latissimus dorsi C) Psoas fascia D) Quadratus lumborum fascia E) Transverse abdominal

C. Perinephric abscesses can occur as a complication of pyelonephritis, especially in the setting of abnormalities such as renal stone or vesicoureteral reflux. Less commonly, they can occur via hematogenous spread in the setting of bacteremia. Offending microbes in perinephric abscess are typically Gram-negative bacilli, though may be polymicrobial. In the setting of hematogenous spread, Staphylococcus aureus is the most common cause. If untreated, abscesses can spread and affect adjacent structures such as the flank muscles or the psoas muscle. If located inferiorly, they can cause inflammation and infection in the pelvis. Superiorly, they can lead to a subphrenic abscess. This patient's abscess is located on the posterior-medial aspect of the kidney, which is adjacent to the psoas muscle. Śmall abscesses may be treated with antibiotic therapy alone, but larger abscesses may need percutaneous or operative drainage. Incorrect Answers: A, B, D, and E. The internal oblique muscle (Choice A) is a component of the anterior abdominal wall. It runs superomedially from the thoracolumbar fascia and the iliac crest to the inferior ribs and the linea alba. Since the kidney is retroperitoneal, it would be unlikely to affect the anterior abdominal wall muscles. The latissimus dorsi (Choice B) is a large muscle that originates at thoracolumbar spinous processes and fascia, iliac crest, and the inferior angle of the scapula and inserts on the humerus. It is a superficial muscle and is not adjacent to the kidneys. The quadratus lumborum fascia (Choice D) originates at the posterior border of the iliac crest and inserts on the inferior border of the 12th rib and the lumbar spine. It is lateral to the psoas muscle. A renal abscess that is on the posterior medial aspect of the kidney would more likely affect the psoas fascia. The tran

17 Exam Section 1: Item 17 of 50 National Board of Medical Examiners® Comprehensive Basic Science Self-Assessment 17. The patient whose cardiac function is illustrated most likely has which of the following? Normal A) Arteriovenous malformation B) Cardiac tamponade C) Congestive heart failure Patient D) Cor pulmonale E) Restrictive cardiomyopathy Left ventricular end-diastolic volume

C. The Frank-Starling mechanism describes the phenomena by which cardiac output is dependent on the amount of cardiomyocyte fiber stretch prior to contraction, as represented by the left ventricular end-diastolic volume. A greater pre-contraction stretch results in a greater force of contraction (to a point), and the relationship is demonstrated by Frank-Starling curves. A given Frank-Starling curve applies for constant afterload and inotropy. Changes in afterload and/or inotropy shift the curve up or down. This patient has a Frank-Starling curve that is shifted down, indicating that for a given preload, there is reduced cardiac output relative to normal. This may occur in decreased inotropic states such as congestive heart failure, with the administration of negative inotropes, or in the setting of increased afterload. The curve shifts up in positive inotropic states and/or with decreased afterload. Incorrect Answers: A, B, D, and E. Arteriovenous malformation (Choice A) results in low-resistance, high-volume flow of blood from the arterial to the venous system with greatly increased venous return. The increase in preload causes a greater distension in the cardiomyocyte fibers at the end of diastole, which results in increased cardiac output per the Frank-Starling relationship. Cardiac tamponade (Choice B) result in decreased ventricular filling because of compression of the heart by fluid in the pericardium. In the absence of other factors affecting afterload or cardiac contractility, the Frank-Starling curve would not be depressed. Cor pulmonale (Choice D) describes right ventricular failure resulting from chronic pulmonary hypertension. Left ventricular contractility and afterload are not affected, and the Frank-Starling curve for the left ventricle would not shift. Restrictive cardiomyopathy (Choice E) results in

130 Exam Section 3: Item 30 of 50 National Board of Medical Examiners' Comprehensive Basic Science Self-Assessment 30. A paper says, "We chose the sample size to have an 80% power of detecting a 15% mean difference with a significance level (two-sided) of 5%." If there really is no difference between the groups overall, which of the following best represents the chance that the study will find a statistically significant difference, and what is this error called? A) 0%, Type I error B) 0%, Type Il error C) 5%, Type I error D) 5%, Type Il error E) 15%, Type I error F) 15%, Type IIl error G) 20%, Type I error H) 20%, Type |l error

C. When conducting any experiment, two hypotheses are created. The null hypothesis, or Ho, states that there is no difference between the two groups being compared. The alternative hypothesis, or H, suggests that there is a difference between the two groups being compared. The likelihood of erroneously rejecting the null hypothesis (concluding that the alternative hypothesis is true) is called a type I error. The acceptable rate of a type I error is a percentage called the alpha level and is set by the investigators. When the alpha level is converted to a decimal, it becomes the p-value of the experiment. An alpha level of 5% or 0.05 indicates that if the null hypothesis is true, there is 5% chance that the investigators will erroneously reject it based on the study results. A p-value of 0.05 or less is generally considered statistically significant. In other words, if there really is no difference between the groups overall, 5% is the chance that the study will find a statistically significant difference, and this error is called a Type I error. 17 Incorrect Answers: A, B, D, E, F, G, and H. The investigators have set their significance level at 5%, not 0% (Choice A), 15% (Choice E), or 20% (Choice G). The significance level is equivalent to the p-value which is synonymous with the type I error rate. A level of 15% or 20% would not be considered significant, as it would indicate that the investigators had a 15% or 20% chance of finding a false positive result in their study. 5% is generally the threshold for what is considered statistically significant. A type Il error (Choices B, D, F, and H) occurs when a study demonstrates no difference between the null and alternative hypotheses, when in fact, a difference exists. It is known as a 'false negative' error and can be reduced by increasing the power of the study throu

199 Exam Section 4: Item 49 of 50 National Board of Medical Examiners® Comprehensive Basic Science Self-Assessment 49. A 53-year-old man is brought to the emergency department because of progressive fever, cough, fatigue, and shortness of breath during the past 3 days. His temperature is 39.2°C (102.6°F), and respirations are 21/min. Physical examination shows diaphoresis. There is increased tactile fremitus, dullness to percussion, and bronchovesicular breath sounds over the left lower hemithorax posteriorly. A chest x-ray is shown. Arterial blood gas analysis on oxygen via nasal cannula (2 L/min) shows: pH Pco2 Рог HCO,- 7.46 22 mm Hg 70 mm Hg 15.3 mEq/L Which of the following is the primary cause of the hypoxemia in this patient? A) Global reduction in alveolar aeration B) Inadequate concentration of oxygen through nasal cannula C) Pulmonary arterial blood shunting directly into pulmonary veins D) Pulmonary

D. This patient's signs, symptoms, and chest x-ray are consistent with a diagnosis of community acquired bacterial pneumonia. In adults, commonly implicated organisms include Streptococcus pneumoniae and Haemophilus influenza. Pneumonia classically presents with fever, dyspnea, productive cough, and pleuritic chest pain. Physical examination may disclose bronchovesicular breath sounds, increased tactile fremitus, dullness to percussion, and whispered pectoriloquy over the affected lung segment. Bacterial pneumonia is typically lobar and involves a single lobe, as in this patient. The affected lobe demonstrates increased consolidation caused by the inflammatory infiltration of neutrophils and edema into the alveoli. Consolidation impairs gas exchange within the affected parenchyma and, if sufficient, can cause hypoxia secondary to pulmonary arterial blood traversing poorly aerated alveoli. Incorrect Answers: A, B, C, and E. Global reduction in alveolar aeration (Choice A) is unlikely to occur in the setting of lobar bacterial pneumonia. The unaffected lobes are able to perform gas exchange without impairment. Inadequate concentration of oxygen through nasal cannula (Choice B) is incorrect. While inadequate supplemental oxygen administration may be contributing to hypoxia, the primary cause of the patient's hypoxia is the underlying pneumonia and associated poor gas exchange within the affected lobe. Pulmonary arterial blood shunting directly into pulmonary veins (Choice C) may occur in the setting of arteriovenous malformations. This cause is unlikely to be acute in nature and would therefore be incidental to the patient's acute presentation of bacterial pneumonia. Thickening of the alveolar-capillary interface (Choice E) may result in the impaired diffusion of oxygen, which presents with consequent hypoxia. This may oc

176 Exam Section 4: Item 26 of 50 National Board of Medical Examiners' Comprehensive Basic Science Self-Assessment 26. A study is designed to evaluate the relationship between ambient noise and hearing loss in an automobile manufacturing plant. Different locations in the manufacturing plant have strikingly different levels of ambient noise. Each employee is given a test for hearing acuity, and then the ambient noise level is measured at his or her workstation. Which of the following best describes this study design? A) Case series B) Cohort C) Crossover D) Cross-sectional E) Randomized clinical trial

D. A cross-sectional study seeks to identify the prevalence of a condition at a particular point in time. An example of a cross-sectional study would be a single survey of a population inquiring whether patients have a condition and concurrently inquiring about potential risk factors or exposures for it. Thus, the risk factor and the outcomes are measured simultaneously. This described study does not follow patients over time. All information is collected at a single time point. Causation cannot be determined from a cross-sectional study, only correlation. In this example, the relationship between ambient noise and hearing loss in a manufacturing plant is assessed. Employees of a particular plant are selected for inclusion, and their hearing, along with the ambient noise levels at their stations are measured. This study does not follow participants over time to see if their exposure to increased noise levels lead to hearing loss; it merely assesses the prevalence of the condition at a point in time. Incorrect Answers: A, B, C, and E. A case series (Choice A) is a descriptive study design in which a number of consecutive or nonconsecutive cases of a disease and/or treatment are described in detail, with information about exposure, demographics, and comorbidities. Case series do not imply a cause-and-effect relationship. They do not test a hypothesis nor are they randomized. They are useful in characterizing the natural history of a disease or response to treatment. They are also useful in describing rare diseases, as the small population size may not permit conduction of a larger cohort or randomized trials with sufficient power. A cohort study (Choice B) identifies a group of patients with or without an exposure and follows them over time to identify whether an exposure is associated with an outcome of interest. Cohort

39 Exam Section 1: Item 39 of 50 National Board of Medical Examiners® Comprehensive Basic Science Self-Assessment 39. A 20-year-old man is brought to the physician because of a 4-hour history of abdominal pain, nausea, and vomiting. He says that he had been drinking ethanol heavily all weekend, and he took three doses of acetaminophen within 2 hours after the onset of a severe headache Monday morning. This patient is at increased risk for liver injury because of which of the following actions of ethanol? A) Activation of IgE-mediated mast cell degranulation B) Decreased acetaminophen clearance via glucuronidation C) Increased bioavailability of acetaminophen D) Induction of cytochrome P450 enzymes that activate acetaminophen to a hepatotoxic metabolite E) Metabolic acidosis due to an increased NADH:NAD+ ratio

D. Acetaminophen poisoning is more common in patients with underlying risk factors, including exposure to alcohol and hepatotoxic drugs. When taken at therapeutic doses, acetaminophen is safely metabolized through phase II conjugations, including glucuronidation and sulfation. In the setting of acetaminophen overdose, saturation of phase II metabolic pathways leads to excess acetaminophen metabolized by CYP-mediated reactions to N-acetyl-p-benzoquinoneimine (NAPQI), which has strong oxidizing properties and is directly hepatotoxic. Chronic alcohol use increases the risk for hepatotoxicity in patients who take high doses of acetaminophen. Ethanol leads to the induction of P450 enzymes that result in the increased production of NAPQI when high doses of acetaminophen are ingested. The antioxidant molecule glutathione conjugates NAPQI, allowing it to be safely excreted, and depletion of glutathione is a hallmark of acetaminophen toxicity. Acetaminophen toxicity is treated by repleting hepatic stores of glutathione through intravenous or oral administration of N-acetylcysteine. Incorrect Answers: A, B, C, and E. Activation of IgE-mediated mast cell degranulation (Choice A) is the mechanism of action of type I hypersensitivity reactions and underlies many allergic reactions. Alcohol does not increase hepatic sensitivity to acetaminophen through an allergic mechanism. Decreased acetaminophen clearance via glucuronidation (Choice B) does not occur as a result of alcohol exposure. Rather than affecting the capacity of the liver to perform glucuronidation reactions, alcohol increases the capacity of the liver to produce NAPQI through the induction of P450 enzymes. Increased bioavailability of acetaminophen (Choice C) does not occur. Acetaminophen has high oral bioavailability, which is not altered by the ingestion of alcohol. Me

129 Exam Section 3: Item 29 of 50 National Board of Medical Examiners' Comprehensive Basic Science Self-Assessment 29. Which of the following is the implication of degeneracy of the genetic code? A) The amino acid sequence of a protein derived from a given MRNA can vary B) The code is not universal throughout the biologic species C) A common codon exists for at least two amino acids D) Multiple species of tRNAS exist for most amino acids E) There is a remarkable degree of inaccuracy during transcription

D. Degeneracy describes a unique property of the genetic code in that different codons can encode for the same individual amino acid. Exceptions to this principle include methionine and tryptophan, which are each encoded by only one codon. 64 codons exist based on statistical combination of four nucleotides in groups of three. Three of these codons are stop codons which do not encode for a protein. Therefore, the remaining 61 codons encode for 20 amino acids, demonstrating that there must be redundancy in the codons that correspond to a particular amino acid. As a consequence, each codon has a corresponding transfer-RNA that shuttles its amino acid to the ribosome. Since multiple codons can correspond to any one amino acid, multiple types of transfer RNA exist for most amino acids. The first two positions of the nucleotide match between messenger RNA and transfer RNA are the most important; codons differing in the third position may encode for the same transfer RNA or amino acid, a principle known as "wobble" that stems from degeneracy of the genetic code. Incorrect Answers: A, B, C, and E. The amino acid sequence of a protein derived from a given mRNA can vary (Choice A) is incorrect. The genetic code is unambiguous; that is, each codon designates only one specific amino acid. The code is not universal throughout biologic species (Choice B) is incorrect; the code is widely conserved. A common codon exists for at least two amino acids (Choice C) is incorrect. A specific codon exists for each amino acid, although multiple codons can correspond to any particular amino acid (except for methionine and tryptophan, as described above). There is a remarkable degree of inaccuracy during transcription (Choice E) is incorrect outside of errors in transcription. Educational Objective: Degeneracy describes a unique property of the

70 Exam Section 2: Item 20 of 50 National Board of Medical Examiners® Comprehensive Basic Science Self-Assessment 20. A 45-year-old man who is a firefighter comes to the physician because of a 3-month history of persistent cough and repeated lung infections. He has smoked 2 packs of cigarettes daily for 30 years. His respirations are 16/min. Physical examination shows dry cough and decreased lung sounds. A chest x-ray shows no abnormalities. Histologic examination of this patient's respiratory epithelium is most likely to show which of the following types of metaplasia? A) Columnar B) Connective tissue C) Neuroendocrine D) Squamous E) Transitional

D. Chronic tobacco use is associated with numerous detrimental changes to the respiratory system. Tobacco smoke directly disrupts the ability of the airway to clear foreign material by resulting in increased mucus production and secretion by the respiratory epithelium, decreased activity of airway cilia, and inhibition of alveolar macrophage function. Chronic irritation from tobacco exposure also results in squamous metaplasia of the respiratory epithelium, which is normally primarily composed of pseudostratified columnar cells in the trachea and bronchi, and cuboidal cells in the bronchioles. Šquamous metaplasia is a preneoplastic lesion that has the potential to transform into squamous cell carcinoma, which is the second most common type of primary lung cancer following adenocarcinoma. Features associated with squamous cell carcinoma of the lung include pulmonary cavitations, central location, and hypercalcemia caused by paraneoplastic parathyroid hormone-related peptide (PTHRP) production. Incorrect Answers: A, B, C, and E. Columnar (Choice A) metaplasia occurs with Barrett esophagus because of chronic chemical irritation from gastric contents. The normal squamous epithelium of the esophagus transforms into columnar epithelium and is associated with an increased risk for esophageal adenocarcinoma. Connective tissue (Choice B) metaplasia refers to abnormal formation of mesenchymal cells (bone, cartilage, adipose) in tissue. An example is myositis ossificans, the formation of bone in muscle tissue that may occur following trauma. Neuroendocrine (Choice C) tumors of the lung, such as small cell lung cancer, result from hyperplasia and malignant transformation of normal neuroendocrine cells residing in the lung, rather than metaplasia in response to an irritant. Transitional (Choice E) epithelium is located in the geni

92 Exam Section 2: Item 42 of 50 National Board of Medical Examiners® Comprehensive Basic Science Self-Assessment 42. A 25-year-old man comes to the physician with his wife because they have been unable to conceive for 2 years. Previous examination of the wife showed no abnormalities. Examination of the patient shows no abnormalities. The patient's laboratory studies show autoantibodies to sperm. Dysfunction of which of the following cell types is the most likely cause of these findings? A) Leydig cells B) Primary spermatocytes C) Secondary spermatocytes D) Sertoli cells E) Type A spermatogonia

D. In normal male anatomy, the Sertoli cells lining the seminiferous tubules create a blood-testis barrier through tight junctions which allows the seminiferous tubule to exist as an immune-protected site. The spermatozoa developing within the seminiferous tubule contain genetic material that may be foreign to the body as a result of recombination during meiosis. Thus, when this barrier is intact, it prevents potential novel antigens from leaving the seminiferous tubule and encountering the host immune system. If the barrier is disrupted, as may occur in testicular trauma, the spermatozoa can elicit an immune response that may include production of autoantibodies against the gametes. Sperm autoantibodies are a theoretical cause of male infertility. Other causes of male infertility include Klinefelter syndrome (XXY karyotype causing dysgenesis of the seminiferous tubules and testicular atrophy), increased testicular temperature (seen in cryptorchidism or varicocele), or gonadotropin-releasing hormone (GnRH) deficiency. Incorrect Answers: A, B, C, and E. Spermatogenesis takes place in the seminiferous tubules of the testes. Pulsatile GNRH produced by the hypothalamus causes the pituitary to secrete follicle stimulating hormone (FSH) and luteinizing hormone (LH) which drive spermatogenesis. LH acts on the Leydig cells (Choice A), located adjacent to but outside of the seminiferous tubules, and causes them to secrete testosterone, which is necessary to create the proper hormonal environment within the seminiferous tubule required for spermatogenesis. Dysfunction of Leydig cells would not damage the blood-testis barrier leading to antibody formation. Lining the seminiferous tubules are spermatogonia, which generate the primary spermatocytes (Choice B) that then mature into secondary spermatocytes (Choice C) and will ultimat

t disease) D) Osteoporosis E) Tuberculosis

D. Osteoporosis is a common condition that is characterized by the progressive loss of bone mineral density leading to decreased bone strength. This decreased bone mineral density often leads to fragility fractures, which can greatly decrease mobility and even increase the risk for death in elderly individuals. The disease classically affects postmenopausal women with inflammatory disorders, and individuals with metabolic or endocrine disorders such as hypercortisolism or long-term steroid use, as in this patient with chronic ulcerative colitis. Glucocorticoids significantly alter calcium metabolism by decreasing gastrointestinal calcium absorption, enhancing osteoclastogenesis, and inhibiting osteoblast differentiation. Primary prevention of osteoporosis is indicated in the setting of chronic glucocorticoid use to prevent premature, pathologic fragility fractures. Vitamin D and calcium are recommended for all patients at low risk for fracture. Patients at moderate to high risk for fracture are treated with calcium, vitamin D, and oral bisphosphonates, such as alendronate. Bisphosphonates have been shown to reduce fracture risk and increase bone density and are the mainstay of osteoporosis treatment and prevention in patients that are at moderate to high risk for bone mineral density loss caused by corticosteroid use. In this case, this young patient sustained a fracture during exercise, which for most persons would not present high risk. Her x-ray shows compression fracture deformities of the lumbar spine, an uncommon injury in a young person, suggesting an underlying diagnosis of osteoporosis. Incorrect Answers: A, B, C, and E. Ankylosing spondylitis (Choice A) is an inflammatory condition of the spine that classically presents in younger males with back pain and stiffness relieved with exercise; it results in calcif

151 Exam Section 4: Item 1 of 50 National Board of Medical Examiners® Comprehensive Basic Science Self-Assessment 1. A 16-year-old girl is brought to the emergency department because of a 2-hour history of rash and burning pain on her shoulders and arms. She had been hiking outdoors on a sunny day for several hours before the symptoms began. Her only medication is oral tetracycline for facial acne. Her temperature is 37°C (98.6°F). Physical examination shows diffuse erythema over the shoulders and upper extremities, and bullous lesions over the shoulders. Which of the following is the most likely diagnosis? A) Atopic dermatitis B) Contact dermatitis C) Drug-induced lupus D) Phototoxic reaction O E) Polymorphous light eruption

D. Phototoxicity is a common adverse effect of oral tetracyclines including tetracycline, doxycycline, and minocycline. All patients prescribed these medications should be counseled on the importance of sun-protective behaviors including using sunscreen, avoiding mid-day sun, and wearing sun-protective clothing. Clinical manifestations of phototoxicity include erythema and blistering in the distribution of sun exposure, similarly to a severe sunburn. Óral tetracyclines may be used for moderate to severe inflammatory acne. While there is a component of bacterial overgrowth to the pathogenesis of acne, these antibiotics are used primarily for their anti-inflammatory effect. In addition to counseling on phototoxicity, all patients prescribed oral tetracyclines should be counseled on the possibility of gastrointestinal upset and pill esophagitis. Incorrect Answers: A, B, C, and E. Atopic dermatitis (Choice A), or eczema, is a chronic pruritic inflammatory skin disease common in children that is suggested by a personal or family history of atopic disease (eg, asthma or allergic rhinitis). The rash is initially characterized by erythematous papules and vesicles with exudates and crusting; with chronic scratching, the rash becomes dry, scaly, and excoriated in a process called lichenification. It is not usually confined to photo-exposed areas. Contact dermatitis (Choice B) is a type IV hypersensitivity reaction caused by exposure of the skin to either an irritant or allergen causing an eczematous rash to occur in the distribution of the exposure. Acute contact dermatitis is characterized by small weeping vesicles with surrounding erythema. Photoallergic dermatitis is a form of contact dermatitis in which an antigen is altered by sun exposure to become allergenic. However, a phototoxic reaction is more characteristic of tetra

172 Exam Section 4: Item 22 of 50 National Board of Medical Examiners' Comprehensive Basic Science Self-Assessment 22. Protection against paralytic poliomyelitis can be achieved either by oral administration of live attenuated poliovirus (Sabin) vaccine or by parenteral administration of killed poliovirus (Salk) vaccine. The common features of these two vaccines, which accounts for their efficacy, is their ability to induce which of the following poliovirus-specific immune responses? A) Activated CD8+ cytotoxic T lymphocyte effectors in the circulation B) Activated CD8+ cytotoxic T lymphocyte effectors in the gut C) CD8+ memory T lymphocytes D) Neutralizing antibodies in the circulation E) Neutralizing secretory IgA antibodies in the gut

D. Production of neutralizing antibodies in the circulation is the goal of vaccination and can be achieved through several methods. The poliovirus vaccine can be given orally as a live attenuated virus or parenterally as a killed virus. Regardless of the route of administration, poliovirus is endocytosed by antigen presenting cells (APCS) and sampled by B lymphocytes directly. B lymphocytes do not require antigen presentation by other cells in order to recognize a specific peptide, so their activation is not necessarily dependent upon the presence of other APČS. Binding of poliovirus-specific antigen to the Fab region of a B lymphocyte receptor leads to activation of the B lymphocyte, followed by somatic hypermutation of the Fab region that increases the affinity of the B-cell receptor for its antigen. This promotes B lymphocyte differentiation into plasma cells, which are able to secrete specific neutralizing immunoglobulins. This process is enhanced by concomitant stimulation of T lymphocytes. Endocytosis of poliovirus particles by APCS leads to processing of these peptides in the endosome and conjugation with class II MHC molecules. This antigen-MHC complex is expressed on the surface of the APC and is recognized by specific CD4+ T lymphocytes which, when activated, further augment the process of B-lymphocyte development and maturation. Incorrect Answers: A, B, C, and E. Activated CD8+ cytotoxic T lymphocyte effectors in the circulation (Choice A) and the gut (Choice B) is a characteristic part of viral infection and may also occur with poliovirus vaccine depending upon the route of administration, although it is not the primary means by which long-lasting immunity is attained. CD8+ T lymphocytes are commonly activated after recognizing viral particles conjugated to class I MHC molecules on the surface of APCS and

80 Exam Section 2: Item 30 of 50 National Board of Medical Examiners® Comprehensive Basic Science Self-Assessment 30. A 12-year-old girl is brought to the physician by her mother after the dentist found many unerupted and supernumerary teeth. Physical examination shows frontal bossing, hypertelorism, and retained deciduous teeth. When prompted, the patient is able to place her shoulders together anteriorly. Serum studies show a decreased alkaline phosphatase activity. A chest x-ray shows hypoplasia of the clavicles. Skull x-rays show open sutures and multiple Wormian bones. Genetic testing shows a mutation in the CBFA1 gene. Which of the following cell types is directly affected by the mutation in this patient? A) Chondroblasts B) Chondroclasts C) Chondrocytes D) Osteoblasts E) Osteoclasts F) Osteocytes

D. Osteoblast differentiation is likely impaired in this patient with a physical presentation concerning for cleidocranial dysplasia (CCD). CCD is inherited either de novo or in an autosomal dominant manner and presents with bony dysplasia. Diminution or absence of the clavicle permits abnormal movement of the shoulders anteriorly as seen in this patient. Additional symptoms include impaired skull development, often involving the frontal or parietal bones, the presence of Wormian (intrasutural) bones, supernumerary teeth, hypoplasia of the maxilla, frontal bossing, hypertelorism, and delayed ossification of the pubic symphysis. CBFA1 (core binding factor activity 1) gene may be mutated. This gene codes for a transcription factor involved in osteoblast differentiation. Membranous bone, often found along the midline, is most commonly affected. The life expectancy of affected patients is generally normal, as is neurologic development. Incorrect Answers: A, B, C, E, and F. Chondroblasts (Choice A), chondroclasts (Choice B), and chondrocytes (Choice C) are the primary cell types involved in the synthesis (-blast) and breakdown (-clast) of cartilage. In pathology, chondroblastoma can result from the oversynthesis of cartilage from chondroblast stimulation. They are not involved in the pathophysiology of CCD, which results from impaired osteoblast differentiation as opposed to imbalanced synthesis or breakdown of cartilage. Osteoclasts (Choice E) are the primary cell type involved in the degradation of bone. Excess osteoclast activation results in loss of bone mineral density and weakened bone, as seen in multiple myeloma and

177 Exam Section 4: Item 27 of 50 National Board of Medical Examiners' Comprehensive Basic Science Self-Assessment 27. A 27-year-old woman with a 15-year history of recurrent urinary tract infections develops hypertension, proteinuria, and renal failure. Her serum creatinine concentration is 2.6 mg/dL. X-rays show irregular scarring of the kidneys and blunting of the renal calyces, most prominent in the upper and lower poles. Which of the following is the most likely cause of the renal abnormalities? A) Acute glomerulonephritis B) Acute pyelonephritis C) Amyloidosis D) Reflux nephropathy E) Type 2 diabetes mellitus

D. Reflux nephropathy refers to renal damage secondary to vesicoureteral reflux (VUR), in which there is retrograde flow of urine from the bladder into the ureters and renal calyces. VUR often begins during childhood and may occur because of anatomical abnormalities in the structure of the ureteric valves or in the setting of recurrent urinary tract infections. VUR often presents with postrenal azotemia and hydroureteronephrosis, with the possibility of developing chronic renal failure if uncorrected. Patients may present with varying degrees of dilated ureters, renal pelves, blunted renal calyces, and/or compressed, atrophic renal parenchyma. The extent and distribution of upper urinary tract dilation and renal parenchymal atrophy depends on the severity and chronicity of vesicoureteral reflux. Patients are at an increased risk for recurrent urinary tract infections, including pyelonephritis. Incorrect Answers: A, B, C, and E. Acute glomerulonephritis (Choice A) may result in damage to the glomerular architecture and changes to the renal parenchyma on biopsy but would not be expected to produce blunting of the renal calyces, which is consistent with a postrenal process. Acute pyelonephritis (Choice B) can lead to inflammation of the renal parenchyma and may cause complications such as renal papillary necrosis. While calyceal blunting and scarring is sometimes observed in the setting of chronic pyelonephritis, it is more suggestive of VUR and associated reflux nephropathy in this patient. Amyloidosis (Choice C) of the kidneys can lead to nephrotic syndrome. It is characterized by the presence of amyloid deposits that are visible by Congo red staining and appear as apple-green birefringent deposits in the mesangium under polarized light. Amyloidosis would not cause other findings suggestive of VUR, such as recurrent uri

114 Exam Section 3: Item 14 of 50 National Board of Medical Examiners' Comprehensive Basic Science Self-Assessment 14. A 25-year-old man comes to the emergency department because of a 3-day history of abdominal cramps and diarrhea. He appears anxious. Physical examination shows piloerection and diffuse abdominal tenderness. Neurologic examination shows no abnormalities. He is oriented to person, place, and time. The most likely cause of this patient's condition is withdrawal from which of the following substances? A) Alcohol B) Benzodiazepine C) Cocaine D) Heroin O E) Marijuana

D. This patient is likely withdrawing from heroin, an opiate. Opiates are central nervous system (CNS) depressants. Opiate intoxication causes sedation, bradycardia, hypotension, depressed respiratory drive, and constricted pupils. Opiates are commonly used for analgesia and act on receptors within the gastrointestinal tract, reducing gut motility and causing constipation. Opiate withdrawal typically presents with restlessness, tachycardia, hypertension, pupillary dilation, tremors, muscle cramping, abdominal cramping, nausea, vomiting, and diarrhea. Lacrimation and rhinorrhea can also occur, along with yawning and piloerection, which are not typically seen in withdrawal from other recreational drugs. Withdrawal from opiates is not fatal though is generally unpleasant for patients. Treatment is supportive and includes benzodiazepines, antiemetics, and clonidine during the acute withdrawal period, ideally with transition to a partial opioid agonist (eg, buprenorphine) or long-acting agonist (eg, methadone) if the patient is at risk for relapse. Incorrect Answers: A, B, C, and E. Alcohol (Choice A) and benzodiazepines (Choice B) are CNS depressants that, in toxic doses, can cause slurred speech, ataxia, emotional lability, and memory lapses. Withdrawal from alcohol and benzodiazepines can lead to anxiety, tremors, perceptual disturbances, and seizures. Alcohol withdrawal can additionally cause nausea, vomiting, and the life-threatening complication delirium tremens (benzodiazepine withdrawal less commonly leads to delirium tremens). This patient experienced piloerection, which would not occur in alcohol or benzodiazepine withdrawal. Cocaine (Choice C) is a CNS stimulant that causes euphoria and increases sympathetic tone, leading to restlessness, tachycardia, hypertension, and pupillary dilation. Because of increased syn

97 Exam Section 2: Item 47 of 50 National Board of Medical Examiners® Comprehensive Basic Science Self-Assessment 47. A 46-year-old woman comes to the physician because of a 2-year history of progressive shortness of breath while supine; she now requires two pillows to help her sleep. She also has had a cough with occasional production of rusty-colored sputum during this period. Her respirations are 16/min. Bilateral crackles and a grade 4/6 mid-diastolic murmur are heard. Echocardiography shows left atrial dilation with mitral stenosis. No other valvular lesions are noted. The photograph shown is characteristic of her disease process. Which of the following is the most likely cause of these findings? A) Congenital rubella B) Coronary artery atherosclerosis C) Illicit intravenous drug use D) Recurrent group A streptococcal infections E) Systemic sclerosis (scleroderma) with CREST syndrome

D. Rheumatic heart disease is a complication of acute rheumatic fever caused by group A streptococcal infection resulting in progressive inflammatory damage to the heart valves with fibrosis and calcification. It commonly causes insufficiency in the acute phase and stenosis later in life, predominantly affecting the mitral valve. Rheumatic fever typically presents with joint inflammation, pancarditis, subcutaneous inflammatory nodules, erythema marginatum, and Sydenham chorea. Mitral stenosis presents with a diastolic murmur radiating to the axilla. If severe enough, it can result in left atrial enlargement, cardiogenic pulmonary edema, and arrhythmias such as atrial fibrillation and flutter. Incorrect Answers: A, B, C, and E. Congenital rubella (Choice A) syndrome is caused by fetal rubella infection during pregnancy, which results in severe malformations in the fetus. It is diagnosed in the neonatal period and classically presents with bilateral hearing loss, congenital heart defects, a violaceous rash, and hepatosplenomegaly. Coronary artery atherosclerosis (Choice B) may result in chronic ischemic changes to the myocardium and risk for papillary muscle rupture in the setting of acute tissue infarction. It does not cause chronic inflammation and calcification of the mitral valve. Illicit intravenous drug use (Choice C) is a risk factor for infective endocarditis caused by the introduction of microbes into the venous circulation. The tricuspid valve is most commonly affected. Vegetations may form on the valve, and patients may present with fever, dyspnea, tricuspid regurgitation, and septic pulmonary emboli. Systemic sclerosis (scleroderma) with CREST syndrome (Choice E) is a subtype of scleroderma characterized by fibrotic skin changes of the distal extremities and face; it is associated with calcinosis cutis, Rayna

44 Exam Section 1: Item 44 of 50 National Board of Medical Examiners® Comprehensive Basic Science Self-Assessment 44. A 34-year-old woman comes to the physician because of a 10-year history of constipation. She usually has fewer than three bowel movements weekly. She often has bloating and cramping, and she has to strain and apply perineal pressure to defecate. She says that she does not have problems with sleep, appetite, or energy. Palpation of the abdomen produces diffuse discomfort; bowel sounds are present. Which of the following is the most likely cause of this patient's gastrointestinal symptoms? A) Colon polyps B) Congenital megacolon (Hirschsprung disease) C) Factitious disorder D) Gluten enteropathy E) Irritable bowel syndrome F) Major depressive disorder

E. Irritable bowel syndrome (IBS) is characterized by recurrent, intermittent abdominal pain and alteration of bowel habits as either a change in stool frequency or consistency. The abdominal pain may be either exacerbated or relieved by defecation and can vary widely in location and character. IBS commonly presents with intermittent episodes of diarrhea and/or constipation, often which alternate. When diarrhea does occur, it usually does so during waking hours which serves to differentiate it from other causes of diarrhea. It is most common in middle-aged women and the pathogenesis is unknown. It is not caused by a structural abnormality, and physical examination will show nonspecific abdominal discomfort rather than localized pain. Stress management may improve bowel habits in patients with irritable bowel syndrome, which is often associated with concomitant anxiety. If medication is needed, dicyclomine is an antispasmodic and diphenoxylate slows gut motility. Both are used in the management of irritable bowel syndrome. Incorrect Answers: A, B, C, D, and F. Colon polyps (Choice A) present in a variety of subtypes, from non-neoplastic polyps (eg, hamartomatous, mucosal, inflammatory, hyperplastic) to potentially malignant polyps (adenomatous, serrated). Polyps are typically asymptomatic and are only recognized after screening colonoscopy. They are not a common cause of chronic constipation. Inflammatory polyps may be seen in inflammatory bowel disease, but not irritable bowel syndrome. Congenital megacolon (Hirschsprung disease) (Choice B), or intestinal aganglionosis, is caused by the congenital absence of the distal portion of the myenteric plexus, a part of the enteric nervous system located between the inner and outer layers of the muscularis externa. This often leads to a failure to pass stool within the first fe

38 Exam Section 1: Item 38 of 50 National Board of Medical Examiners® Comprehensive Basic Science Self-Assessment 38. A 29-year-old woman comes to the physician because of a 6-day history of rash, joint pain, and fatigue. Her temperature is 37.4°C (99.4°F), and blood pressure is 150/90 mm Hg. Physical examination shows a raised, erythematous, blotchy malar rash and mild peripheral edema. There is tenderness and swelling of several joints. Laboratory studies show: Leukocyte count Segmented neutrophils Eosinophils Lymphocytes Monocytes Platelet count 3200/mm3 62% 3% 29% 6% 74,000/mm3 2.8 g/dL positive positive Serum albumin Antinuclear antibody Antibody to double-stranded DNA Urine Protein RBC casts 3+ 2+ Which of the following serologic studies is most likely to produce a false-positive result in this patient? A) Cold agglutinins B) Hepatitis B surface antigen O C) Heterophile antibody D) p24 antigen E) Rapid plasm

E. A rapid plasma reagin false positive result in this patient would be expected. This young woman with malar rash, arthritis, leukopenia, thrombocytopenia, positive antinuclear antibody (ANA), positive anti double-stranded DNA antibody (anti-dsDNA), and an active urine sediment meets criteria for a diagnosis of systemic lupus erythematosus (SLE). Many patients with SLE also possess one or more antiphospholipid antibodies, including anticardiolipin antibody, lupus anticoagulant, or anti-beta-2-glycoprotein-1 antibody, which predisposes to the development of venous or arterial thrombosis and recurrent pregnancy loss. Additionally, these antibodies are known to interfere with the rapid plasma reagin (RPR) test for syphilis. The RPR tests for the presence of antibodies against a cardiolipin-cholesterol-lecithin antigen, which are called reagin antibodies. In patients who possess an anti-cardiolipin antibody, the RPR will be falsely positive. Follow up tests to evaluate for infection with syphilis should involve a treponemal test such as Treponema pallidum particle agglutination assay (TPA) or fluorescent treponemal antibody absorption (FTA-ABS) test. Incorrect Answers: A, B, C, and D. Cold agglutinins (Choice A) are circulating IgM antibodies against erythrocyte antigens that, upon binding to the erythrocyte antigen, cause agglutination and extravascular hemolysis. These antibodies are active at colder temperature, so agglutination occurs primarily in the extremities. This is a rare cause of autoimmune hemolytic anemia in patients with SLE. Hepatitis B surface antigen (HBSA9) (Choice B) may persist for several weeks after hepatitis B vaccination, but a false positive HBSAg would not be expected in SLE. Heterophile antibodies (Choice C) are antibodies that react to antigens from unrelated species such as sheep or horse ery

76 Exam Section 2: Item 26 of 50 National Board of Medical Examiners® Comprehensive Basic Science Self-Assessment 26. A 45-year-old woman comes to the physician because of a 3-week history of pain over her lateral left hip that is most severe when she awakens in the morning. She is unable to sleep on her left side due to the pain. Range of motion of the left hip is full with no pain on internal or external rotation. There is tenderness to palpation over the lateral hip. Sensation is intact. Inflammation of which of the following structures is the most likely cause of this patient's symptoms? A) Hip joint B) Inguinal ligament C) Lateral femoral cutaneous nerve D) Sacroiliac joint E) Trochanteric bursa

E. Bursae are thin, sac-like, fluid-filled structures that reduce friction and permit motion between layers of connective tissue. Bursae are frequently implicated in pathology and are often associated with pain and swelling, such as the subacromial bursa, which can cause pain in the shoulder, the trochanteric bursa, which can cause pain over the lateral aspect of the hip (in this case), and the olecranon bursa, which can cause pain and swelling posterior to the elbow. When inflamed or injured, fluid can collect inside the potential space of the bursa causing swelling, which can be appreciated on examination as a fluctuant, tender, circumscribed mass. Trochanteric bursitis presents with pain to palpation over the affected bursa and swelling and erythema in the affected area, but generally without limited range of motion or pain in the hip joint itself. If erythema and warmth are present, these findings can signify infection of the bursa (septic bursitis), which should be treated with antibiotics. In this patient's case, an intact sensory examination, full range of motion of the joint, and focal tenderness in the area of a bursa implicates trochanteric bursitis as the likely diagnosis. Incorrect Answers: A, B, C, and D. Hip joint (Choice A) pain can arise from pathologies such as osteoarthritis, avascular necrosis, synovitis, or septic arthritis. Pathology within the joint would present with pain and tenderness with range of motion of the joint, whereas this patient has normal range of motion and pain focally over the lateral hip. Inguinal ligament (Choice B) inflammation is uncommon, however pain in the inguinal region is a frequent presenting complaint. Etiologies of such pain include but are not limited to hernias (inguinal or femoral), ligament and muscle strain and sprain, neuropathic pain, stress fractures, referre

90 Exam Section 2: Item 40 of 50 National Board of Medical Examiners® Comprehensive Basic Science Self-Assessment 40. A 16-year-old boy is brought to the emergency department 30 minutes after he dove into a 3-foot-deep swimming pool at night. On examination, he is unable to move the right upper and lower extremities. The most likely cause of the movement deficits in this patient is damage to which of the following labeled regions in the photograph of a cross section of the spinal cord shown? A BC D Right Left A) B) C) D) E) F) G) H)

E. Label E represents the right lateral corticospinal tract in this cervical spinal cord section (as indicated by the ovoid shape and predominance of white matter). Upper motor neurons of the corticospinal tract originate in the primary motor cortex, descend ipsilaterally through the internal capsule and midbrain, decussate in the caudal medulla, and then descend contralaterally in the spinal cord to synapse with the contralateral lower motor neuron. Since the decussation occurs superior to the cervical spinal cord, lesions involving the lateral corticospinal tract within the cervical spinal cord will lead to ipsilateral motor weakness and an upper motor neuron pattern of dysfunction (eg, increased muscle tone, hyperreflexia, Babinski reflex). Traumatic spinal cord injuries can occur through diverse mechanisms such as cord compression by a vertebral column fracture. Given this patient's hemiparesis of the right upper and lower extremities, the location of the injury was likely at the C5 level or higher. Incorrect Answers: A, B, C, D, F, G, and H. Labels A and D represent the fasciculi cuneatus of the dorsal column-medial lemniscus tract, while labels B and C represent the fasciculi gracilis of the dorsal column-medial lemniscus tract. The dorsal column-medial lemniscus tract decussates in the medulla. Dorsal column-medial lemniscus tract lesions within the cervical spinal cord lead to ipsilateral deficits in pressure, vibration, fine touch, and proprioception. The fasciculi gracilis contain these sensory fibers for the lower body, while the fasciculi cuneatus contain these sensory fibers for the upper body. Label F represents the left lateral corticospinal tract. Lesions in this location would result in an upper motor neuron pattern of dysfunction in the left upper and/or lower extremities. Labels G and H represent the

33 Exam Section 1: Item 33 of 50 National Board of Medical Examiners® Comprehensive Basic Science Self-Assessment 33. A 56-year-old woman comes to the physician because of a 3-week history of increasing weakness of her arms and legs. She has smoked 1 pack of cigarettes daily for 30 years. Physical examination shows pronounced weakness of the hip girdle muscles and lesser weakness of the shoulder girdle muscles, both of which improve with repetitive testing. A chest x-ray shows a hilar lung mass. Electromyography is compatible with a malfunction of the neuromuscular junction. Impairment of which of the following is the most likely cause of these findings? A) Acetylcholinesterase B) Binding of acetylcholine on the postsynaptic membrane C) Direct depolarization of muscle fibers by Ca2+ D) Postsynaptic membrane potential E) Presynaptic release of acetylcholine

E. Lambert-Eaton myasthenic syndrome (LEMS) is an uncommon neurologic condition that affects neuromuscular junction transmission. Normally, action potentials lead to depolarization in the terminal bouton of the axon. Voltage-gated calcium channels consequently open and allow calcium influx, which triggers the exocytosis of neurotransmitter-filled vesicles into the synaptic cleft. Patients with LEMS possess antibodies to voltage-gated calcium channels on presynaptic lower motor neurons, which decreases calcium influx and acetylcholine release to postsynaptic skeletal muscle cells. This patient with a smoking history and central lung mass likely has a small cell lung cancer that is producing autoantibodies to the voltage-gated calcium channel, causing a paraneoplastic LEMS. Patients consequently demonstrate progressive proximal muscle weakness with depressed or absent deep tendon reflexes. The muscle weakness typically improves with repetitive motion and exercise as small amounts of acetylcholine accumulate in the synaptic cleft over time, increasing the frequency of postsynaptic action potentials in skeletal muscle cells. Autonomic symptoms such as decreased salivation and consequent dry mouth may also occur. The diagnosis may be confirmed with nerve conduction studies and electromyography with exercise testing. Removal of the antibody-producing malignancy is crucial for treatment. Incorrect Answers: A, B, C, and D. Acetylcholinesterase (Choice A) inhibition leads to decreased degradation of synaptic acetylcholine and is the mechanism of medications (eg, pyridostigmine) utilized for myasthenia gravis. This mechanism typically improves muscle strength and would not be expected to lead to the progressive muscle weakness demonstrated in LEMS. Impairment of the binding of acetylcholine on the postsynaptic membrane (Choice B

154 Exam Section 4: Item 4 of 50 National Board of Medical Examiners® Comprehensive Basic Science Self-Assessment 4. A 45-year-old man comes to the physician for evaluation of worsening heartburn, which has been present for the past 15 months. The pain has become more frequent and severe in the past month and is no longer relieved by antacids. He also has occasional trouble swallowing. Endoscopic examination shows erythema of the lower third of the esophagus. Microscopic biopsies of this area show basal zone hyperplasia and an eosinophilic infiltrate of the squamous mucosa. Which of the following is the most likely diagnosis? A) Achalasia B) Barrett esophagus C) Candidiasis D) Early squamous carcinoma E) Reflux esophagitis

E. Reflux esophagitis, or gastroesophageal reflux disease (GERD), typically presents with burning epigastric and lower chest pain, often in association with consumption of a large meal or trigger food. It is often worse with supine positioning. Acid reflux in the esophagus causes mucosal irritation and inflammation, which can present as mucosal erythema, hyperplasia, eosinophilic infiltrate, and erosions in reflux esophagitis. Over time, if GERD remains untreated, metaplasia can occur leading to Barrett esophagus, a state of pre-malignancy marked by intestinal metaplasia of the distal esophagus. This can subsequently lead to esophageal adenocarcinoma. Omeprazole irreversibly inhibits the HK ATPase in gastric parietal cells. Reduction of stomach acid production allows for neutralization of stomach contents, limiting the inflammatory and erosive sequelae of GERD in the distal esophagus, and allowing for appropriate healing of previous esophagitis. Incorrect Answers: A, B, C, and D. Achalasia (Choice A) is an esophageal dysmotility disorder resulting from deficient peristalsis along the length of the esophagus and impaired relaxation of the lower esophageal sphincter. It manifests as dysphagia, odynophagia, weight loss, halitosis, and regurgitation of undigested food. It is diagnosed through a barium swallow plus esophageal manometry. Barrett esophagus (Choice B) is a state of pre-malignancy marked by intestinal metaplasia on histology of the distal esophagus. Untreated, this can subsequently lead to esophageal adenocarcinoma. Candidiasis (Choice C) in the esophagus can cause dysphagia. Candida esophagitis is a common opportunistic infection in an immunocompromised host such as in HIV. On examination, patients may have oropharyngeal thrush. Early squamous carcinoma (Choice D) of the esophagus can cause dysphagia and weigh

78 Exam Section 2: Item 28 of 50 National Board of Medical Examiners® Comprehensive Basic Science Self-Assessment 28. A 25-year-old man is brought to the emergency department because of a 2-hour history of nausea, vomiting, abdominal cramping, and difficulty passing flatus. Examination of the face shows the lesions in the photograph; similar lesions are seen on the fingers and toes. Laboratory studies show hypochromic microcytic anemia. Test of the stool for occult blood is positive. Which of the following is the most likely diagnosis? A) Cowden disease B) Gardner syndrome C) Muir-Torre syndrome D) Neurofibromatosis E) Peutz-Jeghers syndrome

E. Peutz-Jeghers syndrome is an autosomal dominant polyposis in which hamartomatous polyps occur in the colon and gastrointestinal (GI) tract, and pigmented macules are found in or around the mouth, lips, hands, and genitalia. It is associated with increased risk for breast and Gl tract cancers (eg, colorectal, stomach, small bowel, pancreatic). Hamartomatous polyps are characterized by disorganized growth of tissue similar to normal native tissue. When solitary, they are usually benign and do not carry a significant risk for malignant transformation, however when syndromic they are associated with increased risk in gastrointestinal tract cancers. Patients with Peutz-Jeghers syndrome may have intestinal obstruction or intussusception as their presenting symptom, such as in this patient. Polyps can serve as a lead point for intussusception or obstruct the gastrointestinal lumen. Polyps can also cause chronic occult bleeding, leading to a hypochromic microcytic anemia. Incorrect Answers: A, B, C, and D. Cowden disease (Choice A) is an autosomal dominant polyposis associated with hamartomatous polyps in the GI tract. It is associated with mucocutaneous neuromas, oral papillomas, and cutaneous trichilemmomas. Gardner syndrome (Choice B) is a familial adenomatous polyposis characterized by the presence of thousands of adenomatous polyps arising after puberty. It is associated with osteomas, soft tissue tumors, supernumerary teeth, and hypertrophy of the retinal pigment epithelium. Muir-Torre syndrome (Choice C) is a form of hereditary nonpolyposis colorectal cancer. Affected individuals are prone to developing colon, genitourinary, or skin cancers. Cutaneous manifestations can include keratoacanthomas and sebaceous gland tumors. Neurofibromatosis (Choice D) type 1 is a heritable neurocutaneous disorder that presents with cu

175 Exam Section 4: Item 25 of 50 National Board of Medical Examiners' Comprehensive Basic Science Self-Assessment 25. A 22-year-old man is brought to the emergency department 15 minutes after sustaining a gunshot wound to the abdomen. His pulse is 110/min, respirations are 20/min, and blood pressure is 100/60 mm Hg. Abdominal examination shows an entrance wound in the left upper quadrant and no exit wound. A CT scan of the abdomen is shown. The injured organ is removed. The bullet is most likely lodged in which of the following organs? A) Adrenal gland B) Kidney C) Left ventricle D) Lung E) Spleen O F) Stomach

E. The axial image of the CT scan shows a hyperdense foreign body, the bullet, in the left upper quadrant of the abdomen. Structures in the left upper quadrant of the abdomen include the stomach, spleen, and tail of the pancreas. The liver is found in the right upper quadrant. The base of the heart lies anteriorly in the thorax. The spleen is a uniformly hyperdense solid organ that is typically seen opposite the liver and posterior to the stomach in the left upper quadrant. It sits inferior to the diaphragm. In the CT scan, hypodense fluid is seen surrounding the spleen, likely reflecting hemoperitoneum, a common complication of penetrating trauma to the spleen. Incorrect Answers: A, B, C, D, and F. The adrenal gland (Choice A) is not in view on this CT scan. The adrenal glands are located superior to each kidney, and the kidneys (Choice B) are retroperitoneal structures inferior to the axial plane of this CT. Neither are seen in this image. The adrenal glands and kidneys are located near vertebral levels T12-L2. The left ventricle (Choice C) is located inferior, anterior, and lateral to the left atrium. It abuts the left lung and constitutes the majority of the left border of the heart. It is located anterior in the thorax, whereas the hyperdense bullet is seen in the posterior left upper quadrant of the abdomen, with the diaphragm demarcating the superior boundary of the abdomen. The lung (Choice D) is hypodense on CT scan because of a large composition of air when compared to other solid organs. The lungs are contained within in the thorax, whereas and the hyperdense bullet is seen in a solid organ in the left upper quadrant, the spleen. The right and left lungs are visible on this axial section as hypodense crescents abutting the body wall. The stomach (Choice F) is a hollow structure containing air and chyme, foun

135 Exam Section 3: Item 35 of 50 National Board of Medical Examiners' Comprehensive Basic Science Self-Assessment 35. Which of the following labeled structures in the sagittal MRI of a knee is largely responsible for preventing excessive posterior movement of the tibia with respect to the femur? 102/5 21/2 2.8 A E Anterior Posterior A) B) C) D) E)

E. The posterior cruciate ligament (PCL) is a dense band of connective tissue that connects the tibia to the femur within the knee joint and prevents excessive posterior motion of the tibia with respect to the femur. It connects the posterior intercondylar tibia to the medial condyle of the femur. By contrast, the anterior cruciate ligament (ACL) prevents excess anterior motion of the tibia with respect to the femur and connects the lateral condyle of the femur to the anterior tibia. Both the PCL and ACL are intracapsular ligaments contained within the joint space of the knee; they are, however, isolated from the synovial space by a synovial membrane. Along with the lateral and medial collateral ligaments, they make up the four main structural ligamentous supports of the joint. They are commonly injured in athletes. The PCL specifically is commonly injured with posteriorly directed force applied to the tibia that creates rapid tension within the ligament causing rupture (such as a knee strike against the dashboard of a car in rapid deceleration during an accident). Diagnosis of an injury to the PCL is through physical examination, which may disclose excess posterior translation of the tibia with a posteriorly directed force compared to the contralateral, intact side, and with imaging that shows discontinuity of the ligament. An MRI is the study of choice. Treatment includes physical therapy, as some PCL injuries may heal without surgery. In cases of severe pain, ongoing gait impairment, or instability, surgical correction may be required. Incorrect Answers: A, B, C, and D. Choice A identifies the subcutaneous fat, which plays no significant role in joint stability. Choice B is the anterior aspect of the medial femoral condyle. It is the attachment site of the PCL. Choice C is the patellar ligament. It connects the pate

12 Exam Section 1: Item 12 of 50 National Board of Medical Examiners® Comprehensive Basic Science Self-Assessment 12. An investigator is conducting a study of the effects of a newly discovered spider venom on neuromuscular transmission. An isolated nerve muscle preparation is bathed in a solution containing the venom; there is a decrease in the end-plate potential amplitude following stimulation of the nerve. The presence of the venom does not change the amplitude of the nerve action potential or the muscle potential change in response to the exogenous application of acetylcholine to the neuromuscular junction. Blockade of which of the following by the venom best explains the decreased end-plate potential amplitude in this study? A) Chloride conductance in the nerve terminal B) Function of acetylcholinesterase C) Inactivation of voltage-gated sodium channels in the muscle D) Insertion of acetylcholine receptors into

E. The blockade of presynaptic, voltage-gated calcium channels would lead to decreased end-plate potential amplitude. Normally, action potentials lead to depolarization in the terminal bouton of the axon. Presynaptic, voltage-gated calcium channels consequently open and allow calcium influx, which triggers the exocytosis of acetylcholine-filled vesicles into the synaptic cleft. Acetylcholine diffuses across the synaptic cleft and binds nicotinic acetylcholine receptors (NÁCHRS) on the skeletal muscle cell membrane. The bound NACHRS allow sodium influx (and a lesser degree of potassium efflux), leading to depolarization of the postsynaptic membrane potential. Once the threshold membrane potential is reached, voltage-gated sodium channels open and the depolarization propagates down the postsynaptic membrane, representing the muscle action potential. Each NACHR is associated with its own end-plate potential, or voltage change, that results from the binding of acetylcholine. The summation of the end-plate potentials represents the membrane potential. Decreased synaptic acetylcholine concentrations result in decreased end-plate potential amplitudes. Incorrect Answers: A, B, C, and D. Blockade of chloride conductance in the nerve terminal (Choice A) would lead to decreased chloride influx and a more positive presynaptic membrane potential. Consequently, more voltage-gated calcium channels would open, leading to calcium influx and increased release of acetylcholine vesicles. The amplitude of end-plate potentials would increase. Blockade of the function of acetylcholinesterase (Choice B) leads to decreased degradation of synaptic acetylcholine and is the mechanism of medications (eg, pyridostigmine) utilized for myasthenia gravis. An increased synaptic acetylcholine concentration would increase the amplitude of end-plate pote

13 Exam Section 1: Item 13 of 50 National Board of Medical Examiners® Comprehensive Basic Science Self-Assessment 13. A 38-year-old woman with asthma comes to the physician for advice concerning contraception. She is sexually active with one partner, and they use condoms inconsistently. Current medications include inhalational fluticasone and albuterol. She has smoked one-half pack of cigarettes daily for 20 years and drinks two alcoholic beverages daily. She does not use illicit drugs. Her maternal grandmother developed breast cancer at the age of 58 years. She is 157 cm (5 ft 2 in) tall and weighs 82 kg (180 lb); BMI is 33 kg/m2 Her vital signs are within normal limits. Physical examination shows no other abnormalities. This patient should be advised to avoid the use of an oral contraceptive because of which of the following historical factors? A) Age B) Alcohol use C) Asthma D) Family history E) Obesity F) Tobacc

F. Combined oral contraceptive pills (OCPS) contain a combination of estrogen and progesterone. They are primarily used for contraceptive purposes but can also be used in a variety of gynecologic disorders, such as polycystic ovarian syndrome (PCOS), menorrhagia, and endometriosis. A sustained release of progestin and estrogen prevents ovulation and causes thickening of the cervical mucus along with thinning of the endometrial layer by inhibiting the release of gonadotropin-releasing hormone, follicle- stimulating hormone, and luteinizing hormone. OCPS are typically administered as a daily pill for three weeks, followed by a daily placebo pill for one week, during which time withdrawal bleeding occurs. They are associated with increased hypercoagulability and are therefore contraindicated in patients with a history of deep venous thrombosis, pulmonary embolism, stroke, or myocardial infarction. They are also contraindicated in patients who are 35 years of age or older and concomitantly smoke 15 or more cigarettes per day, along with patients diagnosed with hypertension, migraine with aura, severe cirrhosis, and breast cancer. Common adverse effects include nausea, breast tenderness, irregular menstrual periods, and a mildly delayed return to fertility following cessation. OCPS are contraindicated in this patient caused by her tobacco use. Incorrect Answers: A, B, C, D, and E. Age (Choice A) can increase the risks associated with OCPS, as there is an increased baseline risk for cardiovascular disease. However, this patient is only 38 years old, and age alone is not a contraindication to OCP use without smoking history. Alcohol use (Choice B) can preclude the use of OCPS if severe and associated with consequent cirrhosis, as they are generally contraindicated in severe decompensated cirrhosis. However, alcohol use alone

29 Exam Section 1: Item 29 of 50 National Board of Medical Examiners® Comprehensive Basic Science Self-Assessment 29. A 28-year-old woman delivers a full-term newborn who has generally flattened facial features, including a broad, flat nose, and bilateral clubfoot deformity with tibial torsion. Ultrasonography at 18 weeks' gestation showed oligohydramnios and small dysplastic fetal kidneys with scattered cysts. Which of the following general classifications best describes the extrarenal physical features? A) Deformation B) Disruption C) Malformation D) Multifactorial E) Teratogenic effect

A. Deformation describes a disruption in organ morphogenesis that occurs after the embryonic period. It arises because of an extrinsic force that limits normal development. In this example, renal dysplasia has resulted in minimal fetal urine production, and therefore limited production of amniotic fluid (oligo- or anhydramnios) as fetal urine is a major contributor. Without the production of amniotic fluid to distribute force evenly over the surface of the developing fetus, deformities will result. Fetuses with renal agenesis or dysplasia typically exhibit compressed, flattened facial features, limb dysplasia, and pulmonary hypoplasia caused by inadequate amniotic fluid-induced lung expansion. This is known as the Potter sequence and is an example of an extrinsic influence on organ morphogenesis from an inadequate intrauterine environment. Incorrect Answers: B, C, D, and E. Disruption (Choice B) defines an error in organ morphogenesis that arises from the breakdown of an organ or body region that was developmentally normal. The classic example of disruption is the presence of amniotic bands that constrict, compress, or encircle the developing fetus, resulting in morphogenic abnormalities involving, for example, the limbs, penis, fingers, or toes. Malformation (Choice C) defines an intrinsic disruption in organ development, which generally occurs earlier in the embryonic period. Multifactorial (Choice D) defines the presence of many influences that result in organ morphogenic errors, an example of which could be a fetus exposed to alcohol in utero with resultant abnormal facial development that also experienced disruption from amniotic bands later in utero. Teratogenic effect (Choice E) describes the result of embryonic exposure to an external toxic substance (often a medication or illicit drug) that results in malforma

51 Exam Section 2: Item 1 of 50 National Board of Medical Examiners® Comprehensive Basic Science Self-Assessment 1. The graph shows the rate of breakdown of skeletal muscle glycogen by phosphorylase in the presence and absence of AMP. Based on the data, which of the following best describes the action of AMP? Vmax +AMP A) Allosteric activator No effector Vmax B) Allosteric inhibitor C) Catabolite activator D) Catabolite repressor [S] E) Covalent modifier of the enzyme F) Covalent modifier of phosphorylase kinase

A. Enzymes are kinetically categorized by their Michaelis constant, Km, which inversely reflects their affinity for their substrate, and by Vmax, the maximum catalysis rate of the enzyme. In other words, as Km decreases, affinity for the substrate increases. Km is defined as the substrate concentration at which the enzyme reaches one-half of its Vmax. In this case, the kinetic analysis of the patient's glycogen phosphorylase shows that the concentration of substrate [S] at one-half of Vmax decreases in the presence of AMP. Therefore, AMP must increase the affinity of the enzyme for its substrate, a phenomenon that classically results from a conformational change in the active site of the enzyme where substrate binds. Allosteric activators influence enzymes in exactly this manner - inducing conformational change at the active site which in turn increases binding affinity for the substrate. This is reflected by the decreased Km. AMP, a breakdown product of adenosine triphosphate (ATP), is generated in states of fasting. Glycogen, a storage form of glucose in liver and muscle, is broken down by glycogen phosphorylase to liberate free glucose molecules when glucose is depleted in fasting. AMP, when present in excess, signals a state of fasting. It upregulates glycogen degradation through glycogen phosphorylase by acting as an allosteric activator of the enzyme. Incorrect Answers: B, C, D, E, and F. Allosteric inhibitors (Choice B) modify enzymes in a manner that their affinity for their target substrate is reduced at the active site, not increased. Allosteric inhibitors would be reflected in enzyme kinetic analysis by an increase in the Km, indicating that a greater concentration of substrate would be required to elicit the same binding. Catabolite activator (Choice C) and catabolite repressor (Choice D) describe phenomena

57 Exam Section 2: Item 7 of 50 National Board of Medical Examiners® Comprehensive Basic Science Self-Assessment 7. A 12-year-old boy with seizure disorder is brought to the physician by his parents for a follow-up examination. His mother has two neurofibromas, and his brother and maternal uncle each have numerous café au lait spots. Physical examination shows numerous flat, irregular pigmented skin lesions. He is individual IV-2 in the pedigree shown. Which of the following is the most likely pattern of inheritance of this family's disorder? | || A) Autosomal dominant with variable expressivity B) Autosomal dominant with variable penetrance II C) Autosomal recessive with high heterozygote frequency D) Autosomal recessive with low heterozygote frequency IV E) X-linked dominant O F) X-linked recessive O Unaffected female Unaffected male Affected female Affected male

A. Neurofibromatosis 1 is an autosomal dominant neurocutaneous disorder that typically results from an inherited mutation of the NF1 tumor suppressor gene on chromosome 17. Neurofibromatosis 1 is inherited with complete penetrance and variable expressivity, signifying that all family members who inherit the mutation manifest the disease but may demonstrate different phenotypic presentations. Rarely, the mutation in the NF1 gene will occur sporadically instead. The disorder presents with cutaneous neurofibromas (benign neoplasms derived from neural crest cells), café au lait spots (flat, irregular pigmented lesions), pigmented iris hamartomas (Lisch nodules), optic gliomas (which may lead to seizures), and/or pheochromocytomas. The diagnosis is typically based on physical examination findings, but genetic testing may be utilized to confirm ambiguous cases. Treatment includes surgical debulking of symptomatic tumors, management of seizures and other neurologic complications, and genetic counseling. Incorrect Answers: B, C, D, E, and F. Autosomal dominant disorders with variable penetrance (Choice B) typically result in less than half of the progeny of an affected parent manifesting the disease, as not all offspring with the mutation will manifest the disease. This patient's pedigree instead shows that slightly more than half of the offspring of affected parents manifest the disease, which is more consistent with complete penetrance. Autosomal recessive inheritance with either high or low heterozygote frequency (Choices C and D) would necessitate that both parents of the affected offspring either have the disease or are heterozygous carriers. A high heterozygote frequency signifies a low homozygote frequency (ie, a low frequency of offspring with two mutated alleles) and therefore a low frequency of disease in the offspr

87 Exam Section 2: Item 37 of 50 National Board of Medical Examiners® Comprehensive Basic Science Self-Assessment 37. A51-year-old man develops diaphoresis, tachycardia, and a blood pressure of 155/100 mm Hg 24 hours after undergoing an abdominal operation. Two hours later, he has a generalized tonic-clonic seizure. Which of the following is most likely responsible for these adverse effects in this patient? A) Acute renal failure B) Alcohol withdrawal C) Anaphylactic reaction D) Narcotic pain medication E) Sepsis from a gram-positive organism

B. Alcohol withdrawal typically leads to sympathetic overdrive and, in severe cases, generalized seizures. Chronic alcohol use causes changes in the expression of different subunits of the N-methyl-d-aspartate (NMDA) and gamma-aminobutyric acid (GABA) receptors. As a result of these adaptations, the discontinuation of alcohol leads to sympathetic overdrive. Initially, this hyperexcitability results in tremors, anxiety, diaphoresis, hypertension, tachycardia, and nausea. Within hours of the last drink, alcoholic hallucinosis (auditory or visual hallucinations without confusion or autonomic instability) and seizures (caused by cortical hyperactivity) can occur. Delirium tremens, the most concerning and life-threatening complication of alcohol withdrawal, involves severe confusion, disorientation, fluctuations in consciousness, agitation, visual hallucinations, and autonomic instability (fluctuations in pulse and blood pressure with hyperthermia). Incorrect Answers: A, C, D, and E. Acute renal failure (Choice A) can lead to oliguria and may be associated with hyperkalemia, metabolic acidosis, and uremia. None of these changes typically cause seizures. Additionally, sympathetic overactivation would be uncommon. An anaphylactic reaction (Choice C) is an acute, life-threatening reaction to an allergen that leads to systemic mast cell degranulation. Anaphylactic reactions typically present with hypotension caused by increased vascular permeability from histamine. Furthermore, a seizure would be atypical in the presentation of an anaphylactic reaction. Narcotic pain medications (Choice D) cause symptoms of central nervous system depression including sedation, respiratory depression, bradycardia, and hypotension as opposed to this patient's tachycardia and hypertension. Sepsis from a gram-positive organism (Choice E) typically

99 Exam Section 2: Item 49 of 50 National Board of Medical Examiners® Comprehensive Basic Science Self-Assessment 49. A 60-year-old man is being evaluated for episodes of carpopedal spasm. Laboratory studies of serum show: 6.2 mg/dL 4.0 g/dL 8.6 mg/dL 200 pg/mL (N=10-65 pg/mL) Ca2+ Albumin Phosphorus Parathyroid hormone Which of the following conditions would most likely account for these findings? A) Acute tubular necrosis B) Chronic renal failure C) Dehydration D) Rapidly progressive glomerulonephritis E) Renal cell carcinoma

B. Chronic renal failure can have variable cause, such as with chronic prerenal disease in the setting of heart failure or cirrhosis with decreased renal perfusion, intrinsic renal disease such as nephrosclerosis or atherosclerotic renal artery disease, chronic nephritic or nephrotic glomerular disease, or chronic postrenal obstructive disease. Chronic kidney disease is more common in older patients after years of underlying kidney injury. Active vitamin D (1,25-dihydroxycholecalciferol) is synthesized through cutaneous, hepatic, and renal routes, with the final conversion of 25-hydroxycholecalciferol to active 1,25-dihydroxycholecalciferol in the kidney via 1-alpha hydroxylase. Patients with chronic kidney disease have diminished capacity for native vitamin D synthesis because of underlying renal parenchymal damage. The decreased production of active vitamin D results in impaired gastrointestinal uptake of dietary calcium, leading to hypocalcemia. This, in turn, leads to increased serum PTH and secondary hyperparathyroidism. Chronic renal failure is also characterized by the inability of the kidney to excrete phosphate, resulting in hyperphosphatemia. Hyperphosphatemia and hypocalcemia result in the upregulation of PTH, which increases bony turnover in an attempt to raise serum calcium. Incorrect Answers: A, C, D, and E. Acute tubular necrosis (Choice A) typically occurs following an ischemic or nephrotoxic insult to the kidneys, which results in necrosis of the tubular epithelium. Granular, muddy brown casts are common on urinalysis. It would not cause hypocalcemia. Dehydration (Choice C) can result in prerenal acute kidney injury but would not lead to the hypocalcemia and secondary hyperparathyroidism seen in chronic renal failure. Rapidly progressive glomerulonephritis (Choice D) is a nephritic syndrome characteriz

15 Exam Section 1: Item 15 of 50 National Board of Medical Examiners® Comprehensive Basic Science Self-Assessment 15. A 4-year-old boy has had a viral upper respiratory tract infection for the past 3 days. Clearance of the virus is most dependent on which of the following? A) Activation of macrophages B) Class I MHC-viral peptide complex presentation C) Class II MHC-viral peptide complex presentation D) Phagocytosis of viral particles by CD8+ T lymphocytes E) Production of memory B lymphocytes F) Proliferation of memory T lymphocytes G) Proliferation of plasma cells

B. Class I MHC-viral peptide complex presentation with subsequent activation of CD8+ T lymphocytes will most likely contribute to clearance of the virus. Viruses are obligate intracellular pathogens and must enter a cell to proliferate. Viral particles that are released from infected cells are taken up by professional antigen presenting cells (APCS) such as dendritic cells. These large proteins are shuttled to the proteasome, which generates smaller fragments of protein that are in turn shuttled to the endoplasmic reticulum (ER). In the ER, these peptides are attached to class I MHC molecules before being transferred to the Golgi apparatus and then to the cytoplasmic membrane where they are expressed. Viral particles may also be presented on class I MHC molecules on the surface of infected cells, although these cells often lack the costimulatory capabilities of APCS. CD8+ T lymphocytes possess unique T-cell receptors (TCRS) that recognize particular viral antigens, and binding of the TCR to the class I MHC-antigen complex activates the CD8+ T lymphocytes. In turn, they release cytokines such as interferon-y and tumor necrosis factor; they also interact with cell surface Fas and FasL to induce apoptosis of infected cells. They can release granzyme and perforin, which cause membrane pore formation and lead to apoptosis of infected cells. Incorrect Answers: A, C, D, E, F, and G. Activation of macrophages (Choice A) occurs primarily through IL-12/IFN-y signaling. Activated macrophages can become histiocytes and may form granulomas in the case of some infections (eg, Mycobacterium tuberculosis). Class II MHC-viral peptide complex presentation (Choice C) activates CD4+ T lymphocytes, which assist B lymphocytes in making antibodies, and recruit macrophages, CD8+ T lymphocytes, and leukocytes to the site of activation. While t

11 Exam Section 1: Item 11 of 50 National Board of Medical Examiners® Comprehensive Basic Science Self-Assessment 11. A 26-year-old man comes to the physician because of fever, cough, chest pain, and malaise for 2 weeks. He moved to central California 6 months ago. A complete blood count shows mild eosinophilia. A chest x-ray shows patchy bronchopneumonia. Culture of the sputum grows a mold with the morphology shown in the photomicrograph, with the arrow indicating the infectious particle. Which of the following is the most likely causal organism? A) Actinomyces israelii B) Coccidioides immitis C) Histoplasma capsulatum D) Legionella pneumophila E) Mycobacterium tuberculosis OF) Nocardia brasiliensis G) Staphylococcus aureus H) Streptococcus pneumoniae

B. Coccidioides immitis is an endemic fungus of the Southwestern United States and central valley of California that typically causes coccidioidomycosis, a self-limited respiratory illness. Signs and symptoms include fever, fatigue, cough, arthralgia, and myalgia. Patients may also present with erythema nodosum. Coccidioidomycosis can potentially present with disseminated disease, especially in immunocompromised patients, and cause infections of the skin, bone, and central nervous system. Silver stain of sputum or tissue biopsy demonstrates large spherules containing endospores. Diagnosis can be facilitated with enzyme-linked immunosorbent assay (ELISA) testing and be confirmed with polymerase chain reaction. Treatment is with oral or intravenous antifungals, including agents from the azole or polyene classes. Incorrect Answers: A, C, D, E, F, G, and H. Actinomyces israelii (Choice A) is a Gram-positive, anaerobic bacillus that forms branching filaments and yellow granules. It typically causes abscesses of the head and neck, often with draining fistulous tracts. Histoplasma capsulatum (Choice C) is a fungus native to the Ohio river and Mississippi river valleys that can cause pneumonia and is spread by the droppings of birds or bats. It is identifiable as oval yeasts within macrophages. Legionella pneumophila (Choice D) is a Gram-negative bacillus that is transmitted primarily through aerosols from water sources. It causes Legionnaire disease, characterized by fever, pneumonia, and gastrointestinal symptoms. It can be identified by positive staining with silver stains and its fastidious nature, requiring culture on charcoal yeast extract with supplemental iron and cysteine. Mycobacterium tuberculosis (Choice E) is an acid-fast bacillus that causes tuberculosis. It is identifiable by its thick, waxy capsule, positive ac

2 Exam Section 1: Item 2 of 50 National Board of Medical Examiners® Comprehensive Basic Science Self-Assessment 2. During an experiment, a solution of mixed fatty acids is injected into the duodenum of an experimental animal. Under these conditions, the clearance rate of an intravenous glucose load from the circulation is doubled. In contrast, an injection of an equal volume amount of 0.9% saline into the duodenum has much less effect on the plasma clearance rate of glucose. These findings are most likely caused by the secretion of which of the following hormones? A) Gastrin B) Glucose-dependent insulinotropic peptide C) Motilin D) Secretin E) Somatostatin

B. Glucose-dependent insulinotropic peptide (GIP) is secreted by K cells in the duodenum and jejunum and functions to decrease gastric acid production and stimulate insulin release from the pancreas. Its secretion is promoted by the presence of fatty acids, amino acids, and intestinal glucose. While serum glucose also stimulates insulin secretion by the pancreas, the effect of intraluminal glucose on GIP and subsequent insulin secretion leads to increased concentrations of insulin compared to parenteral glucose administration. Insulin promotes peripheral tissue uptake of glucose, glycolysis, glycogen synthesis, protein synthesis, and fatty acid synthesis, resulting in decreased glucose concentration in the serum. Incorrect Answers: A, C, D, and E. Gastrin (Choice A) is produced by G cells in the gastric antrum and stimulates parietal cells within the gastric body to produce hydrochloric acid. Gastric acid has no effect on serum glucose concentration. Motilin (Choice C) is secreted by the small intestine and stimulates intestinal peristalsis. Motilin receptors are targeted by erythromycin and metoclopramide, used therapeutically in gastroparesis. Secretin (Choice D) is produced by duodenal S cells. It promotes the release of bicarbonate-rich pancreatic secretions and bile and inhibits gastric acid production. Somatostatin (Choice E) is a regulatory peptide secreted by D cells of the pancreas and gastrointestinal mucosa that inhibits gastric acid and pepsinogen secretion, gallbladder contraction, and insulin and glucagon release. Somatostatin would have an indirect effect on glucose through counterregulatory action of both insulin and glucagon. Educational Objective: Glucose-dependent insulinotropic peptide (GIP) is secreted by K cells in the duodenum and jejunum, and it functions to decrease gastric acid production and

18 Exam Section 1: Item 18 of 50 National Board of Medical Examiners® Comprehensive Basic Science Self-Assessment 18. A 28-year-old woman has had fatigue and heavy menstrual periods during the past year. She is a vegetarian who eats eggs and dairy products. Her diet contains little fruit. Which of the following erythrocyte types is most likely to be present on a peripheral blood smear from this patient? OA) Macrocytes B) Microcytes C) Ovalocytes D) Spherocytes E) Stippled cells

B. Microcytes are most likely to be present in this patient's peripheral blood smear, which is suggestive of iron deficiency anemia (IDA) in the setting of heavy menstrual periods and low dietary iron intake (vegetarianism). Erythrocytes in the setting of IDA are pale (hypochromic) and small in size (microcytic) but normal in shape. Iron is required for the synthesis of heme, which is a necessary component of the hemoglobin molecule, and thus, of erythrocytes. It functions to shuttle oxygen to and from peripheral tissues. In individuals who do not have an adequate intake of dietary iron in the form of heme obtained from animal meat, especially when other causes of ongoing blood loss such as heavy menstruation are present, deficiency can develop. IDA may also develop as the result of chronic blood loss from colorectal bleeding, as a result of malabsorption syndromes (eg, celiac disease), and in patients who have undergone gastric bypass surgery. Erythrocytes on the peripheral blood smear are hypochromic and microcytic as a result of deficient hemoglobin concentration. It is hypothesized that erythrocytes are microcytic as a result of continuing erythrocyte division in order to reach an adequate hemoglobin concentration; because hemoglobin stores are inadequate, cell division continues beyond what would normally occur and causes the cells to be smaller than normal. Treatment for this patient would include management of her menstrual bleeding and oral iron supplementation. Incorrect Answers: A, C, D, and E. Macrocytes (Choice A) refer to larger than normal erythrocytes and are seen in the setting of folate or vitamin B 12 deficiency. This patient has chronic blood loss most likely leading to iron deficiency and is likely to have adequate stores of folate and vitamin B 12 which are consumed in high amounts in most vegetari

56 Exam Section 2: Item 6 of 50 National Board of Medical Examiners® Comprehensive Basic Science Self-Assessment 6. A 70-year-old woman comes to the physician for a routine pelvic examination. During speculum examination of the vagina and cervix, the Valsalva maneuver causes a bulge of the anterior vaginal wall. Which of the following is the most likely cause of this finding? A) Bartholin gland cyst B) Cystocele C) Obturator hernia D) Rectocele E) Uterine prolapse

B. Normal pelvic support is caused by the action of several pelvic muscles and ligaments of the pelvic floor, including the levator muscles, urogenital diaphragm, perineal body, endopelvic fascia, uterosacral and cardinal ligaments, anal sphincter, and urethral sphincter. Defects in this pelvic support may result in urogenital prolapse, which is caused by the loss of connective tissue and muscular support of the uterus, paravaginal tissue, bladder wall, urethra, or distal rectum. Risk factors for pelvic support defects include collagen and connective tissue disorders, previous vaginal delivery, menopause, prior pelvic surgery, and increased intra-abdominal pressure (eg, obesity and chronic constipation). Signs and symptoms include a vaginal mass, vaginal pressure, perineal discomfort, and urinary or fecal retention or incontinence. In the case of a cystocele, the bladder prolapses into the anterior vaginal wall. It is associated with stress urinary incontinence, as well as urinary retention. Physical examination will show a bulge of the anterior vaginal wall, as in this patient. Management for a symptomatic cystocele may include the insertion of a vaginal pessary, which is a mechanical device that provides pelvic support, along with pelvic floor exercises or surgical correction. Incorrect Answers: A, C, D, andE. A Bartholin gland cyst (Choice A) appears as a swollen, fluctuant mass at the posterior labium majora caused by obstruction of Bartholin gland outflow. If left untreated, a cyst can progress to an abscess, with the development of erythema and tenderness to palpation. It does not cause a mass on the anterior vaginal wall. An obturator hernia (Choice C) is a herniation of the abdominal contents through the obturator foramen, along with the obturator vessels and nerve. It presents with abdominal pain and a potenti

67 Exam Section 2: Item 17 of 50 National Board of Medical Examiners® Comprehensive Basic Science Self-Assessment 17. A 54-year-old man comes to the physician because of a 2-week history of burning epigastric pain. He is recently divorced. His diet mainly consists of fast food. He smokes cigars and drinks one to two 12-ounce cans of beer on the weekends. He was treated for Helicobacter pylori infection 1 year ago. He uses high doses (600 mg daily) of over-the-counter ibuprofen for chronic joint pain. Abdominal examination shows tenderness to deep palpation of the epigastric area. A peptic ulcer is suspected. Which of the following is the strongest predisposing risk factor for this patient's condition? A) High-fat diet B) Ibuprofen use C) Ingestion of alcohol D) Recent psychosocial stressor E) Smoking cigars

B. Peptic ulcer disease describes the presence of ulcers in the stomach or duodenum, which classically present with worsening abdominal pain related to consumption (gastric) or lack of consumption (duodenal) of food. The most common cause of peptic ulcer disease is infection with the bacterium Helicobacter pylori, which accounts for nearly all duodenal ulcers. Gastric ulcers can be caused by prolonged or excess usage of nonsteroidal anti-inflammatory drugs (NSAIDS), such as ibuprofen, which inhibit cyclooxygenase in the gastrointestinal tract, leading to a reduction of prostaglandin secretion and decreased protection of the gastric mucosa. Persistent inflammation related to a peptic ulcer can result in complications including fibrosis, stricture, hemorrhage, and perforation. Proton pump inhibitor therapy is first line in the management of gastroesophageal reflux and peptic ulcer disease along with eradication of H. pylori and cessation of inciting factors (eg, smoking, alcohol, NSAIDS). Incorrect Answers: A, C, D, and E. High-fat diet (Choice A) can be related to biliary colic or hepatobiliary pathology. Biliary colic is characterized by epigastric or right upper quadrant pain after ingestion of a fatty meal promoting gallbladder contraction. Ingestion of alcohol (Choice C) in excessive amounts can cause damage to the gastric mucosal barrier. This patient drinks a moderate amount of alcohol occasionally, making it unlikely to be the strongest cause of his peptic ulcer disease. Recent psychosocial stressor (Choice D) has been associated to a limited extent with increased risk for peptic ulcer disease. However, NSAID use has a stronger association with the development of peptic ulcer disease. Smoking cigars (Choice E) has been associated with peptic ulcer disease. Risk for peptic ulcer disease increases accordingly with

6 Exam Section 1: Item 6 of 50 National Board of Medical Examiners® Comprehensive Basic Science Self-Assessment 6. A 33-year-old woman comes to the physician because of a 3-day history of nausea and light-headedness. Her last menstrual period was 5 weeks ago. She is apprehensive. Physical examination shows no abnormalities. Her serum B-human chorionic gonadotropin concentration is increased. At this stage in the pregnancy, progesterone is most likely produced within which of the following structures? A) Corpus albicans B) Corpus luteum C) Pituitary gland D) Placental cytotrophoblast E) Placental syncytiotrophoblast

B. Pregnancy is suspected when there is a missed or delayed menstrual period. During a normal menstrual period, follicle-stimulating hormone and luteinizing hormone (FSH and LH, respectively) concentrations increase and stimulate the developing follicle. The follicle produces estrogen, which leads to proliferation of the endometrium in preparation for implantation of a fertilized ovum. As estrogen rises, a surge occurs, which in turn stimulates a surge in LH that causes ovulation. Immediately following ovulation, the corpus luteum forms. The corpus luteum secretes progesterone to maintain the endometrial lining. If no implantation occurs, the corpus luteum degrades to the corpus albicans and estrogen and progesterone concentrations decrease, causing menstruation. However, during pregnancy, the placenta develops from embryo implantation. The placenta then begins to secrete human chorionic gonadotropin, which acts to maintain the corpus luteum and its secretion of progesterone, which is necessary for maintenance of the pregnancy. As the placenta develops, it becomes primarily responsible for progesterone production around seven to ten weeks of gestation. As this patient is at five weeks of gestation, her progesterone production is still primarily performed by the corpus luteum. Incorrect Answers: A, C, D, andE. The corpus albicans (Choice A) is the degraded corpus luteum that develops because of the absence of embryo implantation. It does not secrete hormones. The pituitary gland (Choice C) is responsible for secreting FSH and LH, as well as prolactin and thyroid-stimulating hormone. While FSH and LH play a role in the development of an ovarian follicle, the pituitary gland does not directly produce progesterone. Placental cytotrophoblast (Choice D) is the inner layer of the chorion and is vital for the implantation of a

7 Exam Section 1: Item 7 of 50 National Board of Medical Examiners® Comprehensive Basic Science Self-Assessment 7. A study is done to determine the relationship between use of oral contraceptives and cervical cancer. Study subjects include 50,000 women who are using oral contraceptives and 50,000 women who have had a tubal ligation. After 2 years of follow-up, the rate of in situ cervical cancer is 18 per 10,000 in the oral contraceptive group and 3 per 10,000 (p < 0.05) in the tubal ligation group. Which of the following is the estimated relative risk of cervical cancer among women who have had a tubal ligation compared with women who use oral contraceptives? A) 3 - 50,000 = 0.00006 B) 3 18 = 0.17 C) 18 - 3 = 6 D) 18 - 3 +10 = 17.7 E) 18 - 3 = 15 F) 18 + 3 = 21 G) Indeterminable from the data given

B. Relative risk (RR) describes the difference in likelihood of the occurrence of a particular disease outcome between two groups of patients with or without a particular exposure. In this case, the outcome of cervical cancer in situ is compared between two exposures, the use of oral contraceptives or the use of tubal ligation for contraception. Calculations of relative risk are commonly performed in cohort studies. RR is calculated by dividing the fraction of patients with a positive exposure and who developed disease (a) amongst all patients who were exposed which includes those exposed who did not develop disease (b), (a + b), by the fraction of patients with a negative exposure and who developed disease (c) amongst all patients who were not exposed which includes those who did not develop disease (d), (c + d). RR thus equals (a / (a + b) / (c/ (c + d)). RR values greater than 1.0 indicate an increased risk for developing disease in association with the exposure, whereas values less than 1.0 indicate a reduced risk for developing disease, and RR equal to 1.0 indicates that the disease outcome and the exposure are not related. In this case, RR of cervical cancer in situ among women who have had a tubal ligation compared with women who use oral contraceptives is calculated as (3/ 10,000) - (18/10,000) = 3 18 = 0.17. %3D Incorrect Answers: A, C, D, E, F, and G. 3 - 50,000 = 0.00006 (Choice A) is an incorrect computation of the total fraction of women with a tubal ligation who developed cervical cancer in situ, as the data are presented per 10,000 patients. Therefore, if the fraction of women with a tubal ligation who developed cervical cancer in situ were to be calculated, it would be 3/10,000 persons or 15/50,000 in absolute numbers. 18 3 = 6 (Choice C) computes the RR of developing of cervical cancer in situ among wo

8 Exam Section 1: Item 8 of 50 National Board of Medical Examiners® Comprehensive Basic Science Self-Assessment 8. A study is conducted to assess the accuracy of a new rapid test to detect a virulent bacterial infection. This infection has an 80% mortality rate if it is not identified early in its course; however, prompt administration of antibiotics decreases the mortality rate to less than 5%. The risks of this antibiotic therapy are minimal. A total of 10,000 participants are enrolled and undergo assessment with the new test. The graph shows the distribution of infected and noninfected participants according to the results of the test. Which of the following labeled points is most appropriate for use as the optimal diagnostic cut point for results of this test? Not Infected infected A B C D E A) B) C) D) E)

B. The cut point of the test should be set such that all persons with the infection result as positive so that there are few false negatives. The threshold should be set to maximize sensitivity, which equals the true positive test results divided by the sum of true positive and false negative test results. Therefore, as sensitivity approaches 1.0, all patients who have the disease will be detected by the test (there will be no false negatives). In this example, high sensitivity is critical, as the disease has an 80% mortality rate if not detected and treated early. Therefore, a priority should be placed on sensitivity when deciding the cut point threshold for positive and negative results. Specificity is calculated by the true negative test results divided by the sum of true negative and false positive results. Increasing sensitivity comes at the expense of specificity; by setting the cut point to include all persons with the disease, many false positives will be introduced as persons without the disease have results above the cut point. Patients testing negative on a sensitive test can be safely considered disease free, whereas patients who test positive should receive additional diagnostic evaluation or treatment. In this example, a bimodal distribution of patients with and without disease is presented. All patients without disease are included in the area beneath the not-infected curve, while all patients with disease are included in the area beneath the infected curve. There is overlap between the two, suggesting that some patients will test equivalently on the test but may or may not have the disease. Setting the cut point to point B would permit all patients within the area under the infected curve to be identified as positive, which maximizes sensitivity, while minimizing the number of false positives, and there

5 Exam Section 1: Item 5 of 50 National Board of Medical Examiners® Comprehensive Basic Science Self-Assessment 5. A 76-year-old woman comes to the emergency department because of a 6-hour history of moderate right low back and flank pain. Vital signs are within normal limits. Physical examination shows mild vertebromediastinal recess (costovertebral angle) tenderness. Urinalysis shows no blood. A CT scan of the pelvis shows hydronephrosis with obstruction of the right ureter caused by external compression at the pelvic brim from a vascular structure. This patient most likely has an aneurysm of which of the following arteries? A) Abdominal aorta B) Common iliac C) Femoral D) Inferior mesenteric E) Renal

B. The ureter courses from the renal pelvis to the urinary bladder. It initially descends through the lower abdomen anterior and medial to the psoas major. It then enters the pelvis and crosses immediately anterior to the bifurcation of the common iliac artery, before continuing its descent anterior to the internal iliac artery. The ureters are commonly constricted at three locations: at the ureteropelvic junction, at the crossing point over the common iliac artery, and at the vesicoureteral junction at the entrance to the bladder. The ureter may be constricted by a vascular aneurysm of the common iliac artery and, additionally, renal calculi tend to lodge at these three locations. Ureteral obstruction leads to hydroureteronephrosis and possible postrenal kidney injury. Incorrect Answers: A, C, D, and E. The abdominal aorta (Choice A) is located medial to the ureters, just anterolateral to the vertebral bodies. It bifurcates into the two common iliac arteries and does not come in direct proximity to either ureter. Abdominal aortic aneurysms are usually not of sufficient size to cause ureteral obstruction. The femoral artery (Choice C) is found in the thigh as a continuation of the external iliac artery. It does not come in proximity to the ureters. Femoral arterial aneurysms occur but are uncommon. The inferior mesenteric artery (Choice D) originates from the abdominal aorta near the level of the third lumbar vertebra and provides arterial supply to the distal colon. As it is a midline structure, it does not come in close proximity to the ureters. The renal artery (Choice E) supplies blood to the kidney and the proximal portion of the ureter. While aneurysms of the renal artery may form, they are rare and are unlikely to obstruct the ureter. Educational Objective: The ureter is commonly constricted at three sites: at t

37 Exam Section 1: Item 37 of 50 National Board of Medical Examiners® Comprehensive Basic Science Self-Assessment 37. A previously healthy 18-year-old man is brought to the emergency department by his mother 45 minutes after she found him crouched in his closet, saying he had to hide from the intruders "from that other place." There were no intruders in the house. He has no history of alcohol or drug use. He appears afraid of the physician and sits far away from her. During the interview, he says he has heard two male voices in his head for the past 2 weeks. Physical examination and laboratory studies show no abnormalities. Which of the following is the most likely diagnosis? A) Bipolar disorder B) Brief psychotic disorder C) Delusional disorder D) Schizoaffective disorder E) Schizophrenia

B. This patient meets the diagnostic criteria for brief psychotic disorder. Brief psychotic disorder is characterized by the acute onset of one or more psychotic symptoms (eg, delusions, hallucinations, disorganized speech, disorganized behavior) lasting less than one month. This patient is experiencing a paranoid delusion, disorganized behavior (crouching in his closet despite the absence of an intruder), and auditory hallucinations. Risk factors for brief psychotic disorder include stressful life events and personality disorders. Treatment may include hospitalization and antipsychotic treatment depending on symptom severity and functional impairment, though some symptoms and the presentation overall may resolve without medications. Incorrect Answers: A, C, D, and E. Bipolar disorder (Choice A) is characterized by discrete episodes of depression and mania, which can sometimes be accompanied by psychotic symptoms such as hallucinations and delusions. This patient demonstrates psychotic symptoms without evidence of mood symptoms, which makes a primary psychotic disorder such as brief psychotic disorder more likely. Delusional disorder (Choice C) features one or more delusions for a month or longer without other psychotic symptoms. This patient does not meet the duration criterion for delusional disorder and demonstrates disorganized behavior and auditory hallucinations, which are atypical of delusional disorder. Patients with chronic psychotic disorders such as schizoaffective disorder (Choice D) or schizophrenia (Choice E) demonstrate at least two of the following five symptoms: delusions, hallucinations, disorganized speech, disorganized behavior, or negative symptoms (eg, flat affect, apathy, alogia). Many patients also illustrate prodromal periods of strange behavior and decreased functioning. Patients with schizoaf

34 Exam Section 1: Item 34 of 50 National Board of Medical Examiners® Comprehensive Basic Science Self-Assessment 34. An 83-year-old woman is brought to the physician by her daughter to discuss the results of a complete dementia work-up. The patient has had mild memory impairment for 8 months. She takes no medications. Vital signs are normal. Her Mini-Mental State Examination score is 23/30. A rapid plasma reagin is 1:4, and a microhemagglutination assay for Treponema pallidum is positive. Which of the following is the best next step for the physician? A) Discussion of the diagnosis with the daughter privately B) Discussion of the diagnosis with the patient privately C) Disregarding the results since the patient is too old for treatment D) Repeated tests E) Lumbar puncture

B. This patient's syphilis diagnosis should be discussed with her privately. All patients, including those with dementia, should be assumed to possess decisional capacity to make medical decisions unless the physician determines the patient does not possess decisional capacity. This physician should first discuss this sensitive diagnosis with the patient privately and assess her decisional capacity surrounding the diagnosis. Patients who are deemed to lack decisional capacity should still be informed of the diagnosis, which may give the patient a partial understanding of their situation and relieve potential distress about the diagnostic uncertainty. The next-of-kin or designated medical decision maker of a patient without decisional capacity should also be informed after the diagnosis is discussed with the patient. Neurosyphilis refers to the spread of syphilis to the central nervous system. Neurosyphilis can manifest as meningitis, a progressive dementia syndrome known as general paresis, or disease of the posterior columns of the spinal cord known as tabes dorsalis. Diagnosis involves confirming the diagnosis of syphilis with both treponemal and nontreponemal (eg, rapid plasma reagin and VDRL) serum testing followed by examining the cerebrospinal fluid (CSF) and performing CSF treponemal or nontreponemal tests. Treatment of neurosyphilis is crucial to prevent neurologic progression and requires the administration of penicillin G. Incorrect Answers: A, C, D, and E. Rather than discuss this sensitive diagnosis with the patient's daughter (Choice A), the physician should honor this patient's autonomy by first discussing the diagnosis with the patient. After this discussion, if the physician believes that the patient lacks decisional capacity and the daughter is the next-of-kin or a designated medical decision-maker, th

48 Exam Section 1: Item 48 of 50 National Board of Medical Examiners® Comprehensive Basic Science Self-Assessment 48. The local health department is investigating an outbreak of acute hepatitis A in two city districts by reviewing the medical records at a health center in each district. The investigators plan to analyze the clinical and epidemiologic characteristics of patients who tested positive for hepatitis A serum IgM antibody compared with characteristics of those who tested negative for the antibody. Which of the following best describes this study design? A) Case series B) Case-control study C) Prospective study D) Randomized trial E) Retrospective cohort

B. This study compares one group of patients with an outcome under study (seropositive for Hepatitis A IgM antibody) (cases) against a second matched group without that outcome (controls) and identifies the associated exposure within each group. This study design is known as a case-control study. Case-control studies can be conducted in a prospective or retrospective manner but are always observational studies. By grouping patients by outcome and comparing differences in the odds of exposure, case-control studies can detect associations between exposure and outcome, such as exposure to certain risk factors and outcome of contracting hepatitis A as in this study. This is described statistically as an odds ratio (OR). Two unrelated variables will have an OR of 1.0, whereas positive association between an exposure and an outcome will have an OR greater than 1.0 and negatively associated variables will have an OR less than 1.0. Case-control studies are therefore capable of establishing association between exposure and outcome, but they do not establish causality. Incorrect Answers: A, C, D, and E. A case series study (Choice A) is a descriptive study that describes the history, possible exposures, and clinical findings of a group of patients with a similar diagnosis. Case series are non-analytic studies. They do not test a hypothesis and do not generally contain a control group. A prospective study (Choice C) is one in which one group of patients experiences an intervention or exposure and an associated control group does not. The two groups are followed and the desired outcome tracked. A randomized trial (Choice D) is a stringent type of prospective study wherein patients are randomly assigned to receive a particular intervention. The intervention may be compared against placebo therapy or against standard therapy, depend

68 Exam Section 2: Item 18 of 50 National Board of Medical Examiners® Comprehensive Basic Science Self-Assessment 18. A 27-year-old woman delivers monozygotic twins at 34 weeks' gestation. The larger twin has a hematocrit of 68%; the smaller twin is pale and has a hematocrit of 25%. Which of the following is the most likely explanation for these findings? A) Amniotic fluid leak across intervening membranes B) Artery-to-artery chorionic surface anastomoses C) Chronic abruptio placentae D) Funisitis E) Knotting of the umbilical cords F) Oligohydramnios

B. Twin-twin transfusion syndrome (TTTS) and twin anemia polycythemia sequence (TAPS) are complications of monochorionic twin gestation. TTTS occurs because of the formation of arteriovenous anastomoses in the chorion of the placenta that allow blood to pass from one fetus to the other. Less commonly, it can also involve the formation of artery-to-artery chorionic surface anastomoses. It typically presents on prenatal ultrasound with unequal amniotic fluid indices between the two amniotic sacs. It can also present with anemia of one fetus and polycythemia of the other fetus when chronic, which is referred to as TAPS. Monochorionic twin gestations are typically monitored with serial ultrasounds to watch for the development of these conditions, as they have a high morbidity and mortality. Ultrasound findings also include discrepancies in nuchal translucency and crown-rump length, and abnormal ductus venosus flow. Inequalities in amniotic fluid distribution are caused by relative hypovolemia of one fetus, with resultant activation of the renin-angiotensin-aldosterone system and consequent oliguria. In contrast, the hypervolemia of the other twin causes release of atrial natriuretic peptide, which results in diuresis and relative increases in the amniotic fluid index. Complications of this syndrome also include congenital anatomic abnormalities, hydrops fetalis, heart failure, and growth restriction. Options for management include laser ablation of the anastomotic vessels, amnioreduction, and/or selective fetal reduction. Incorrect Answers: A, C, D, E, and F. Amniotic fluid leak across intervening membranes (Choice A) could lead to oligohydramnios in one fetus and polyhydramnios in the other fetus if the movement of fluid was unidirectional. However, movement of amniotic fluid from one fetus to another would not cause disc

22 Exam Section 1: Item 22 of 50 National Board of Medical Examiners® Comprehensive Basic Science Self-Assessment 22. A 3-month-old girl is brought to the physician 2 days after the mother felt masses in the infant's groin while bathing her. Physical examination shows bilateral inguinal hernias and a shallow vagina that ends blindly. Serum studies show increased concentrations of luteinizing hormone, follicle-stimulating hormone, testosterone, and dihydrotestosterone. During repair of the hernias, gonads are found. A biopsy specimen of the gonads shows bilateral testes. The karyotype is 46,XY. Ultrasonography shows no uterus or fallopian tubes. Which of the following is the most likely cause of the findings in this patient? A) Decreased 21-hydroxylase activity B) Decreased 5a-reductase activity C) Defective androgen receptor D) Failure of testis to secrete anti-müllerian hormone E) Mutation of the sex-determining r

C. Androgen insensitivity syndrome is caused by a defect in the androgen receptor complex resulting in a genotypic XY male to develop external female or ambiguous genitalia and female secondary sexual characteristics. Testes are present and produce testosterone normally, but absence of a functioning androgen receptor prevents hormone binding and thereby prevents the development of male sexual characteristics. Patients present with female external genitalia, scant pubic and axillary hair, absent uterus and fallopian tubes, and a rudimentary vagina. Results of laboratory studies show increased concentrations of testosterone, estrogen, and luteinizing hormone. Menses will not occur because of the lack of cycled progesterone and estrogen, and the lack of a functional uterus with endometrial lining. Incorrect Answers: A, B, D, and E. Decreased 21-hydroxylase activity (Choice A) occurs in the setting of congenital adrenal hyperplasia, of which the most common form is 21-hydroxylase deficiency. Lack of this enzyme prevents production of aldosterone and cortisol, and results in excessive androgen production. Genetically female patients present with hypoaldosteronism as well as virilization during infancy, and genetically male patients present with precocious puberty in childhood. Decreased 5a-reductase activity (Choice B) results in the insufficient conversion of testosterone to dihydrotestosterone (DHT), leading to decreased concentrations of DHT and impaired virilization of the male urogenital tract. Individuals with 5a-reductase deficiency exhibit either female or ambiguous external genitalia, although demonstrate normal male internal genitalia because of normal concentrations of testosterone. Increased concentrations of DHT, as in this infant, would not be found. Failure of testis to secrete anti-müllerian hormone (Choice

54 Exam Section 2: Item 4 of 50 National Board of Medical Examiners® Comprehensive Basic Science Self-Assessment 4. A 34-year-old woman comes to the physician because of a 2-week history of easy fatigability. She is pale. Physical examination shows scleral icterus and mild jaundice. Laboratory studies show: 9.2 g/dL 10% of red cells Hemoglobin Reticulocyte count Serum Total bilirubin Direct bilirubin 3.0 mg/dL 0.3 mg/dL Which of the following is most likely responsible for the development of anemia in this patient? A) Chronic disease OB) Hemoglobinopathy C) Hemolysis OD) Hypersplenism E) Ineffective erythropoiesis

C. Autoimmune hemolysis explains this patient's anemia, unconjugated hyperbilirubinemia, and reticulocytosis. Autoimmune hemolytic anemia (AIHA) occurs because of production of antibodies targeting circulating red blood cell (RBC) surface antigens, marking them for removal in the reticuloendothelial system (RES) or fixing complement leading to intravascular hemolysis. IgG antibodies are active at the physiologic temperature of the human body (called warm AIHA); these do not routinely activate complement. Instead, they mark the cells for removal. Phagocytosis of the IgG antibody along with a component of the erythrocyte membrane creates sphere-shaped RBCS seen on peripheral smear. In contrast, IgM antibodies bind and fix complement at lower temperatures and result in intravascular hemolysis (called cold AIHA). IgG antibodies are known as warm antibodies, and IgM antibodies are known as cold antibodies. Regardless of the type of antibody, laboratory findings show normo- or microcytic anemia, an increased reticulocyte count consistent with increased hematopoiesis, and unconjugated hyperbilirubinemia from hemoglobin released from lysed erythrocytes. Diagnosis is made by laboratory analysis, a compatible clinical history, and positive direct antibody test (DAT, Coombs). In the DAT, the patient's erythrocytes are washed free of plasma, and incubated with Coombs reagent, an anti-IgG and anti-complement antibody. If an autoantibody or complement is bound to the surface of the erythrocytes, the Coombs reagent will bind to it and cause agglutination. Incorrect Answers: A, B, D, and E. Chronic disease (Choice A) such as chronic infection or autoimmune disease can cause anemia. The mechanism involves increased hepcidin concentrations that decrease iron absorption and increase iron storage in the bone marrow, preventing use of iron

49 Exam Section 1: Item 49 of 50 National Board of Medical Examiners® Comprehensive Basic Science Self-Assessment 49. A 1-day-old female newborn with aniridia undergoes genetic testing. Her father and several other paternal family members also have aniridia. Chromosomal microarray analysis of the patient shows no gain or loss of gene dosage throughout the genome, including the chromosome 11p15 region that contains the PAX6 commonly mutated in patients with aniridia. Routine chromosomal analysis shows a balanced translocation between chromosomes 1 and 11, with the chromosomal breakpoint on chromosome 11 being 11p15. The same balanced translocation is also found in all of the affected family members but not in the unaffected family members. However, DNA sequencing of the PAX6 located at 11p15 shows no mutation in any of the exons of PAX6 in the patient. Which of the following is the most likely cause of this patient's

C. Balanced translocations occur when a segment of a chromosome has switched places with another broken chromosomal segment. In this patient scenario, a segment of chromosome 1 has been relocated and joined onto chromosome 15 just as a segment of chromosome 15 has been joined onto chromosome 1 in place of the translocated segment. Frequently, chromosomal translocations carry no significant consequence unless the break point affects a regulatory sequence or section of an exon that results in reduced or absent translation of the encoded protein. In this case, chromosomal analysis showed that the exons of PAX6, a highly conserved gene that encodes the iris, demonstrated no mutations. Therefore, the translocation must affect a promoter, enhancer, silencer, or similar regulatory sequence that prevents appropriate expression of the gene itself. Aniridia is an autosomal dominant mutation; homozygosity at this gene locus is often severe and fatal. This patient inherited one copy of the abnormal chromosome from her father and a normal copy from the mother, leading to the observed phenotype. Regulatory sequences in DNA include promoters, the sites bound by RNA polymerase and transcription factors, enhancers, sites bound by transcription factors that generally upregulate transcription, and silencers, regions bound by repressor proteins. Incorrect Answers: A, B, D, and E. Creation of a new stop codon (Choice A) would result in cessation of ribosomal translation of the protein prior to completion. A stop codon mutation would likely be detected in the chromosomal analysis of the patient's PAX6 gene, which was reportedly found to be without mutation. Deletion of an intron (Choice B) would generally have no effect on the phenotype, as introns are non-translated intervening regions of DNA between exons that are spliced out during pre-m

25 Exam Section 1: Item 25 of 50 National Board of Medical Examiners® Comprehensive Basic Science Self-Assessment 25. A 57-year-old man dies 5 days after a stroke. He had progressively severe hypertension over the past 2 years but refused treatment. Blood pressure just prior to the stroke was 220/110 mm Hg. Which of the following is the most likely histologic finding in his kidneys at autopsy? A) Amyloidosis B) Chronic pyelonephritis C) Hyperplastic arteriolitis D) Nodular glomerulosclerosis E) Renal papillary necrosis

C. Chronic hypertension is associated with vascular changes in the small arteries and arterioles called arteriosclerosis, marked by thickening of the vessel walls and loss of elasticity. There are two primary types, which are hyaline arteriosclerosis and hyperplastic arteriosclerosis (also called hyperplastic arteriolitis). Hyaline arteriosclerosis is characterized by protein deposition in the vessel walls. Hyperplastic arteriosclerosis is characterized by concentric thickening of the vessel wall with a laminar appearance caused by the proliferation of smooth muscle cells and is associated with severe, chronic hypertension, as in this patient. Incorrect Answers: A, B, D, and E. Amyloidosis (Choice A) is an infiltrative disorder caused by the deposition of abnormal proteins in tissue. The kidneys are commonly involved, with abnormal protein deposits in the mesangium that display an apple-green birefringence with Congo red dye on histologic analysis. Renal impairment and nephrotic syndrome may develop. It is not caused by chronic hypertension. Chronic pyelonephritis (Choice B) may develop from recurrent infections of the genitourinary tract with reflux into the renal pelvis, which may occur in the setting of obstructive uropathy, nephrolithiasis, and vesicoureteral reflux. The kidneys display atrophy, calyceal deformities, and fibrosis of the renal parenchyma. Nodular glomerulosclerosis (Choice D) is associated with diabetic nephropathy and amyloidosis. It may progress to nephrotic syndrome. Renal papillary necrosis (Choice E) may result from severe ischemic injury to the kidney. Risk factors include sickle cell disease, obstructive nephropathy, nonsteroidal anti-inflammatory analgesic use, diabetes mellitus, and severe pyelonephritis. Educational Objective: Chronic hypertension can result in arteriosclerosis, which comm

66 Exam Section 2: Item 16 of 50 National Board of Medical Examiners® Comprehensive Basic Science Self-Assessment 16. The high therapeutic index displayed by acyclovir and its derivatives in treating herpesvirus infections is due to the fact that the drug must be activated by a viral enzyme. Which of the following enzymes is most likely being described? A) DNA ligase O B) Guanylyltransferase C) Thymidine kinase D) Uracil DNA glycosylase E) Viral protease

C. Herpetic infections are treated with drugs that inhibit viral DNA polymerase, classically by guanosine analogs such as acyclovir, valacyclovir, and famciclovir. Prior to exerting their antiviral effects, guanosine analogs must be phosphorylated by the viral enzyme thymidine kinase. Phosphorylation is the process by which a phosphate group is added to the molecule. These molecules are then able to inhibit the viral DNA polymerase by terminating the nascent DNA chain during replication. These drugs are effective against herpes simplex virus and varicella zoster virus, weakly effective against Ēpstein-Barr virus, and not effective against cytomegalovirus. Development of a mutation in the viral thymidine kinase enzyme would prevent drug phosphorylation and confer resistance to guanosine analog medications. Incorrect Answers: A, B, D, and E. DNA ligase (Choice A) is an enzyme related to DNA replication and modification. It joins Okazaki fragments together by catalyzing the synthesis of the phosphodiester bond within double-stranded DNA. It does not interact with guanosine analogs. Guanylyltransferase (Choice B) is an enzyme which transfers a guanosine monophosphate group to another molecule, usually pyrophosphate, in the process of MRNA formation. While the class of medications used to treat herpetic infections are guanosine analogs, they are activated by phosphorylation, not via transfer of or to a larger group. Uracil DNA glycosylase (Choice D) is a component of the base excision repair pathway and removes uracil groups from DNA, which form from deamination of cytosine residues. It is not involved in the activation of guanosine analogs. Viral proteases (Choice E) cleave the initial polypeptides produced by the translation of viral RNA into smaller, functional parts. Protease inhibitors are used in the treatment of HIV

20 Exam Section 1: Item 20 of 50 National Board of Medical Examiners® Comprehensive Basic Science Self-Assessment 20. A 67-year-old white woman undergoes punch biopsy of an urticarial lesion on her back. When the biopsy specimen container is received in the laboratory, it is determined that it contains two specimens instead of one. Review of the additional specimens received from the physician's office indicates that specimens from two separate patients were inadvertently shipped in the same container. The other specimen is from the back of a dark-skinned, 58-year-old Nigerian woman. An increase in which of the following in the specimen from the Nigerian patient is most likely to differentiate these two specimens? A) Number of melanocytes B) Number of melanophages C) Number of melanosomes D) Size of melanocytes

C. Melanin is a pigment made within a specialized cellular organelle called a melanosome found in melanocytes, which are dendritic cell derivatives that reside in the basal layer of the epidermis. Melanocytes are derived from neural crest cells, which originate at the dorsal neural tube. During embryologic development, they migrate from dorsal to ventral, and then to the epidermis. The initial substrate in the creation of melanin is the amino acid tyrosine. After producing melanin from tyrosine, the melanosomes are transferred to the surrounding keratinocytes. The variation in skin tones seen in different ethnic groups is based on the number of the melanosomes and the distribution of melanin within them in the keratinocytes, not the number of melanocytes themselves. In patients with darker skin, the melanosomes are more numerous and more densely packed with melanin. In patients with lighter skin, the melanin is distributed with less density, and there are fewer melanosomes. Melanin production is regulated in part by melanocyte-stimulating hormone (MSH). MSH is a byproduct of proopiomelanocortin (POMC) which also produces adrenocorticotropic hormone (ACTH). In adrenal insufficiency, stimulation of ACTH production by POMC simultaneously produces MSH, leading to diffuse hyperpigmentation in affected patients. Incorrect Answers: A, B, and D. Neither the number of melanocytes (Choice A) nor their size (Choice D) differs between patients with different skin types. Increased number, size, and nuclear/cytoplasmic ratio of melanocytes would be seen in melanoma, a neoplasm of melanocytes. Lesions concerning for melanoma are characterized clinically by asymmetry, irregular-appearing borders, variable coloration, a diameter greater than 6 mm, and rapid evolution in characteristics. Melanophages are macrophages which have taken up

30 Exam Section 1: Item 30 of 50 National Board of Medical Examiners® Comprehensive Basic Science Self-Assessment 30. A transgenic animal containing a targeted mutation in the gene coding for macrophage colony-stimulating factor is prepared. Which of the following generalized skeletal abnormalities is expected in this animal? A) Achondroplasia B) Osteomalacia C) Osteopetrosis D) Osteoporosis E) Skeletal agenesis

C. Osteopetrosis is a rare, inherited condition in which an imbalance in the function of osteoclasts and osteoblasts arises, leading to excess bone mineral deposition with resulting dense, abnormally mineralized bones. Consequences of imbalanced bone density in osteopetrosis include early mineralization and fusion of the physis with stunted growth, cranial nerve impingement with resultant blindness, deafness, or loss of cranial nerve function, bone pain, craniosynostosis, scoliosis, hypocalcemia, anemia, and immunodeficiency. In contrast, conditions of low bone density such as osteoporosis generally present with fragility fractures. There are many genetic causes of osteopetrosis, with the final common pathway being the failure of the osteoclast to appropriately resorb bone. Genetic mutations may include channelopathies (CLCN7), receptor activator of nuclear factor kappa-B ligand (RANKL), the RANK receptor, or carbonic anhydrase (CÁ2) deficiencies. The net effect of any such defect is impaired resorption by the osteoclast. Osteoclasts originate from the monocyte-macrophage lineage (a hematopoietic line); therefore, mutations that inactivate macrophage colony-stimulating factor would prevent differentiation of these cells. RANKL is also required for their differentiation and activation. By contrast, osteoblasts arise from mesenchymal stem cells. Osteoclasts are large, multinucleated cells found in pits at the surface of bone, where they produce matrix metalloproteases, cathepsins, and hydrogen ions, which dissolve bone matrix. Incorrect Answers: A, B, D, and E. Achondroplasia (Choice A) results in impaired growth of long bones caused by an autosomal dominant mutation in the fibroblast growth factor receptor 3 (FGFR3) gene. Affected patients have short extremities and a torso of normal length. Intelligence is generally u

21 Exam Section 1: Item 21 of 50 National Board of Medical Examiners® Comprehensive Basic Science Self-Assessment 21. An investigator is studying the adverse effects of a proteasome inhibitor, bortezomib, on immune responses. Bortezomib is administered to a group of patients with relapsed multiple myeloma, and immune responses are observed. Which of the following immunologic processes is primarily affected by this drug? A) Activation of the complement cascade B) Activation of perforin C) Presentation of antigens to CD8+ T lymphocytes D) Secretion of histamine by mast cells E) Secretion of interleukin-1 (IL-1) by macrophages

C. Presentation of antigens to CD8+ T lymphocytes would be affected by the use of the proteasome inhibitor bortezomib. Proteasomes are large proteases with numerous domains that are present in both the cytoplasm and the nucleus. They are made of a and B subunits and have an extensive role in cell cycle regulation, but also in immune function. The caspase, chymotrypsin, and trypsin-like protease capabilities of this enzyme complex are also critical to their role as antigen processing centers; intracellular peptides are processed in the proteasome for presentation on class I major histocompatibility complex (MHC). Antigen-bound class I MHC molecules are recognized by cytotoxic CD8+ T lymphocytes, so proteasome inhibitors blunt the response of CD8+ T lymphocytes by altering antigen presentation on the surface of MHC I molecules. Incorrect Answers: A, B, D, and E. Activation of the complement cascade (Choice A) is not a feature of proteasome inhibitors. The complement cascade is activated by the classical, alternative, or lectin pathways. Activation leads to opsonization of invading microbes, direct microbial killing via formation of the membrane attack complex (MAC), and proinflammatory signaling (eg, C5a mediation of neutrophil chemotaxis). Activation of perforin (Choice B) occurs as a result of activated natural killer cells, which are immune cells that primarily respond to an absence of MHC I molecules on the surface of cells. Perforins, along with granzyme, are proapoptotic agents and induce cellular death. Proteasome inhibitors interfere with antigen presentation on MHC I molecules but do not affect the presence of MHČ I molecules. Secretion of histamine by mast cells (Choice D) occurs as a result of mast cell degranulation, a process that primarily occurs during allergic reactions but can also occur in the setting

35 Exam Section 1: Item 35 of 50 National Board of Medical Examiners® Comprehensive Basic Science Self-Assessment 35. A 22-year-old primigravid woman at 20 weeks' gestation comes to the physician because of a 4-day history of dull pelvic pain that radiates to the right side of her labia majora. She says, "It feels like something is pulling." She is concerned that she is going into labor. Physical examination shows no dilation of the cervix. Stretching of which of the following ligaments is the most likely cause of this patient's pain? A) Broad ligament B) Mesosalpinx C) Round ligament D) Suspensory ligament of the ovary E) Uterosacral ligament

C. The round ligament of the uterus is a fibromuscular band of tissue that extends from the superolateral part of the uterus where the fallopian tubes insert, passes through the inguinal canal, and ends in the labia majora. It is a remnant of the gubernaculum, an embryonic structure which assists in the descent of the gonads. In pregnancy, as the uterus increases in size, the round ligament becomes stretched. Increased concentration of the hormone relaxin during pregnancy also contributes to its stretching. Relaxin allows for the ligaments of the pelvis and hip girdle to stretch so the birth canal can accommodate the fetus during delivery. The combination of increased elasticity and increased size of the uterus strains the round ligament and commonly leads to pain during pregnancy. Sudden movements or vigorous activity may precipitate this pain. There is no association of round ligament pain with preterm labor, but other causes of abdominal pain during pregnancy should be ruled out before attributing pain to the round ligament. No treatment is necessary, as the pain is self-limited and typically mild. Incorrect Answers: A, B, D, and E. The uterus is supported by the broad ligament (Choice A) which attaches to the superior aspect of the uterus and is a layer of peritoneum. The mesometrium, mesosalpinx (Choice B), and mesovarium are named portions of the broad ligament. While these structures are all contiguous, the mesometrium covers the uterus and creates the majority of the broad ligament, the mesosalpinx covers the fallopian tubes, and the mesovarium covers the ovaries. Stretching of this ligament during pregnancy does not cause labial pain. The suspensory ligament of the ovary (Choice D) extends laterally from the ovary to the wall of the pelvis and runs adjacent to the ovarian artery and vein. Its size does not sig

75 Exam Section 2: Item 25 of 50 National Board of Medical Examiners® Comprehensive Basic Science Self-Assessment 25. A 20-year-old woman comes to the office because of a 1-month history of a mass in her neck. She has a 16-year history of cerebellar ataxia and progressive scleral telangiectatic lesions. Her brother has a similar condition. Physical examination shows a large, firm, nonmoveable lymph node palpable in the left anterior side of the neck. A biopsy specimen of the node shows evidence of Hodgkin lymphoma. Which of the following genetic mechanisms is the most likely cause of this patient's lymphoma? A) Activation of a tumor suppressor gene B) Deletion of the BCL gene C) DNA repair defect D) Initiation of somatic hypermutation

C. This patient with ataxia telangiectasia possesses a genetic mutation of the ATM gene leading to a DNA repair defect. The ATM gene product delays the progression of cells with DNA damage through the cell cycle, which allows the cell to repair the DNA. When the ATM gene product is defective, somatic mutations accumulate, leading to the dysfunction of diverse proteins and an increased risk for malignancy. Patients present with cerebellar atrophy and progressive ataxia, telangiectatic lesions of the eyes and exposed areas of the skin, and other movement abnormalities such as dystonia, abnormal eye movements (eg, saccades), and decreased immunoglobulin concentrations (IgA, IgG, and IgE). Conditions associated with ataxia telangiectasia include pulmonary disease (eg, bronchiectasis from recurrent sinopulmonary infections) and hematologic malignancies such as Hodgkin lymphoma. Ataxia telangiectasia demonstrates an autosomal recessive inheritance pattern. Management requires the surveillance and treatment of disease complications. Incorrect Answers: A, B, and D. Activation of a tumor suppressor gene (Choice A) such as P53 would arrest the cell cycle. Activating tumor suppressor genes would decrease cellular proliferation and would not result in malignancy. Deletion of the BCL gene (Choice B) may result in hematologic deficiencies such as anemia. The BCL gene is an oncogene that normally inhibits apoptosis and may promote malignancy (eg, follicular lymphoma) when activated. Initiation of somatic hypermutation (Choice D) occurs during normal immunoglobulin class switching. Somatic hypermutation is not directly associated with malignancy. Educational Objective: Ataxia telangiectasia is an autosomal recessive disorder resulting from a DNA repair defect. Consequently, somatic mutations accumulate, leading to protein dysfunction

45 Exam Section 1: Item 45 of 50 National Board of Medical Examiners® Comprehensive Basic Science Self-Assessment 45. Human gene fragments in appropriate vectors may be introduced into bacterial cells by transformation. Which of the following is most likely to be used to transfer cloned human genetic material to bacteria? A) DNA complexed with bacterial histones O B) Human chromosomal fragments C) Naked DNA D) Purified euchromatin E) Purified heterochromatin

C. Transformation describes the ability of bacteria to uptake and incorporate exogenous genetic material (naked DNA) from the environment. Following lysis of a cell DNA may exist extracellularly; a bacterium may uptake such material through the cell membrane. Transformation is one of three methods of horizontal gene transfer by which bacteria can acquire novel genes which may confer a survival benefit. Notably, Streptococcus pneumoniae and Haemophilus influenzae type B have demonstrated acquisition of genetic material by transformation. In order for transformation to occur, a bacterium must demonstrate competence (the ability to uptake such material). Competence occurs in states of stress such as starvation. Beyond transformation, horizontal gene transfer may also occur via conjugation (in which two cells in direct contact, typically involving a sex pilus), exchange material, and transduction (transfer of genetic material via a viral vector such as a bacteriophage). Incorrect Answers: A, B, D, and E. DNA complexed with bacterial histones (Choice A), human chromosomal fragments (Choice B), purified euchromatin (Choice D), and purified heterochromatin (Choice E) all describe fragments of DNA associated with additional protein sizes. The process of transformation requires naked DNA, which describes DNA that is not associated with proteins, lipids, or molecules that may shield it. exes and of variable Educational Objective: Transformation describes the ability of bacteria to uptake and incorporate genetic material (naked DNA) that is not associated with proteins or lipids from the environment. This mechanism may promote genetic diversity and acquisition of survival advantage. %3D Previous Next Score Report Lab Values Calculator Help Pause

23 Exam Section 1: Item 23 of 50 National Board of Medical Examiners® Comprehensive Basic Science Self-Assessment 23. A 35-year-old man comes to the physician because of a 3-year history of an enlarging nose, coarsening of his facial features, muscle weakness, and increased hand and foot size. Physical examination shows a large fleshy nose and prognathism. Fasting serum studies show increased insulin-like growth factor-l, glucose, and triglyceride concentrations. Increased serum growth hormone concentrations do not decrease after the administration of oral glucose. Compared with a healthy patient, which of the following best describes the metabolic changes in this patient? Muscle Glucose Adipose Lipolysis Неpatic Gluconeogenesis Uptake O A) ↑ ↑ ↑ B) ↑ ↑ C) ↑ D) ↑ ↑ E) ↑ F)

D. Acromegaly is caused by a growth hormone (GH) secreting pituitary adenoma that stimulates the excessive production of insulin-like growth factor-1 (IGF-1) by the liver. IGF-1 interacts with its receptor, a tyrosine kinase-based receptor, that stimulates cell growth, proliferation, and growth of the axial and appendicular skeleton. Acromegaly occurs after the closure of growth plates, and in this context excessive IGF-1 leads to expansion of flat bones and tissues. Excessive GH and IGF-1 leads to deranged glucose homeostasis by increasing peripheral insulin resistance, impairing muscle and adipose uptake of glucose, increasing adipose lipolysis, and increasing hepatic gluconeogenesis. Consequently, up to half of patients with acromegaly develop diabetes mellitus. Patients with acromegaly also exhibit increased rates of hypertriglyceridemia. Incorrect Answers: A, B, C, E, and F. Choices A, B, and C reflect states of increased muscle glucose uptake, which would occur in the setting of increased peripheral insulin concentrations or receptor sensitivity. Excessive concentrations of GH and IGF-1 lead to reduced peripheral insulin sensitivity. Choices B, C, and F reflect states of decreased lipolysis, whereas deficient insulin signaling will lead to increased lipolysis. Choices C, E, and F reflect states of decreased hepatic gluconeogenesis. GH directly stimulates gluconeogenesis, which is increased in acromegaly. Educational Objective: Acromegaly is associated with excessive GH and IGF-1 concentrations that lead to deranged glucose homeostasis by increasing peripheral insulin resistance, impairing muscle and adipose uptake of glucose, and increasing lipolysis and hepatic gluconeogenesis. Patients with acromegaly experience increased rates of diabetes mellitus and hypertriglyceridemia. %3D Previous Next Score Report Lab Va

60 Exam Section 2: Item 10 of 50 National Board of Medical Examiners® Comprehensive Basic Science Self-Assessment 10. A 45-year-old woman comes to the physician because of a 2-month history of fatigue, nausea, and generalized bone pain. She has had a 2-kg (4.4-lb) weight loss during this period because of loss of appetite. She has a 4-month history of renal insufficiency. Her blood pressure is 170/100 mm Hg. Physical examination shows pallor and 2+ pitting edema of the feet. Laboratory studies show anemia. Her serum urea nitrogen concentration is 55 mg/dL, and serum creatinine concentration is 4 mg/dL. Bone x-rays show widened osteoid seams and subperiosteal erosions. Which of the following additional sets of serum findings is most likely in this patient? 1,25- Dihydroxycholecalciferol Calcium Phosphate Parathyroid Hormone O A) ↑ ↑ ↑ B) ↑ ↑ ↑ C) ↑ D) ↑ E) ↑ F) G)

D. Chronic renal disease is a cause of secondary hyperparathyroidism. Secondary hyperparathyroidism typically presents with increased parathyroid hormone (PTH), hypocalcemia, and hyperphosphatemia. It results from chronic renal failure caused by the inability of the kidney to excrete phosphate, reabsorb calcium, and produce active vitamin D, as the final conversion of 25-hydroxycholecalciferol to active 1,25-dihydroxycholecalciferol occurs in the kidney via 1-a hydroxylase. Hypocalcemia is exacerbated by the decreased intestinal absorption of calcium in secondary to the deficiency of active vitamin D. Hyperphosphatemia and hypocalcemia result in the upregulation of PTH, which increases bony turnover to raise serum calcium and promotes renal excretion of phosphate. Unregulated, this process results in renal osteodystrophy, the breakdown of bone caused by excessive PTH stimulus of osteoclasts. This is clinically manifest with widened osteoid seams and subperiosteal erosions, as seen in this patient. Incorrect Answers: A, B, C, E, F, and G. While parathyroid hormone and phosphate are increased in chronic kidney disease (Choice A), calcium and 1,25-dihydroxycholecalciferol are decreased, not increased. Active vitamin D is decreased because the kidney parenchyma is the site of its activation and is lost in chronic disease. Hypocalcemia, not hypercalcemia, occurs as a result of the combination of decreased calcium reabsorption, lack of activation of vitamin D, and decreased phosphate excretion. In chronic renal disease, secondary hyperparathyroidism, not hypoparathyroidism (Choices B, C, E, and G) occurs. This is in response to the hypocalcemia that is perpetuated by decreased calcium reabsorption, lack of activation of vitamin D, and decreased phosphate excretion. Parathyroid hormone is increased, calcium is decreased, and

85 Exam Section 2: Item 35 of 50 National Board of Medical Examiners® Comprehensive Basic Science Self-Assessment 35. A physician prescribes a recently marketed drug for 20 patients. After several months, five patients develop increased serum AST and ALT activities and show clinical symptoms of hepatotoxicity. The physician discontinues the drug for all patients and reports the adverse effects to the FDA. The physician has participated in which of the following phases of clinical drug testing? A) Phase 1 B) Phase 2 C) Phase 3 D) Phase 4

D. Clinical trials are an important aspect of the development of any new pharmaceutical or intervention. They involve the experimental study on human subjects and compare a new treatment against a placebo or an alternative treatment. The FDA regulates trials for newly developed drugs, and a series of phases must be undertaken to demonstrate safety, efficacy, comparability to standard of care, and long-term adverse effects. In this example a drug that has been recently marketed is prescribed to 20 patients, 25% of whom show signs of hepatotoxicity after several months. The action of reporting this to the FDA describes the physician participating in Phase 4 of the drug trial, which is characterized by post-distribution and post-prescription surveillance following treatment approval. Long-term adverse effects and rare adverse effects or outcomes are often identified this way, and failure of a drug during Phase 4 can result in discontinuation/withdrawal of the drug from the markets and loss of approval by the FDA. Incorrect Answers: A, B, and C. Phase 1 (Choice A) is the initial phase of a clinical trial. In this phase, a small number of volunteers receive the treatment, and data is collected to gauge safety and toxicity, adverse effects, interactions, pharmacodynamics, and pharmacokinetics. The volunteers in this stage are healthy. Failure at this stage limits further progress of the trial, as the intervention is deemed unsafe for human use. Phase 2 (Choice B) involves the trial of the intervention on patients who have the disease to be treated. As in Phase 1, a small number of volunteers are involved in this phase. Efficacy is assessed in this phase along with dosing. Continued study of safety and adverse effects occurs. Phase 3 (Choice C) involves the conduction of a larger, randomized controlled trial once safety and e

53 Exam Section 2: Item 3 of 50 National Board of Medical Examiners® Comprehensive Basic Science Self-Assessment 3. A3-month-old girl is brought to the physician by her mother for a well-child examination. Her mother says that the infant can lift her head when lying prone. The patient is eating well and recently started sleeping about 6 uninterrupted hours through the night. The mother adds that the baby does not yet smile in response to human faces and has not started to coo or gurgle with attention. The patient is at the 25th percentile for length and weight and 10th percentile for head circumference. Physical examination shows no other abnormalities. Which of the following best describes this infant's development? Motor Social Verbal and Cognitive A) Normal normal normal B) Normal normal delayed C) Normal delayed normal D) Normal delayed delayed OE) Delayed normal normal F) Delayed normal delayed G) Delayed delay

D. In the first 3 months of life, infants are expected to demonstrate the gross motor skills of holding their head up and pushing their body up with their arms when lying prone. Expected fine motor skills include grasping and shaking toys and bringing their hands to their mouth. Most 3-month-olds demonstrate a social smile. Verbally, 3-month-old infants are expected to coo, babble, and imitate vowel sounds. This patient does not meet social or verbal/cognitive milestones. Screening for developmental delays alerts physicians to the potential need for further medical evaluation and/or early referral to developmental specialists. Early intervention has been demonstrated to improve outcomes. Incorrect Answers: A, B, C, E, F, G, and H. This patient, who does not smile or coo yet, does not meet the social or verbal/cognitive milestones for a 3-month-old infant (Choices A, B, C, E, F, G, and H). This baby should be further evaluated for causes of developmental delay. Educational Objective: By age 3 months, infants are typically able to hold their head up, grasp toys, smile socially, and babble or coo. %3D Previous Next Score Report Lab Values Calculator Help Pause

46 Exam Section 1: Item 46 of 50 National Board of Medical Examiners® Comprehensive Basic Science Self-Assessment 46. A 4-year-old girl has acute lymphoblastic leukemia that has not responded to aggressive treatment. The physician recommends palliative care to the parents and suggests that they talk to their daughter about her impending death and answer any questions that she may have. The parents ask the physician to help them respond to her questions. When speaking with the child, the parents should anticipate that she will most likely understand death as which of the following? A) She will blame God for her illness and death B) She will comprehend that all living things die C) She will have no understanding of death D) She will view death as temporary and reversible

D. Most children between the ages of 3 and 5 years view death as temporary and reversible. Partly related to a lack of experience of death, they also do not understand that all living things inevitably die. Children of this age range who are dying may believe dying is a punishment from their parents, signifying an inability to abstract. However, these gaps in understanding can be filled by adults explaining death to children and by children's experiences of death. It is suggested that the parents of dying children help their child understand the situation. Incorrect Answers: A, B, and C. This patient is unlikely to blame God for her illness and death (Choice A) as it is too abstract a concept for a child this age. The patient is more likely to blame her parents. She may, depending on her parents' religious beliefs, believe that death represents going to Heaven without an abstract understanding of Heaven. Starting at age five, children begin to understand that death is inevitable and irreversible (Choice B). At this age, children may continue to demonstrate an incomplete understanding of death. Media images of death (eg, ghosts) may be part of this incomplete understanding. At age ten, children typically understand that death is universal, irreversible, and renders people inanimate (versus believing in ghosts). Only infants have no understanding of death (Choice C). Toddlers typically begin to become aware of the concept of death over time. Educational Objectives: Most children between the ages of 3 and 5 years view death as temporary and reversible. Further, preschool age children are frequently unaware of the inevitability of death. %3D Previous Next Score Report Lab Values Calculator Help Pause

16 Exam Section 1: Item 16 of 50 National Board of Medical Examiners® Comprehensive Basic Science Self-Assessment 16. The graph shows diastolic and end-systolic relationships of the left ventricle. The solid line shows the control pressure-volume loop in a single cardiac cycle. If a healthy person is given nitroprusside and reflex responses are blocked, which of the following labeled end-systolic pressure points is most likely? 200 A •B 100- E• C D 100 300 400 200 Left ventricular volume (mL) A) B) C) D) E)

D. Nitroprusside is an intravenous, titratable vasodilator that can be used for the treatment of hypertensive emergencies. It breaks down in circulation to release nitric oxide, which in turn activates guanylate cyclase in vascular smooth muscle to result in vascular smooth muscle relaxation and vasodilation via a cyclic GMP pathway. It typically induces a reflex tachycardia. Sodium nitroprusside preferentially dilates arterial vessels over venous vessels, resulting in a decrease in afterload without significant change in preload. An isolated decrease in afterload results in a shortened pressure-volume loop (less left ventricular pressure is needed to open the aortic valve) and a decreased end-systolic left ventricular volume (because of an increased stroke volume and ejection fraction). The associated end-systolic pressure point on this graph is represented by point D. Incorrect Answers: A, B, C, and E. Choice A represents a point that may be observed if afterload is increased and cardiac contractility is increased. The left ventricular pressure needed to open the aortic valve is increased, and the increased contractility maintains the stroke volume to result in the same end-systolic left ventricular volume as the control. Choice B represents the end-systolic pressure point associated with an isolated increase in afterload. The left ventricular pressure needed to open the aortic valve is increased and the stroke volume is reduced, resulting in an increased end-systolic left ventricular volume. Choice C represents an end-systolic pressure point that may be seen in the case of decreased afterload and decreased cardiac contractility, with reduced end-systolic left ventricular pressure and a reduced stroke volume resulting in increased end-systolic left ventricular volume. Choice E represents a point that may occur with a

3 Exam Section 1: Item 3 of 50 National Board of Medical Examiners® Comprehensive Basic Science Self-Assessment 3. A 53-year-old man comes to the physician because of a 6-day history of shortness of breath, cough, and pleuritic chest pain. His temperature is 38.9°C (102°F), and respirations are 35/min. Sputum is purulent and rust colored. Physical examination shows decreased breath sounds, and crackles are heard at the left base. A Gram stain of sputum shows gram-positive diplococci. A chest x-ray shows left-sided lobar consolidation. Which of the following anatomic structures most likely allowed rapid spread of organisms between alveoli to involve the entire left lobe in this patient? A) Alveolar capillaries B) Germinal centers C) Lymphatic capillaries D) Pores of Kohn E) Vascular sinusoids

D. The pores of Kohn form connections between alveoli and are present in normal lung tissue. They are composed, at least in part, of type Il alveolar cells and allow for the passage of air, fluid, phagocytes, and in the setting of pneumonia, bacteria between adjacent alveoli. By allowing equilibration between adjacent alveoli, the pores of Kohn aid in normal oxygenation and in the prevention of atelectasis. However, infectious organisms and inflammation may also spread between adjacent alveoli through these apertures. Incorrect Answers: A, B, C, and E. Alveolar capillaries (Choice A) surround each alveolus and are crucial for gas exchange. However, they do not provide a direct connection between adjacent alveoli, provided that the alveolar and capillary endothelia remain intact. Germinal centers (Choice B) allow for the development of B lymphocytes and are important in mounting an adaptive immune response to pulmonary pathogens. They do not form connections between alveoli. Lymphatic capillaries (Choice C) allow for the drainage of interstitial fluid from the lung parenchyma to the lymphatic ducts and eventually to the heart. These passages do not form direct anastomoses between adjacent alveoli. Vascular sinusoids (Choice E) are present in hepatic tissue and create anastomoses between the portal and systemic circulation, allowing for the exchange of nutrients with hepatocytes. They are not present in pulmonary tissue. Educational Objective: The pores of Kohn form connections between alveoli and are present in normal lung tissue. They allow for the passage of air, fluid, phagocytes, and, in the setting of pneumonia, bacteria between adjacent alveoli. %3D Previous Next Score Report Lab Values Calculator Help Pause

82 Exam Section 2: Item 32 of 50 National Board of Medical Examiners® Comprehensive Basic Science Self-Assessment 32. A 45-year-old man is admitted to the hospital because of congestive heart failure. Two first-degree relatives died in their 40s with dilated cardiomyopathy and cirrhosis. Physical examination shows cardiac enlargement and generalized hyperpigmentation. His serum glucose concentration is 320 mg/dL. Histologic examination of endomyocardial tissue is most likely to show an excess of which of the following? A) Amyloid B) a-Antitrypsin C) Cerebroside D) Copper E) Eosinophils F) Iron OG) Lysosomal glycogen H) Mucopolysaccharide

F. Iron deposition will most likely be appreciated on histologic examination of endomyocardial tissue in this patient with a probable diagnosis of hemochromatosis. Hemochromatosis may be acquired or inherited secondary to mutations in the HFE gene, leading to abnormally increased intestinal absorption of iron. This results in accumulation of iron in the body, increased serum iron, and increased ferritin. Iron can accumulate in several organs, including the liver, pancreas, skin, heart, and joints. Because of increased free radical generation and oxidative damage, hemochromatosis can manifest with failure of the affected organs. It typically presents after decades of iron accumulation with liver failure manifest as cirrhosis and portal hypertension, diabetes mellitus, arthritis secondary to calcium pyrophosphate deposition, cardiomyopathy with resultant symptoms of heart failure, darkening of the skin, and gonadal atrophy. Hemochromatosis, when acquired, may occur in the setting of transfusion-dependent anemias such as thalassemia, but family history of similar symptoms makes hereditary hemochromatosis more likely in this patient. Diagnostic studies may include biopsy, which commonly shows iron deposition on Prussian blue stain. Treatment involves serial phlebotomy and management of secondary disorders such as heart failure and diabetes mellitus. Incorrect Answers: A, B, C, D, E, G, and H. Amyloid (Choice A) deposition is seen in patients with amyloidosis, a disorder in which low molecular weight proteins deposit in tissues and cause organ dysfunction. These proteins are often immunoglobulin light chains. Amyloidosis can cause restrictive cardiomyopathy. Cirrhosis may occur but bronzing of the skin and diabetes mellitus are more consistent with hemochromatosis. a-Antitrypsin (Choice B) deficiency results in liver diseas

93 Exam Section 2: Item 43 of 50 National Board of Medical Examiners® Comprehensive Basic Science Self-Assessment 43. A 28-year-old woman has had restlessness, exercise intolerance, palpitations, diarrhea, and excessive sweating for the past 3 days. She had an upper respiratory tract infection 3 weeks ago. The thyroid gland is diffusely enlarged and tender. Serum total thyroxine (T) and total triiodothyronine (T3) concentrations and free T, index are increased. Serum thyroid-stimulating hormone concentration and radioactive iodine uptake by the thyroid gland are both decreased. Which of the following is the most likely cause of her condition? A) Autoimmune (Hashimoto) thyroiditis B) Diffuse toxic goiter (Graves disease) C) Euthyroid sick syndrome D) Subacute thyroiditis E) Toxic adenoma F) Toxic multinodular goiter

D. This patient's presenting findings are highly suggestive of subacute granulomatous thyroiditis, also known as de Quervain thyroiditis. Subacute granulomatous thyroiditis is a self-limited inflammatory condition of the thyroid gland that often follows an acute viral illness. Presenting findings are typically suggestive of clinical hyperthyroidism and may include diaphoresis, palpitations, neck pain, dysphagia, fever, tachycardia, increased serum concentrations of T3 and T4 and a decreased serum concentration of thyroid- stimulating hormone. Patients may eventually become hypothyroid or euthyroid following the acute inflammatory, hyperthyroid phase. Subacute granulomatous thyroiditis is distinguished from other forms of hyperthyroidism by the presence of a painful, tender thyroid, which is highly suggestive of the diagnosis. It is also characterized by the absence of iodine uptake on a radionuclide scan, and by the presence of multinucleated giant cells on fine needle aspiration. Incorrect Answers: A, B, C, E, and F. Autoimmune (Hashimoto) thyroiditis (Choice A) is the most common form of thyroiditis and is characterized by the presence of antithyroid peroxidase and antithyroglobulin antibodies. Patients with chronic disease present with signs and symptoms of hypothyroidism, including fatigue, cold intolerance, weight gain, hyporeflexia, myxedema, and dry, cool skin. While patients in the acute stage of the disease may present with symptoms of hyperthyroidism, the thyroid is not typically painful or tender. Diffuse toxic goiter (Graves disease) (Choice B) is the most common cause of hyperthyroidism and is caused by an autoantibody that activates thyroid stimulating hormone receptors on the thyroid. It presents with symptoms of hyperthyroidism, pretibial myxedema, and thyroid ophthalmopathy, which can cause diplopia, p

27 Exam Section 1: Item 27 of 50 National Board of Medical Examiners® Comprehensive Basic Science Self-Assessment 27. A study is conducted in which the right renal artery of an experimental animal is constricted to decrease arcuate artery pressure by 20 mm Hg. Measurement of inulin clearance shows that renal artery constriction has no effect on glomerular filtration rate (GFR). Which of the following best explains the maintenance of a constant GFR in this experiment? A) Decreased glomerular filtration coefficient B) Decreased glomerular hydrostatic pressure C) Decreased renal blood flow D) Increased afferent arteriolar resistance E) Increased efferent arteriolar resistance F) Increased renal blood flow

E. Autoregulation is an essential process for maintaining perfusion of end organs despite variations in systemic blood pressure and takes place in numerous organs throughout the body. Renal autoregulation is adapted to produce a relatively constant GFR despite changes in glomerular perfusion and is mediated by the delivery of filtered sodium to the macula densa in the early distal convoluted tubule. In renal artery stenosis, there is decreased glomerular perfusion. Reductions in glomerular perfusion result in decreased filtration and decreased sodium delivery to the macula densa. The macula densa compensates for this by promoting activation of the renin-angiotensin system by stimulating the juxtaglomerular apparatus to secrete renin, which subsequently causes an increased angiotensin Il-mediated constriction of the glomerular efferent arteriole. This causes increased efferent resistance and induces an increased intraglomerular pressure to maintain GFR. Systemic hypertension can also be counteracted by renal autoregulation through a similar but opposite mechanism. As with all autoregulatory systems, renal autoregulation is only able to maintain glomerular filtration within a particular range of systemic blood pressures and can be overwhelmed by either profound hyper- or hypotension. Incorrect Answers: A, B, C, D, and F. Decreased glomerular filtration coefficient (Choice A) is not a mechanism of renal autoregulation. The glomerular filtration coefficient is an inherent property of the renal capillary and is not freely modified in response to changes in blood pressure. Some diseases that result in glomerular capillary damage, such as diabetes mellitus, may decrease the filtration coefficient. Decreased glomerular hydrostatic pressure (Choice B) would result in decreased glomerular filtration and exacerbate the hypofiltra

73 Exam Section 2: Item 23 of 50 National Board of Medical Examiners® Comprehensive Basic Science Self-Assessment 1 cm 23. A 56-year-old woman comes to the physician because of cough productive of blood-tinged sputum for 8 days. She also has had fatigue, loss of appetite, and an unintentional 6.8-kg (15-lb) weight loss during the past 2 months. She has a 5-year history of chronic cough and shortness of breath that she had attributed to smoking 2 packs of cigarettes daily for 42 years. She appears ill. Physical examination shows dullness to percussion over the right lower lung lobe. A chest x-ray shows a mass in the right upper lung lobe. The gross and microscopic appearances of a lung from a patient with a similar condition are shown. Which of the following is the most likely type of neoplasm in this patient? A) Adenocarcinoma B) Giant cell carcinoma C) Large cell undifferentiated carcinoma D) Small cell carcinoma E

E. Squamous cell carcinoma of the Ilung, which typically presents as a central lesion, is the second most common type of primary lung cancer following adenocarcinoma, which more often presents as a peripheral lesion. Risk factors for all major types of lung cancer include tobacco use, secondhand smoke, asbestos, and radon exposure, and a family history of lung cancer. Features associated with squamous cell carcinoma of the lung include pulmonary cavitations, central location, and hypercalcemia caused by paraneoplastic parathyroid hormone-related peptide (PTHPP) production. Histologic characteristics include polygonal cells with intercellular bridges, eosinophilic cytoplasm, keratin pearls (as seen in the photomicrograph), and extensive necrosis. Lung cancer, in general, typically presents with cough, unintentional weight loss, hemoptysis, chest pain, dyspnea, and hoarseness; occasionally, wheezing, focal rhonchi, or hypertrophic osteoarthropathy may be noted on examination. Obstruction of the airways can lead to recurrent postobstructive pneumonia. Diagnosis is made by chest imaging and examination of a biopsy specimen. Prognosis is a function of the cancer type along with grading and staging of the disease. It is often detected once metastatic, at which point the prognosis is poor. Incorrect Answers: A, B, C, and D. Adenocarcinoma (Choice A) of the lung is the most common overall primary lung cancer and the most common among nonsmokers. It typically presents as a chronic consolidation in the periphery of the lung rather than centrally. A glandular pattern is classically seen on histology with mucin-positive staining. Giant cell carcinoma (Choice B) is a rare carcinoma of the lung that contains pleomorphic giant, multinucleated cells on histology. It most commonly involves the upper lobes and the lung periphery. Large

91 Exam Section 2: Item 41 of 50 National Board of Medical Examiners® Comprehensive Basic Science Self-Assessment 41. An 88-year-old man loses consciousness when his neck is palpated during a routine health maintenance examination. He has a 40-year history of hypertension that has been well controlled with hydrochlorothiazide and a 20-year history of degenerative osteoarthritis, for which he occasionally takes anti-inflammatory drugs. Which of the following is the most likely cause of the syncope? A) Complete heart block B) Laryngospasm C) Peripheral arteriole constriction D) Peripheral arteriole dilation E) Sinus bradyarrhythmia F) Ventricular tachyarrhythmia

E. Carotid sinus syndrome is characterized by an exaggerated response to carotid baroreceptor stimulation, with resultant sinus bradycardia, hypotension, and syncope. Baroreceptors located at the carotid bifurcations and the aortic arch are important in regulating cardiac output and maintaining adequate perfusion pressures to the brain and organs of the upper body. Baroreceptors are sensitive to mechanical pressure; normally, an increase in blood pressure stimulates baroreceptor activity, which activates a reflex arc resulting in parasympathetic innervation of the sinoatrial and atrioventricular nodes and inhibition of sympathetic vascular tone. This results in decreased pulse and peripheral vascular resistance. Mechanical compression of the baroreceptors from external palpation can stimulate this reflex, which may be exacerbated in the setting of volume depletion, such as the presented case where the patient is taking a thiazide diuretic. Incorrect Answers: A, B, C, D, and F. Complete heart block (Choice A), also called third-degree atrioventricular block, occurs when no sinus impulses are transmitted through the AV conduction pathway, resulting in complete dissociation between the P waves and the QRS complexes. Patients usually require a pacemaker. The temporal association with neck palpation in this patient is more suggestive of carotid sinus syndrome and sinus bradyarrhythmia. Laryngospasm (Choice B) is an exaggerated protective airway reflex to prevent aspiration that can result in partial or complete airway obstruction. It typically presents with acute dyspnea, increased work of breathing, and may result in hypoxemia-induced bradycardia. Peripheral arteriole constriction (Choice C) occurs with increased sympathetic activity and signaling at vascular adrenergic receptor sites, resulting in an increase in blood pre

32 Exam Section 1: Item 32 of 50 National Board of Medical Examiners® Comprehensive Basic Science Self-Assessment 32. A 70-year-old man comes to the physician because of a 3-day history of shortness of breath and right-sided chest pain. His respirations are 25/min. Chest x-rays show a right pleural effusion and an associated ipsilateral, scallop-shaped pleural density. Examination during an open lung biopsy shows a thick, firm, white pleural tumor that ensheathes the right lung. Which of the following is the strongest predisposing risk factor for this patient's condition? A) Family history of lung cancer B) Radiation therapy C) Raising pigeons D) Smoking cigarettes E) Working in a shipyard

E. Chronic cough, shortness of breath, and pleural effusion plus a calcified pleural plaque on x-ray is consistent with a diagnosis of malignant mesothelioma, which can be confirmed by biopsy. Development of malignant mesothelioma is strongly associated with pulmonary asbestosis. A history of working in a shipyard is a known risk factor for exposure to asbestos, along with roofing and plumbing. Asbestosis is a progressive fibrotic disease that manifests decades after exposure. Long-term complications include interstitial fibrosis, chronic respiratory failure, and malignancy (especially malignant mesothelioma and bronchogenic carcinoma). Malignant mesothelioma typically presents with an exudative pleural effusion caused by the malignant pleural disease. Treatment includes chemotherapy, surgical resection, and radiation. Incorrect Answers: A, B, C, and D. Family history of lung cancer (Choice A), radiation therapy (Choice B), and smoking cigarettes (Choice D) are risk factors for developing primary bronchogenic carcinoma. Asbestosis is also a risk factor for bronchogenic carcinoma, although it also has a strong association with the development of malignant mesothelioma. Raising pigeons (Choice C) is a risk factor for chronic hypersensitivity pneumonitis, an inflammatory condition of the lungs in response to environmental irritants. The condition may progress to pulmonary interstitial fibrosis, and the chronic inflammation increases the risk for developing primary bronchogenic carcinoma. Educational Objective: Asbestosis is a chronic progressive pulmonary disorder associated with interstitial fibrosis and an increased risk for both primary bronchogenic carcinoma and malignant mesothelioma. Occupational risk factors for asbestos exposure include shipbuilding, roofing, and plumbing. %3D Previous Next Score Report Lab Values

86 Exam Section 2: Item 36 of 50 National Board of Medical Examiners® Comprehensive Basic Science Self-Assessment 36. A 27-year-old man who is a weightlifter comes to the physician for a routine examination. He has been taking a synthetic androgen for the past year to increase his muscle mass. Examination shows increased scalp hair loss and mild testicular atrophy. Which of the following is most likely responsible for the decreased size of this patient's testicles? A) Compression of the testes by overdeveloped pelvic muscles B) Downregulation of pituitary luteinizing hormone receptors C) Downregulation of testicular follicle-stimulating hormone receptors D) Increased secretion of inhibin E) Increased negative feedback on gonadotropin secretion F) Increased sexual activity

E. Excessive circulating androgen concentrations in the setting of the exogenous use of synthetic androgens results in increased negative feedback on gonadotropin secretion. Testosterone provides negative feedback upon gonadotropin releasing hormone (GNRH) neurons in the hypothalamus, with subsequent decreases in the release of luteinizing hormone (LH) and of follicle-stimulating hormone (FSH) by the anterior pituitary. Downregulation of LH secretion results in decreased stimulus for the testicular production of testosterone, which leads to testicular atrophy. Testicular atrophy may commonly occur in patients who are taking anabolic steroids or in patients who are taking testosterone replacement therapy. Incorrect Answers: A, B, C, D, and F. Compression of the testes by overdeveloped pelvic muscles (Choice A) would not occur. The testes are located within the scrotum, outside of the pelvis, and are not vulnerable to compression by pelvic muscle hypertrophy. Downregulation of pituitary luteinizing hormone receptors (Choice B) would not cause testicular atrophy. The LH receptor is responsible for the transduction of LH-dependent signaling that causes the synthesis of androgens and is primarily located in gonadal tissue. Downregulation of LH receptors in gonadal tissue will lead to testicular atrophy. LH receptor is not expressed in substantial quantities in extragonadal tissues. Downregulation of testicular follicle-stimulating hormone receptors (Choice C) leads to decreased spermatogenesis. In the testis, FSH receptors are primarily expressed on Sertoli cells. Testicular atrophy is primarily mediated by the inhibition of LH signaling, rather than that of FSH. Increased secretion of inhibin (Choice D) is performed by Sertoli cells and provides negative feedback upon the anterior pituitary to decrease FSH secretion. Incre

43 Exam Section 1: Item 43 of 50 National Board of Medical Examiners® Comprehensive Basic Science Self-Assessment 43. A 70-year-old man with metastatic prostate cancer has been taking leuprolide for the past 3 months. Which of the following best describes this patient's current serum luteinizing hormone (LH) and testosterone concentrations compared with concentrations before treatment? LH Testosterone A) ↑ ↑ B) ↑ C) No change ↑ D) no change E)

E. Leuprolide is a gonadotropin-releasing hormone (GNRH) analog. If given in a pulsatile fashion, mimicking the physiologic secretion of GNRH, it will act as a GNRH receptor agonist and increase follicle-stimulating hormone (FSH) and luteinizing hormone (LH). When leuprolide is initially started in a patient, the hypothalamus reacts as if it were an agonist and there is a transient rise in LH and FSH for the first week of treatment. However, with continued use in a non-pulsatile fashion, as in this case, it will act as a GNRH receptor antagonist and subsequently decrease FSH and LH. By decreasing the production of LH and the stimulation of Leydig cells, leuprolide indirectly lowers testosterone. Prostate cancer is a hormonally sensitive cancer; androgens play a critical role in its growth. Thus, by inhibiting testosterone production, this driver of cancer growth is removed, and sensitive tumors begin to shrink. This approach to treating prostate cancer is termed medical castration. Some tumors may become castration-resistant, in which mutations develop that allow the cancer to continue growing without hormonal stimulation. Incorrect Answers: A, B, C, and D. While there is a transient increase in LH and testosterone (Choice A) for the first week of treatment, after three months of treatment leuprolide will be exerting GNRH antagonist effects. This will lead to decreased LH and testosterone concentrations. Because LH stimulates the Leydig cells to produce testosterone, when LH concentration is decreased, testosterone concentrations will also decrease (Choice B). A medication which continuously increases testosterone would not be appropriate for use in prostate cancer as it is an androgen-dependent malignancy and would continue to grow under androgen stimulation. LH decreases rather than showing no change (Choice C) when

71 Exam Section 2: Item 21 of 50 National Board of Medical Examiners® Comprehensive Basic Science Self-Assessment 21. In an animal model of ischemic myocardial injury, a branch of the coronary artery is clamped. Which of the following morphologic changes is most likely to occur first? O A) Defects in the plasma membrane B) Myelin figures O C) Nuclear pyknosis D) Rupture of lysosomes E) Swelling of the endoplasmic reticulum

E. Swelling of the endoplasmic reticulum (ER) is most likely to be the first morphologic change that will occur after clamping a branch of the coronary artery. Myocardial cells have a high metabolic demand, and complete cessation of blood flow to the myocardium leads to myocardial ischemia. If blood flow is not restored, myocardial ischemia progresses to myocardial infarction, at which point cardiomyocyte apoptosis occurs. One of the first cellular changes that occurs after cessation of blood flow is swelling of the cell and its organelles, including the ER. Decreased oxygen impairs the ability of the cell to generate ÁTP, which is used to pump sodium out of the cell and potassium into the cell via the action of the Na -K+-ATPase in the plasma membrane. Cessation of this pump results in intracellular accumulation of sodium, thereby shifting the osmotic gradient and leading to cellular swelling. As protein synthesis is an energy intensive process, ischemia can also alter the folding of cellular proteins. These proteins accumulate in the ER. This process triggers the unfolded protein response, which seeks to restore normal function by degrading unfolded or misfolded proteins and by downregulating protein synthesis. If unsuccessful, the unfolded protein response induces apoptosis. If blood flow is restored, these changes can be reversed, and cardiomyocyte apoptosis is avoided. Incorrect Answers: A, B, C, and D. Defects in the plasma membrane (Choice A) leading to increased plasma membrane permeability is a feature of early coagulative necrosis and indicates irreversible cellular damage that is the hallmark of myocardial infarction. Similarly, nuclear pyknosis (Choice C) describes the condensation of nuclear chromatin that occurs as the cell is undergoing apoptosis, while the rupture of lysosomes (Choice D) is also a feat

24 Exam Section 1: Item 24 of 50 National Board of Medical Examiners® Comprehensive Basic Science Self-Assessment 24. A 45-year-old woman is brought to the emergency department 30 minutes after the sudden onset of left-sided facial drooping and left arm and leg weakness. One week ago, she returned from a business trip that involved a 6-hour airplane flight. She has no history of major medical illness. She does not smoke. Her only medications are an oral contraceptive and occasional acetaminophen for headache. Her pulse is 88/min and regular, and blood pressure is 160/90 mm Hg. Physical examination shows left- sided facial paralysis. Examination of the left lower extremity shows swelling, tenderness, and localized erythema over the calf. Muscle strength is 3/5 in the left upper and lower extremities. Cardiac examination shows no abnormalities. Echocardiography is most likely to show which of the following findings in

E. This patient with a deep venous thrombosis (DVT) most likely possesses a patent foramen ovale, allowing a thromboembolism to bypass the pulmonary circulation and cause a thromboembolic cerebrovascular accident (CVA, or stroke). DVT typically presents with lower extremity edema, erythema, warmth, and pain at rest along with calf pain on dorsiflexion of the foot (Homan sign). Blood clots typically occur in patients who possess some or all of the risk factors of the Virchow triad: stasis (long flight, postoperative immobility), hypercoagulability (oral contraceptive use, hypercoagulable disorders), and endothelial damage (smoking, hypertension, atherosclerosis, injury). DVT may lead to pulmonary embolism if the clot embolizes to the right heart and subsequently to the pulmonary circulation. In patients with a patent foramen ovale (a congenital defect between the right and left atria), the clot may travel from the right atrium to the left atrium and then to the systemic circulation. This patient's clot likely embolized to the right internal carotid artery and then the right middle cerebral artery (MCA), occluding blood flow to the right primary motor cortex resulting in left-sided weakness. CVAS occur because of ischemic or hemorrhagic loss of blood supply to the brain. Approximately 80 to 85% of CVAS are ischemic, commonly arising from thromboembolic disease, whereas 15 to 20% of CVAS are hemorrhagic as a result of blood vessel rupture (eg, hypertension-related intraparenchymal hemorrhage from a perforating artery). Classically, CVAS manifest as a neurologic deficit related to the affected part of the brain. Incorrect Answers: A, B, C, and D. Aortic stenosis (Choice A), hypertrophy of the interventricular septum (Choice C), and mitral valve prolapse (Choice D) are structural abnormalities of the heart that lead to abno

59 Exam Section 2: Item 9 of 50 National Board of Medical Examiners® Comprehensive Basic Science Self-Assessment 00 9. A 4-year-old boy is brought to the physician by his mother because of a 3-month history of hyperactivity and a decreased attention span. Physical examination shows no abnormalities. Laboratory studies show: 10.5 g/dL (N=11-15) 27% (N=28%-45%) 24 pg/cell (N=25.4-34.6) 76 um3 (N=77-98) Hemoglobin Hematocrit Mean corpuscular hemoglobin Mean corpuscular volume A photomicrograph of a peripheral blood smear is shown. Which of the following is the most likely underlying cause of these findings? A) Decreased B-globin synthesis B) Decreased glucose 6-phosphate dehydrogenase activity C) Dietary deficiency of B vitamins D) Dietary deficiency of iron E) Functional asplenia F) Inhibition of ferrochelatase G) a-Spectrin mutation

F. Although lead is generally no longer used in the manufacture of paint and gasoline, children can continue to be exposed to lead through ingestion of paint chips and dust in older homes or from contaminated drinking water. Lead poisoning is characterized by basophilic stippling on peripheral blood smear (shown here) and ringed sideroblasts within the bone marrow. Lead poisoning typically affects the central and peripheral nervous systems, the heme synthesis pathway, and gastrointestinal and kidney function. This patient demonstrates neurobehavioral symptoms which may be permanent if occurring in youth. Children are susceptible to the toxic effects of lead because of an immature central nervous system and lead passage across the blood brain barrier. Acute encephalopathy, hearing loss, and peripheral neuropathy are possible neurologic complications. Clinical manifestations of lead poisoning include renal tubular dysfunction, interstitial nephritis, vomiting, abdominal pain, constipation, and anemia. Lead inhibits the enzymes ferrochelatase and õ-ALA dehydratase, resulting in decreased heme synthesis, increased serum õ-ALA, urinary coproporphyrin, and erythrocyte zinc protoporphyrin. The diagnosis is suspected on exposure and symptoms and is confirmed with an increased serum lead concentration. Management of lead poisoning depends on the blood lead concentration and presence of symptoms, and includes chelation with dimercaprol, ethylene diamine tetraacetate, or succimer. Sources of lead should be investigated and eliminated. Incorrect Answers: A, B, C, D, E, and G. Decreased B-globin synthesis (Choice A) is the cause of B-thalassemia. B-Thalassemia causes microcytic anemia, but peripheral blood smear shows microcytosis and target cells with normal iron concentrations. Patients will have an abnormal hemoglobin electrop

10 Exam Section 1: Item 10 of 50 National Board of Medical Examiners® Comprehensive Basic Science Self-Assessment 10. A 12-year-old boy is brought to the physician by his parents because of bed-wetting. He urinates 10 to 15 times each day. He says that he is constantly thirsty and drinks large quantities of liquid throughout the day. Urinalysis shows a specific gravity of 1.003. A 24-hour urine collection shows a creatinine clearance of 140 mL/min, with a total urine volume of 7600 mL and urine osmolality of 230 mOsmol/kg H,O. A tumor affecting which of the following hypothalamic nuclei is most likely involved in producing this patient's symptoms? A) Anterior B) Arcuate C) Dorsomedial D) Lateral E) Posterior F) Supraoptic G) Ventromedial

F. Central diabetes insipidus may result from a tumor involving the hypothalamic paraventricular and supraoptic nuclei and is characterized by the inadequate secretion of antidiuretic hormone (ADH, also known as vasopressin) from the posterior pituitary. Normally, increased plasma osmolality will be sensed by osmoreceptors and result in the increased production of vasopressin in the hypothalamic paraventricular and supraoptic nuclei. Vasopressin is subsequently released by the posterior pituitary gland. ADH binds V2 receptors of the late distal convoluted tubule and the collecting duct principal cells, which results in the insertion of aquaporin channels in the luminal surface and increases free water absorption. This process is typically utilized to maintain tight regulation of the serum osmolality. Inadequate production of ADH by the hypothalamus and release by the posterior pituitary gland may occur in the setting of various pathologies, such as pituitary tumors, trauma, tuberculosis, or intracranial surgery. This results in a decreased ability to absorb adequate amounts of free water within the collecting duct, which results in dilute urine and increased serum osmolality. Patients typically present with extreme thirst, production of large volumes of dilute urine, and altered mental status. Treatment requires the administration of desmopressin, an analog of vasopressin (ADH), which results in the reabsorption of free water via aquaporin insertion in the collecting duct. Incorrect Answers: A, B, C, D, E, and G. The anterior hypothalamic nucleus (Choice A) is involved in thermoregulation, specifically in cooling. The arcuate hypothalamic nucleus (Choice B) contains complex regulatory functions that are involved in appetite stimulation. The dorsomedial hypothalamic nucleus (Choice C) is involved in the regulation of fe

4 Exam Section 1: Item 4 of 50 National Board of Medical Examiners® Comprehensive Basic Science Self-Assessment 4. A female newborn is born at 39 weeks' gestation following an uneventful pregnancy. The Apgar scores are 10 at 1 and 5 minutes, respectively. The newborn is at the 50th percentile for length and 25th percentile for weight. She has normal morphologic features. The parents are told she has an enzyme defect that, if left untreated, will result in failure to attain early developmental milestones, microcephaly, hyperactivity, seizures, intellectual disability, and a mousy odor of the skin and urine. The patient's diet needs to contain which of the following to provide substrate for catecholamine production? A) Dihydroxyphenylalanine B) Dopamine C) Epinephrine D) Norepinephrine E) Phenylalanine F) Tyrosine

F. Tyrosine supplements are required for catecholamine production in this patient with phenylketonuria (PKU). PKU results from a deficiency in the enzyme phenylalanine hydroxylase (PAH) that converts phenylalanine to tyrosine. Deficiency results in accumulation of phenylalanine and its metabolites phenylacetate and phenyllactate. Rarely, mutations in enzymes that either produce or recycle tetrahydrobiopterin (BH , which is a cofactor for PAH, can have similar clinical manifestations as classic PKÚ. Phenylalanine is a precursor to tyrosine, and tyrosine is used to synthesize catecholamines such as dopamine, epinephrine, and norepinephrine. Tyrosine is converted to dihydroxyphenylalanine by tyrosine hydroxylase, followed by conversion to dopamine by DOPA-decarboxylase. Dopamine can subsequently be converted to norepinephrine, and methylation of norepinephrine results in formation of epinephrine. Patients with PKU from PAH deficiency require supplementation with tyrosine to bypass the dysfunctional enzymatic step of converting phenylalanine to tyrosine. Incorrect Answers: A, B, C, D, and E. Dihydroxyphenylalanine (Choice A), abbreviated as DOPA, is a precursor to dopamine and is formed via the action of tyrosine hydroxylase. This enzymatic pathway is functional in patients with PKU so supplementation with DOPA is not required. Similarly, dopamine (Choice B) is created from DOPA by the enzyme DOPA decarboxylase, while epinephrine (Choice C) and norepinephrine (Choice D) are made from dopamine by separate enzymes, none of which are deficient in PKU. Supplementation of tyrosine is sufficient to allow for normal synthesis of all of these catecholamines. Phenylalanine (Choice E) would worsen the symptoms of PKU as the primary cause of this condition is the inability to convert phenylalanine to tyrosine. Educational Objective:


Kaugnay na mga set ng pag-aaral

Microsoft Excel Vocats Test Modules 1-4

View Set

FIN 3110 Chapter 1, FIN 3110 Chapter 2, FIN 3110 Chapter 3, FIN 3110 Chapter 4

View Set

Fichas de Estudio para el Interpretivo 12/11/20

View Set

Personal Finance Unit 2.1 Planning Your Tax Strategy

View Set

Chapter 4 - Accident Investigation

View Set

Social Studies Chapter 5 Spirit of Independence

View Set